A New Approach To I.C.S.E. Physics Part 1 For Class Ix

You might also like

Download as pdf or txt
Download as pdf or txt
You are on page 1of 135

1 Measurements and Experimentation

UNIT I

EXERCISE 1
(A) Objective Questions
I. Multiple Choice Questions.
Select the correct option:
1. Which of the following is not a fundamental unit?
(a) second (b) ampere (c) candela (d) newton
2. Which of the following is a fundamental unit?
(a) m/s2 (b) joule (c) newton (d) metre
3.
Which is not a unit of distance?
(a) metre (b) millimetre (c) leap year (d) kilometre
Ans. 1. (d) 2. (d) 3. (c)
II. Fill in the blanks.
(i)
The unit in which we measure the quantity is called ..........................
(ii)
One light year is equal to ...........................
(iii)
One mean solar day = .......................... sec
(iv)
One year = .......................... sec
(v) One micrometre = .......................... m.
Ans. (i) standard quantity (ii) 9.46 × 1015 m (iii) 86400
(iv) One year = 3.1536 × 107  (v) 10–6

(B) Subjective Questions


1.
What do you understand by the term measurement?
Ans. Measurement is the comparison of an unknown quantity with a known standard quantity (constant quantity) or
unit.
2.
What do you understand by the terms (i) unit, (ii) magnitude, as applied to a physical quantity?
Ans. (i) The standard quantity (constant quantity), used for comparison is called unit.

(ii) The number of times a standard quantity is present in a given physical quantity, is called magnitude of physical
quantity.

Physical quantity = (magnitude) × (unit).
3. A body measures 25 m. State the unit and the magnitude of unit in the statement.
Ans. In the statement a body measures 25 m, metre is the unit and 25 is the magnitude.
4. Define the term fundamental unit. Name fundamental units of mass, length, time, current and temperature.
Ans. The units, which can be neither derived from one another nor resolved (broken) into any thing more basic are
called fundamental units.

1

Fundamental units of mass, length, time, current and temperature are kilogram, metre, second, ampere and kelvin
respectively.
5. What do you understand by the term derived unit? Give three examples.
Ans. Any unit which can be obtained by the combination of one or more fundamental units, is called derived unit.

Examples of derived unit :

Square metre (m2)
l


Metre per second (ms–1)
l

Kilogram per cubic metre (kg/m3)


l

6. (a) Define mass.


(b) State the units in which mass is measured in (i) C.G.S. system (ii) S.I. system.
(c) Name the most convenient unit of mass you will use to measure :
(i) Mass of small amount of a medicine
(ii) The grain output of a state
(iii) The bag of sugar
(iv) Mass of a cricket ball.
ns. (a) The mass of a body is defined as the quantity of matter it contains.
A
(b) (i) In C.G.S. system mass is measured in gram.
(ii) In SI system mass is measured in kilogram.
(c) (i) milligram (mg)
(ii) Tonne
(iii) kilogram
(iv) gram
7. (a) Define time.
(b) State or define the following terms :
(i) Solar day (ii) Mean solar day (iii) Hour (iv) Minute
(v) Second (vi) Year.
Ans. (a) The interval between two events is called time.
(b) (i) Solar Day : The time taken by the earth to complete one rotation about its own axis is called solar
day.
(ii) Mean solar day : The average of the varying solar days, when the earth completes one revolution
around the sun, is called mean solar day.
(iii) Hour : 1/24th part of the mean solar day is called an hour.
(iv) Minute : 1/1440 part of the mean solar day is called one minute.
(v) Second : 1/86400 part of the mean solar day is called one second.
(vi) Year : Time in which earth completes one complete revolution around the sun is called one year.
One year is equal to 3651/4 days.

UNIT II

Practice Problems 1.
1.
A student calculates experimentally the value of density of iron as 7.4 gcm–3­. If the actual density of iron is
7.6 gcm–3, calculate the percentage error in experiment.

2
Solution :
Absolute error = (7.6 – 7.4) gcm–3 = 0.2 gcm–3
Absolute error 0.2
Percentage error =
¥ 100 = ¥ 100 = 2.63% .
Actual value 7.6

2.
A student finds that boiling point of water in a particular experiment is 97.8°C. If the actual boiling point of
water is 99.4°C, calculate the percentage error.
Solution :

Absolute error = (99.4°C – 97.8°C) = 1.6°C

Percentage error = Absolute error ¥ 100 = 1.6 ¥ 100 = 1.61%


Actual value 99.4
3.
A pupil determines velocity of sound such 320 ms–1. If actual velocity of sound is 332 ms–1, calculate the
percentage error.
Solution :

Absolute error = (332 – 320) ms–1 = 12 ms–1

Percentage error = Absolute error ¥ 100 = 12 ¥ 100 = 3.61%



Actual error 332
EXERCISE 2
1. (a) What do you understand by the term order of magnitude of a quantity?
(b) Why are physical quantities expressed in the order of magnitude? Support your answer by an example.
Ans. (a) The exponential part of a particular measurement is called order of magnitude of a quantity. For example,
The diameters of Sun = 1.39 × 109 m.
Here the order of magnitude of diameter of Sun is 109 m.
(b) If we use the standard scientific unit metre for measuring length of bigger objects like diameter of Sun, it
will be described by a very large number 1,390,000,000 m. Conversely, very small objects like the diameter
of hydrogen atom is 0.000,000,000,106 m.
The above numbers are exceptionally big and are difficult to write or remember. So scientists have found
an easy and compact way of writing such numbers. Any number can be written as product of a number
between 1 and 10, and a number which is a power of 10 known as the order of magnitude of a quantities.
Thus, diameter of Sun, 1,390,000,000 m = 1.39 × 109 m
Here 109 m is the order of magnitude of diameter of the Sun.
2. Express the order of magnitude of the following quantities:
(i) 12578935 m (ii) 222444888 kg (iii) 0.000,000,127 s (iv) 0.000,000,000,00027 m
Ans. (i) 12578935 ~ 1.2 × 107 m
So, order of magnitude is 107 m
(ii) 222444888 kg ~ 2.2 × 108 kg
So, order of magnitude 108 kg
(iii) 0.000,000,127 s = 1.27 × 10–7 s
So, order of magnitude is 10–7 s
(iv) 0.000,000,000,00027 m = 2.7 × 10–13 m
So, order of magnitude = 10–13 m.

3
3. (a) What do you understand by the term degree of accuracy?
(b) Amongst the various physical measurements recorded in an experiment, which physical measurement
determines the degree of accuracy?
Ans. (a) From degree of accuracy we mean how far we can measure a quantity, without any error of estimation.
(b) In any experiment, all observations should be taken with same degree of accuracy. In a given experiment, it
is no use taking one observation of very high degree of accuracy than the rest of observations. It is because
it will not make final result more accurate.
It is observed that the accuracy of the result of an experiment is same as that of least accurate observation.
From the above statement, it should not mean that observations should not be taken very accurately. On the
other hand, a careful sense of proportion should be used.
4. (a) State the formula for calculating percentage error.
(b) Is it possible to increase the degree of accuracy by mathematical manipulations? Support your answer by
an example.
Ans. (a) The percentage error can be calculated by the following formula
Absolute error
Percentage error = ¥ 100
Actual value
(b) It is not possible to produce greater accuracy by mathematical manipulations, like addition, subtraction,
multiplication, division, etc. The examples given below illustrate the point.
Addition and Subtraction
When a number of values are added or subtracted, the result cannot be more accurate than the least
accurate value.
If we add :
5 . 2 8 3
7 2 . 5
+ 2 . 0 0 0 1 4
we get, 7 9 . 7 8 3 1 4

In the above addition the value 72.5 has least accuracy. When we say 72.5, it implies that value lies between
72.45 and 72.55 and 72.5 is the most probable value. Thus, the error in 72.5 is ± 0.05.
As the final result cannot be more accurate than the least accurate observation, therefore, it is absurd to put
the answer 79.78314. The correct and most reliable answer is 79.7.
5. State the factors which determine number of significant figures for the calculation of final result of an experiment.
Ans. The number of significant figures to which final result of an experiment should be stated depends upon the
following factors :
(i) The nature of experiment
(ii) The accuracy with which various measurements are made.
6. The final result of calculations in an experiment is 125,347,200. Express the number in terms of significant places
when
(i) accuracy is between 1 and 10
(ii) accuracy is between 1 and 100
(iii) accuracy is between 1 and 1000
Ans. Final result in an experiment is 125,347,200
(i) Accuracy is between 1 and 10 = 1.2 × 108
(ii) Accuracy is between 1 and 100 = 1.25 × 108
(iii) Accuracy is between 1 and 1000 = 1.253 × 108

4
UNIT III
Practice Problems 1.
1. The main scale of vernier callipers has 10 divisions in a centimetre and 10 vernier scale divisions coincide with
9 main scale divisions. Calculate (i) pitch (ii) L.C. of vernier callipers.
Solution :
1 unit of main scale 1 cm
(i) Pitch = = = 0.1 cm
No. of divisions in the unit 100
Pitch 0.1
(ii) Least count = = cm = 0.01 cm
No. of V.S.D. 10

2.
In a vernier callipers 19 main scale divisions coincide with 20 vernier scale divisions. If the main scale has 20
divisions in a centimetre, calculate (i) pitch (ii) L.C., of vernier callipers.
Solution :
1 unit of main scale 1 cm
(i) Pitch = = = 0.05 cm
No. of divisions in the unit 20

(ii) Least count = Pitch 0.05


= cm = 0.0025 cm
No. of V.S.D. 20

Practice Problems 2.
1. Figure 1.9 shows the position of vernier scale, while measuring the external length of a wooden cylinder.
(i) What is the length recorded by main scale?

Fig. 1.9

(ii) Which reading of vernier scale coincides with main scale?


(iii) Calculate the length.
Solution :
(i) Length recorded by main scale = 10.2 cm
(ii) 7th reading of vernier scale coincides with main scale.
(iii) Pitch = 1 unit of main scale 1
= cm = 0..01 cm
No. of divisions in the unit 10
Pitch 0.1 cm
Least count = = = 0.01 cm
V.S. division 10

Reading shown by the instrument i.e. length
= Main scale reading + L.C. × V.S.D.
= 10.2 + 0.01 × 7
= 10.27 cm.

5
2.
In figure 1.10 for vernier callipers, calculate the length recorded.

Fig. 1.10

Solution :
Least count = 0.01 cm (same as recorded in solution 1)
Length recorded = Main scale reading + L.C. × V.S.D.
= 7.3 + 0.01 × 5
= 7.35 cm
Practice Problems 3
1. (a) A vernier scale has 10 divisions. It slides over a main scale, whose pitch is 1.0 mm. If the number of divisions
on the left hand of zero of the vernier scale on the main scale is 56 and the 8th vernier scale division coincides
with the main scale, calculate the length in centimetres.
(b) If the above instrument has a negative error of 0.07 cm, calculate corrected length.
Solution :

(a) Pitch = 1.0 mm = 0.1 cm
Number of divisions on the left hand of zero of the vernier scale = 56
Hence main scale reading = 56 × 0.1 cm = 5.6 cm
Pitch 0.1
Least count = = = 0.01 cm
V.S. division 10
Length shown by the instrument
= Main scale reading + L.C. × V.S.D.
= 5.6 + 0.01 × 8 = 5.68 cm

(b) As the error is negative, therefore, correction will be positive.
\ Correct length = (5.68 + 0.07) cm = 5.75 cm
2. (a) A vernier scale has 20 divisions. It slides over a main scale, whose pitch is 0.5 mm. If the number of divisions
on the left hand of the zero of vernier on the main scale is 38 and the 18th vernier scale division coincides
with main scale, calculate the diameter of the sphere, held in the jaws of vernier callipers.
(b) If the vernier has a negative error of 0.04 cm, calculate the corrected diameter of sphere.
Solution :

(a) Pitch = 0.5 mm = 0.05 cm
Main scale reading = 38 × 0.05 cm
= 1.90 cm
Pitch 0.05
Least count = = = 0.0025 cm
V.S. division 20
\ Diameter of the sphere = 1.90 + 18 × 0.0025 = 1.90 + 0.045
= 1.945 cm

(b) As the error is negative, therefore, correction will be positive.
\ Correct diameter = 1.945 + 0.04 cm = 1.985 cm

6
Practice Problems 4
1.
The least count of a vernier callipers is 0.0025 cm and it has an error of + 0.0125 cm. While measuring the
length of a cylinder, the reading on main scale is 7.55 cm, and 12th vernier scale division coincides with main
scale. Calculate the corrected length.
Solution :
Least count = 0.0025 cm
Error = + 0.0125 cm
\ Correction = – 0.0125 cm
Main scale reading = 7.55 cm
Corrected length = (Main scale reading + L.C. × V.S.D.) – Correction

= (7.55 + 0.0025 × 12) – 0.0125

= (7.55 + 0.03) – 0.0125

= 7.58 – 0.0125 = 7.5675 cm
2. The least count of a vernier callipers is 0.01 cm and it has an error of + 0.07 cm. While measuring the radius of
a sphere, the main scale reading is 2.90 cm and the 5th vernier scale division coincides with main scale. Calculate
the corrected radius.
Solution :
Least count = 0.01 cm
Error = + 0.07 cm
\ Correction = – 0.07 cm
Main scale reading = 2.90 cm
Corrected diameter = (Main scale reading + L.C. × V.S.D.) – Correction

= (2.90 + 0.01 × 5) – 0.07

= 2.88 cm
Diameter 2.88

Corrected radius = = = 1.44 cm
2 2

EXERCISE 3
1. Who invented vernier callipers?
Ans. Vernier callipers was invented by Pierre Vernier.
2. What is the need for measuring length with vernier callipers?
Ans. For measuring the exact length with greater accuracy upto 0.01 cm (especially when we are measuring a very
small length), vernier callipers is needed.
3. Up to how many decimal places can a common vernier callipers measure the length in cm?
Ans. Up to 2 decimal places a common vernier callipers can measure the length in cm.
4. Define the terms (i) pitch (ii) least count as applied to a vernier callipers.
Ans.
(i) Pitch : The smallest value of length or any other unit which can be read directly from a main scale accurately
is called pitch.
For example, if one centimetre length has ten divisions then pitch is 1/10 cm = 0.1 cm.
However, if one centimetre length has 20 divisions then pitch is 1/20 cm = 0.05 cm.
(ii) Least Count : The magnitude of the smallest measurement which can be measured by an instrument
accurately is called its least count (L.C.).

7
or
The difference between one main scale division and one vernier scale division is called least count.
5.
State the formula for determining (a) pitch (b) least count for a vernier callipers.
(a) The pitch of a vernier callipers can be given by the formula :
1 Unit (length)
Pitch =
Number of divisions in the unit
(b) The least count of a vernier callipers can be found by the following formula :
Smallest value of one M.S.D.
L.C. =
Number of V.S.D.
  or
Pitch
L.C. =
Number of V.S.D.
6. State the formula for calculating length if :
(i) Number of vernier scale division coinciding with main scale and number of division of main scale
on left hand side of zero of vernier scale are known.
(ii) The reading of main scale is known and the number of vernier scale divisions coinciding with
main scale are known.
Ans. (i) Length = (no. of divisions of main scale on the left of zero × pitch) + L.C. × Vernier scale division
(ii) Length = Main scale reading + L.C. × V.S.D.
7. (a) What do you understand by the term zero error?
(b) When does a vernier callipers has (i) positive error (ii) negative error?
(c) State the correction if (i) positive error is 7 divisions (ii) negative error is 7 divisions, when the least count
is 0.01 cm.
Ans. (a) For a correct measurement, the zero of the main scale must coincide with the zero of the vernier scale,
when the movable jaw is in contact with the fixed jaw.
However, if the zeroes of M.S. and V.S. do not coincide, the vernier is said to have an error, known as
zero error.
(b) (i) Positive Zero Error : If the zero of vernier scale is on right hand side of zero of main scale, the error
is said to be positive and the correction is said to be negative.
(ii) Negative Zero Error : If the zero of vernier scale is on the left hand side of zero of the main scale,
the error is said to be negative and the correction is said to be positive.
(c) (i) For positive zero error
Correction = – Coinciding division of V.S. × L.C.
= – 7 × 0.01 cm
= – 0.07 cm
(ii) For negative zero error
Correction = + (n – coinciding divisions of V.S) × L.C.;
Where n is the total no. of vernier scale divisions.
\ Correction = + [10 – 7] × 0.01 cm = 0.03 cm.

UNIT IV
Practice Problems 1
1.
The circular scale of a screw gauge has 50 divisions. Its spindle moves by 2 mm on sleeve, when given four
complete rotations calculate (i) pitch (ii) least count

8
Solution :
Distance moved by screw on sleeve 2 mm
(i) Pitch = = = 0.5 mm = 0.05 cm
No. of complete rotations 4

Pitch 0.05 cm
(ii) L.C. = = = 0.001 cm
No. of circular scale divisions 50

2.
The circular scale of a screw gauge has 100 divisions. Its spindle moves forward by 2.5 mm when given five
complete turns. Calculate (i) Pitch and (ii) least count of the screw gauge.
Solution :
Distance moved by screw on sleeve 2.5 mm
(i) Pitch = = = 0.5 mm = 0.05 cm
No. of complete rotations 5

Pitch 0.05 cm
(ii) L.C. = = = 0.0005 cm
No. of circular scale divisions 50

Practice Problems 2
1.
Figure 1.16 shows a screw gauge in which circular scale has 200 divisions. Calculate the least count and radius
of wire.

Fig. 1.16

Solution :
Pitch 1 mm 0.1

L.C. = = = cm = 0.0005 cm
No. of circular scale divisions 200 200


Diameter of the wire = M.S. reading + L.C. × C.S.D.

= 0.5 + 0.0005 × 34 = 0.5 + 0.0170

= 0.5170 cm
0.5170

\ Radius of the wire = = 0.2585 cm
2

2.
Figure 1.17 shows a screw gauge in which circular scale has 100 divisions. Calculate the least count and the
diameter of a wire.

Fig. 1.17

9
Solution :
Pitch 0.5 mm

L.C. = = = 0.005 mm = 0.0005 cm
No. of circular scale divisions 100
Diameter of the wire = M.S. reading + L.C. × C.S.R.
= 0.45 cm + 0.0005 × 73 cm
= 0.45 + 0.0365 = 0.4865 cm
Practice Problems 3
1.
A micrometre screw gauge having a positive zero error of 5 divisions is used to measure diameter of wire, when
reading on main scale is 3rd division and 48th circular scale division coincides with base line. If the micrometre
has 10 divisions to a centimetre on main scale and 100 divisions on circular scale, calculate
(i) Pitch of screw (ii) Least count of screw (iii)
Observed diameter (iv) Corrected diameter.
Solution :
Unit 1 cm
(i) Pitch = = = 0.1 cm
No. of division in unit 10

(ii) Least count of the screw = Pitch 0.1 cm


= = 0.001 cm
C.S. divisions 100
(iii) Main scale reading = 3 div × 0.1 cm = 0.3 cm
Observed diameter = M.S. Reading + L.C. × C.S. Reading
= 0.3 cm + 0.001 cm × 48
= 0.3 + 0.048 = 0.348 cm
(iv) Error = + 5 divisions
\ Corrections = – (Error × L.C.)
= – (5 × 0.001 cm)
= – 0.005 cm
\ Corrected diameter = (0.348 – 0.005) cm
= 0.343 cm
2.
A micrometre screw gauge has a positive zero error of 7 divisions, such that its main scale is marked in 1/2 mm
and the circular scale has 100 divisions. The spindle of the screw advances by 1 division complete rotation.

If this screw gauge reading is 9 divisions on main scale and 67 divisions on circular scale for the diameter of a
thin wire, calculate
(i) Pitch (ii) L.C. (iii) Observed diameter (iv) Corrected diameter.
Solution :
1
(i) Pitch = mm = 0.5 mm = 0.05 cm
2
Pitch 0.05
(ii) Least count = = = 0.0005 cm
C.S. divisions 100
(iii) Main scale reading = 0.9 × 0.05 cm = 0.45 cm
Observed reading = M.S. Reading + L.C. × C.S. Reading
= 0.45 + 0.0005 × 67
= 0.45 + 0.0335
= 0.4835 cm
(iv) Error = +7 divisions
\ Correction = – (Error × L.C)
= – (7 × 0.0005)
= – 0.0035 cm
\ Corrected diameter = (0.4835 – 0.0035) cm = 0.4800 cm
10
3.
The thimble of a screw gauge has 50 divisions for one rotation. The spindle advances 1 mm when screw is turned
through two rotations
(i) What is the pitch of screw?
(ii) What is the least count of screw gauge?
(iii) When the screw gauge is used to measure the diameter of wire the reading on sleeve is found to be
0.5 mm and reading on thimble is found 27 divisions. What is the diameter of wire in centimetres?
Solution :
1
(i) Pitch of the screw = mm = 0.5 mm
2
Pitch 0.5
(ii) Least count of the screw gauge = = = 0.01 mm = 0.001 cm
C.S. divisions 50
(iii) Reading on sleeve = 0.5 mm = 0.05 cm
Thus, diameter of wire = M.S. Reading + L.C. × C.S. Reading
= 0.05 cm + 0.001 × 27 cm
= 0.05 cm + 0.027 cm
= 0.077 cm
Practice Problems 4
1.
A micrometre screw gauge has a negative zero error of 8 divisions. While measuring the diameter of a wire the
reading on main scale is 3 divisions and 24th circular scale division coincides with base line.

If the number of divisions on the main scale are 20 to a centimetre and circular scale has 50 divisions, calculate
(i) pitch (ii) least count (iii) observed diameter (iv) corrected diameter.
Solution :
Unit 1 cm
(i) Pitch = = = 0.05 cm
No. of divisions in unit 20
Pitch 0.05 cm
(ii) Least count = = = 0.001 cm
Circular scale divisions 50

(iii) Observed diameter = M.S.D. × Pitch + L.C. × C.S.D. reading


= 3 × 0.05 + 0.001 × 24 cm
= 0.15 cm + 0.024 cm
= 0.174 cm
(iv) Error = –8 divisions
\ Correction = +8 × 0.001 cm = 0.008 cm
\ Corrected diameter = 0.174 + 0.008 cm = 0.182 cm

2. A micrometre screw gauge has a negative zero error of 7 divisions. While measuring the diameter of a wire the
reading on main scale is 2 divisions and 79th circular scale division coincides with base line.

If the number of divisions on main scale is 10 to a centimetre and circular scale has 100 divisions, calculate
(i) pitch (ii) least count (iii) observed diameter (iv) corrected diameter.
Solution :
Unit 1 cm
(i) Pitch = = = 0.1 cm
No. of divisions in unit 10
Pitch 0.1 cm
(ii) Least count = = = 0.001 cm
C.S. divisions 100

11
(iii) Observed diameter
= M.S.D. × Pitch + L.C. × C.S.D. reading
= 2 × 0.1 cm + 0.001 × 79 cm
= 0.2 cm + 0.079 cm
= 0.279 cm
(iv) Error = –7 divisions
\ Corrections = +7 × 0.001 cm = 0.007 cm
Hence, corrected diameter = 0.279 + 0.007 cm = 0.286 cm

EXERCISE 4
1. For what range of measurement is micrometre screw gauge used?
Ans. Micrometre screw gauge is used to measure with an accuracy upto 0.001 cm.
2. What is the function of ratchet in screw gauge?
Ans. The ratchet is attached to screw by means of a spring. When the flattened end of the screw comes in contact with
stud, the ratchet becomes free and makes a rattling noise. Thus, end of the screw is not further pushed towards
the stud.
3. What do you understand by the terms (a) pitch of screw (b) least count of screw?
Ans. (a) The pitch of screw is defined as, the distance between two consecutive threads of screw, measured along
the axis of screw.
Pitch of screw can also be defined as the distance travelled by the tip of screw when head of screw is given
one complete rotation.
(b) Least count of the screw is the smallest distance moved by its tip when the screw turns through 1 division
marked on it.
4. State the formula for calculating (i) pitch of screw (ii) least count of screw.
Ans.
(i) The pitch can be found by the formula;
Distance moved by thimble on M.S.
Pitch = [M.S. stands for main scale]
Number of rotations of thiimble
If 5 mm is the distance moved by the thimble on the main scale for 5 rotations, then :
5 mm
Pitch = = 1 mm.
5
(ii) The least count is determined by the formula :
Pitch

L.C. =   [C.S. stands for circular scale]
Number of C.S. divisions
If pitch of screw is 1 mm and number of divisions marked on its thimble are 100, then :
1 mm
L.C. = = 0.01 mm = 0.001 cm
100
5. What do you understand by the following terms as applied to screw gauge?
(a) Zero error (b) Positive zero error (c) Negative zero error.
Ans. (a) If the zero of the main scale does not coincide with zero of circular scale on bringing the screw end in
contact with stud, the micrometre is said to have zero error.
(b) If the zero line marked on circular scale, is below the reference line of the main scale, then there is a
positive zero error and the correction is negative.
(c) If the zero line marked on circular scale, is above the reference line of the main scale, then there is a
negative zero error and the correction is positive.

12
6. How do you account for (a) positive zero error (b) negative zero error, for calculating correct diameter of wires?
Ans. (a) If the error is positive, then the correction will be negative.
For example, if a screw gauge having a positive zero error of 5 divisions then correction is negative.
Let the L.C. be 0.001 cm of a screw gauge
Correction = – coinciding divisions of C.S. × L.C.
= – 5 × 0.001 = – 0.005 cm
If the observed reading of a diameter is 0.568 cm then the corrected diameter
= Observed diameter – Correction
= (0.568 – 0.005) cm
= 0.563 cm
(b) If the error is negative then the correction will be positive.
For example if a screw gauge having a negative zero error of 97 divisions then the correction is positive.
Let the L.C. be 0.001 cm of a screw gauge.
= +[n – coinciding division of C.S. × L.C.]
\ Correction = + [100 – 97] × 0.001 cm = 0.003 cm
Let the observed diameter = 0.593 cm
Then corrected diameter = Observed diameter + Correction
= (0.593 + 0.003) cm
= 0.596 cm.

UNIT V

EXERCISE 5
1. (a) What do you understand by the term volume of a substance?
(b) State the unit of volume in SI system.
Ans. (a) The space occupied by a substance (solid, liquid or gas) is called volume.
(b) The unit of volume in SI system is cubic metre. i.e., m3, one cubic metre is the volume occupied by a cube
whose each side is equal to 1 m.
2. How is SI system of unit of volume is related to 1 litre? Explain.
Ans. One litre = 1000 cc = 1000 ml

In SI system unit of volume is m3.

We know that

1 m3 = 106 cm3 Q 1 m = 100 cm.

or, 1 cm3 = 10–6 m3

or, 1000 cm3 = 1000 × 10–6 m3

or, 1 litre = 10–3 m3

\ 1 m3 = 103 litre = 1000 litre.
3. In which unit, volume of liquid is measured? How is this unit is related to S.I. unit of volume?
Ans. Generally, the volume of liquid is measured in litre (l).

1 litre = 10–6 m3
4. Explain the method in steps to find the volume of an irregular solid with the help of measuring cylinder.
Ans. Assumptions :

1. Solid is heavier than water.

13

2. The solid is not soluble in water.

The volume of irregular solid body is determined with the help of measuring cylinder. As solid body displaces
water of its own volume.

(i) Take a measuring cylinder and fill water up to certain level. Note down the level of water in measuring
cylinder. Let it be V1.
(ii) Tie the irregular solid body with a thin and strong thread and lower the body gently so that the solid body is
completely immersed in the water. The level of water rises. Solid body displaces water of its own volume.
Note down the new level of water. Let it be V2.
(iii) Take the difference of two levels of water, i.e., (V2 – V1). This will give the volume of irregular solid body.
5. Amongst the units of volume (i) cm3 (ii) m3 (iii) litre (iv) millilitre, which is most suitable for measuring :
(a) Volume of a swimming tank (b) Volume of a glass filled with milk
(c) Volume of an exercise book (d) Volume of air in the room.
Ans. (a) litre
(b) millilitre
(c) cm3
(d) m3
6. Find the volume of a book of length 25 cm, breadth 18 cm and height 2 cm in m3.
Solution :

Length of the book = 25 cm = 0.25 m

Breadth of the book = 18 cm = 0.18 m

Height of the book = 2 cm = 0.02 m

Volume of the book = Length × Breadth × Height

= 0.25 × 0.18 × 0.02 m3

= 0.0009 m3
7. The level of water in a measuring cylinder is 12.5 ml. When a stone is lowered in it, the volume is 21.0 ml. Find
the volume of the stone.
Solution :

Volume of the stone = Difference of two levels of water

= 21.0 ml – 12.5 ml

= 8.5 ml

14
8. A measuring cylinder is filled with water upto a level of 30 ml. A solid body is immersed in it so that the level
of water rises to 37 ml. Now solid body is tied with a cork and then immersed in water so that the water level
rises to 40 ml. Find the volume of solid body and the cork.
Solution :

Volume of the solid body = Difference of two levels of water

= (37 – 30) cm3 = 7 cm3

Volume of the cork = (40 – 37) cm3 = 3 cm3.

UNIT VI

Practice Problems 1
1.
Calculate the time period of simple pendulum of length 0.84 m when g = 9.8 ms–2.
Solution :

Length l = 0.84 m; g = 9.8 ms–2; T =?
l 2 ¥ 22 0.84 44 84 44 3 44 44 ¥ 0.29 12.86
T = 2p = ¥ = ¥ = ¥ = ¥ 0.086 = = = 1.82 s
g 7 9.8 7 980 7 35 7 7 7

2.
Calculate the time period of simple pendulum of length 1.44 m on the surface of moon. The acceleration due to
gravity on the surface of moon is 1/6 the acceleration due to gravity on earth. [g = 9.8 ms–2].
Solution :

Let Tm = Time period on the moon

l = length of the pendulum = 1.44 m

gm = Acceleration due to gravity at moon
= 9.8/6 ms–2

l
Tm = 2p

gm

1.44 ¥ 6
= 2p
9.8

22 8.64 44 44 × 0.94 41.36
= 2¥ = ¥ 0.8816 = =
7
9.8 7 7 7

= 5.90 s
Practice Problems 2
1. Length of second’s pendulum is 100 cm. Find the length of another pendulum whose time period is 2.4 s.
Solution :
Let length of the second pendulum l2

Time period = T2
T1 l1
=
T2 l2


2 100 1 10

Þ = fi =
2.4 l2 1.2 l2

15

Squaring both sides, we get
1 100
fi =
1.44 l2
fi l = 144 cm
2
2.
A pendulum of length 36 cm has time period 1.2 s. Find the time period of another pendulum, whose length is
81 cm.
Solution :

Length of pendulum = l1= 36 cm

Time period = T1 = 1.2 s

Length of another pendulum l2 = 81 cm

Time period = T2 =?
T1 l1
=
T2 l2


1.2 36
fi =
T2 81
1.2 6 2
fi = =
T2 9 3
1.2 ¥ 3
fi T2 = = 1.8 s
2
3.
Calculate the length of second’s pendulum on the surface of moon when acceleration due to gravity on moon is
1.63 ms–2.
Solution :

Let l be the length of the second pendulum
l
T = 2p Where g m Æ Acceleration due to gravity on moon
gm

l 22 l
2 = 2p fi 1=
1.63 7 1.63

On squaring both sides, we get
l 7¥7
=
1.63 22 ¥ 22
49 ¥ 1.63 79.87
fi l = = = 0.165 m
484 484

Practice Problems 3
1.
The lengths of two pendulums are 110 cm and 27.5 cm. Calculate the ratio of their time periods.
Solution :
T1 l1 T1 110 4 2
= fi = = =
T2 l2 T2 27.5 1 1


\ Ratio of their time periods = 2 : 1.

16
2.
A pendulum 100 cm and another pendulum 4 cm long are oscillating at the same time. Calculate the ratio of their
time periods.
Solution :
T1 l1 T1 100
= fi = =5
T2 l2 T2 4


\ Ratio of their time periods = 5 : 1.
Practice Problems 4
1.
If the time periods of two pendulums are 1.44 s and 0.36 s respectively. Calculate the ratio of their lengths.
Solution :
Let l1 and l2 be their lengths and T1 and T2 be their time periods respectively.
T1 l 1.44 s l1 l1
\ = 1 fi = fi 4 =
T2 l2 0.36 s l2 l2


On squaring both sides,
l
fi 1 = 16
l
2

Hence, ratio of their lengths = 16 : 1.

2.
The time period of two pendulums are 2 s and 3 s respectively. Find the ratio of their lengths.
Solution :

Let l1 and l2 and T1 and T2 be their length and time periods respectively.
T1 l1
=
T2 l2

T12 l1
On squaring both sides =
T2 2 l2

l1 T2 22 4
Or, = 12 = 2 =
l2 T2 3 9


Hence, ratio of their lengths = 4 : 9
EXERCISE 6
1. (a) Define simple pendulum.
(b) State two factors which determine time period of a simple pendulum.
(c) Write an expression for the time period of a simple pendulum.
Ans. (a) A simple pendulum is a device used for measuring time. It consists of a heavy particles, suspended by a
weighless and inextensible string and oscillating freely without friction about a point to which the upper
end of the string is fixed.
(b) Two factors on which time period depends :
1. Length of pendulum : The time period varies directly with the square root of the effective length
(the length between the point of suspension and the centre of the bob).
If the effective length of the pendulum is l and the time period T, then

17
T µ l
2. Acceleration due to gravity : The time period varies inversely with the square root of the
acceleration due to gravity.
If the time period be T and the acceleration due to gravity at a place be g, then
1
T∝
g
(c) Expression for the time period of simple pendulum :
The time period of oscillation of a simple pendulum is directly proportional to the square root of its effective
length.
i.e. T µ l …(i) where T ® time period
l ® effective length of the pendulum.
The time period of oscillation is inversely proportional to the square root of acceleration due to gravity i.e.
1
i.e. T µ …(ii)
g
Where g = acceleration due to gravity
Combining (i) and (ii), we get
l l
T µ or T = 2p
g g

Where 2p is the constant of proportionality.
2. Define the following in connection with a simple pendulum :
(a) Time period (b) Oscillation (c) Amplitude (d) Effective length.
Ans. Time Period : The time taken by the pendulum to make one complete oscillation.

Oscillation : The complete to and fro motion of the pendulum constitutes one complete oscillation.
Amplitude : The maximum displacement of a pendulum from its mean position is called amplitude.
Effective Length : The length between point of suspension and centre of gravity of bob of a pendulum is called
effective length.
3. (a) What is a second’s pendulum?
(b) A second’s pendulum is taken on the surface of moon where acceleration due to gravity is
1
th
6 of that of earth. Will the time period of pendulum remain same or increase or decrease? Give a reason.
Ans. (a) A pendulum which has a time period (T) of two seconds, is called a second’s pendulum.
(b) The time period will increase.
Since the time period varies inversely with the square root of the aceleration due to gravity. So on decreasing
the acceleration due to gravity, time period increases.
1
T µ
g
4. Which of the following do not affect the time period of a simple pendulum?
(a) mass of bob (b) size of bob (c) effective length of pendulum (d) acceleration due to gravity
(e) amplitude.
Ans. The time period of a simple pendulum is independent of the (a) mass of the bob, (b) size of bob. (c) Time
period is directly proportional to the square root of effective length of the pendulum. (d) Time period is inversely
proportional to the square root of acceleration due to gravity. The time period is independent of the amplitude,
provided that the amplitude is not too large.

18
5. A simple pendulum is hollow from within and its time period is T. How is the time period of pendulum affected
when :
(a) 1/4 of bob is filled with mercury (b) 3/4 of bob is filled with mercury
(c) the bob is completely filled with mercury?
Ans. (a) When 1/4 of the bob is filled with mercury the effective length of the pendulum increases so the time period
increases, as T µ l .
(b) When the 3/4 of the bob is filled with mercury the effective length of the pendulum decrease, so the time
period also decreases. as T µ l .
(c) When the bob is completely filled with mercury there is no change in effective length of the pendulum
hence, there is no change in time period.
6. Two simple pendulums A and B have equal lengths but their bobs weigh 50 gf and 100 gf respectively. What
would be the ratio of their time periods? What is the reason for your answer?
Ans. The ratio of the time periods of A and B = 1 : 1.
It is because time period is independent of the mass of the bob.
7. State the numerical value of the frequency of oscillation of a second’s pendulum. Does it depend on the amplitude
of oscillation?
Ans. Time period (T) of a second pendulum is 2 s.
1 1
Frequency = = Hz = 0.5 Hz.
Time period 2

Frequency of the second pendulum is independent of the amplitude of oscillation, provided this is not too large.
As a matter of fact amplitude should not exceed 5°.
8. (a) Name the two factors on which time period of a simple pendulum depends.
(b) Name the devices commonly used to measure (i) mass and (ii) weight of a body.
Ans. (a) Two factors on which the time period of a simple pendulum depends are
(i) Length of the pendulum (ii) Acceleration due to gravity
(b) (i) A beam balance is used commonly to measure mass of a body.
(ii) Spring balance is used to measure weight, of an object.
9. Draw a graph of l, the length of simple pendulum against T2, the square of its time period.
Ans. An experiment is performed with a simple pendulum for different length, it is observed that the more the length,
the more is the time period.
Length (cm) 120 110 100 90 80
Time (seconds) 2.20 2.11 2.00 1.91 1.80

Table for l and T2
l (in cm) 120 110 100 90 80
T2 (in seconds) 4.84 4.45 4.0 3.65 3.24


From the graph it is clear that T2 ∝ l.
19
10. What do you understand by
(a) amplitude and (b) frequency of oscillations of simple pendulum?
Ans. (a) The maximum displacement of a pendulum from its mean position is called amplitude.
(b) The number of oscillations made by pendulum in one second is called the frequency of oscillations.
EXERCISE 7
1. (a) What do you understand by the term graph?
(b) What do you understand by the terms (i) independent variable (ii) dependent variable?
(c) Amongst the independent variable and dependent variable, which is plotted on X-axis?
Ans. (a) A pictorial representation of two physical variables, recorded by an experimenter is called graph.
(b) (i) A variable whose value does not depend on the other variable is called independent variable.
(ii) A variable whose value depends on the independent variable is called dependent variable.
(c) The independent variable is always plotted on X-axis.
2. (a) State how will you choose a scale for the graph.
(b) State the two ratios of a scale, which are suitable for plotting points.
(c) State the two ratios of a scale, which are not suitable for plotting points.
Ans. (a) Choose any convenient scale to represent a given variable on a given axis, such that whole range of variations
are well spread out on the whole graph paper, to give the graph line a suitable size.
For this a round number, nearest to or slightly less than minimum value should be taken as the origin.
Similarly, a round number nearest to or slightly more than the maximum value should be taken at far end
of the respective axis for a given variable.
Scale on X-axis : Count the number of small squares available on the X-axis and the number of points
to be plotted on X-axis. By dividing available squares by number of points we can find scale on X-axis.
Scale on Y-axis : Count the number of available squares on Y-axis and divide the squares with number of
points to be plotted. This gives the scale on Y-axis.
(b) The scale chosen should be suitable for plotting all points. The scale ratio 1 : 2, 1 : 4 are suitable for
plotting points.
(c) Scale ratio 1 : 3 or 1 : 7 are not suitable for plotting all points.
3. State three important precautions which must be followed while plotting points on a graph.
Ans. Following are three important precautions while plotting points on a graph :
(i) No guidelines must be drawn on graph paper as an aid to plot points.
(ii) The points marked on graph paper should be sharp, but not thick.
(iii) The points should be encircled by a small circle.
4. State two important precautions for drawing a graph line.
Ans. Precaution for drawing a graph line.
(i) The graph line should be thin, single straight line and sharp.
(ii) It is not necessary that graph line should pass through all points. As a matter of fact, a best fit line should
be drawn.
5. (a) What is a best fit line for a graph?
(b) What does best fit line show regarding the variables plotted and the work of experimenter?
Ans. (a) From best fit line we mean the line which either passes through maximum number of points or passes
closest to maximum number of points, which appear on either side of the line.
(b) Graph line helps us to determine the nature of proportional relationship between two variable quantities.
6. (a) What do you understand by the term constant of proportionality?
(b) How can proportionality constant be determined from the best fit straight line graph?

20
Ans. (a) Constant of proportionality is the constant value of the ratio of two proportional quantities. Quantities x and
y; usually written y = kx. Where k is the constant of proportionality. x and y are variable quantities.
(b) The constant of proportionality can be determined from straight line graph, by calculating the slope of graph.
7. State three uses of graph.
Ans. l Graphs can be used to calculate mean average value of large number of observations.

l Graphs can be used for verifying already known physical laws.


l A straight line graph shows that two variable quantities are directly proportional to one another.

8. How does a graph help in determining the proportional relationship between two quantities?
Ans. If a graph is plotted between length and time period of a simple
pendulum, the graph line is a curve, which has a tendency to meet
x-axis or y-axis when produced towards origin.

From the graph, it is clear that length of a simple pendulum is
not directly proportional to its time periods.

However, if a graph is plotted between length and (Time)2, the



graph line is a straight line. Thus we can say
Length ∝ (Time)2.


Thus, from above examples it is clear that graph line helps to
determine the nature of proportional relationship between two
variable quantities.

EEEEE

21
2 Motion in One Dimension

UNIT I
EXERCISE 1
1.
What do you understand by the term (i) rest (ii) motion? Support your answer by giving two examples each.
Ans. (i) When a body does not change its position with respect to its surroundings the body is said to be at rest.
For example a table lying in a room is in the state of rest, because it does not change its position with
respect to the surrounding of the room.
Similarly a person sitting in the compartment of a moving train is in the state of rest, with respect to the
surrounding of compartment.
(ii) When a body changes its position with respect to its surroundings, it is said to be in motion. For example,
a car changing its position with respect to trees, houses, etc., is in the state of motion.
Similarly a person sitting in the compartment of a moving train is in the state of motion, if he compares
himself with the surroundings outside the compartment.
2.
Define (i) scalar quantities (ii) vector quantities.
Give two differences between scalar and vector quantities.
Ans. (i) Scalar Quantities : The physical quantities which are expressed in magnitude only are called scalar
quantities. They do not have any direction.
(ii) Vector Quantities : The physical quantities which are expressed in magnitude as well as direction are
called vector quantities.
Differences between Scalar and Vector Quantities
Scalar Quantities Vector Quantities
1. They are expressed in magnitude only. 1. They are expressed in magnitude as well as drection.
2. They can be added by simple arithmetic means. 2. They cannot be added by simple arithmetic means.
3.
Pick out the scalar and vector quantities from the following list :

(i) mass (ii) density (iii) displacement (iv) distance (v) momentum (vi) acceleration (vii) temperature
(viii) time.
Ans. Scalar Quantities Vector Quantities
(i) Mass (iii) Displacement
(ii) Density (v) Momentum
(iv) Distance (vi) Acceleration
(vii) Temperature
(viii) Time

4. Define (i) Speed (ii) Velocity. Give two differences between speed and velocity.
Ans. (i) Speed : Distance covered by a moving body per unit time is called speed.
Thus, if S is the distance covered by a body in time t, then :
Distance S
Speed = =
Time t
22
(ii) Velocity : Rate of change of displacement is called velocity.
Displacement
Velocity =
Time
Differences between speed and velocity
Speed Velocity
1. The distance travelled by a body in one 1. The distance travelled by a body in one second
second is called speed. in a particular direction is called velocity.
2. It is a scalar quantity. 2. It is a vector quantity.

5. Define (i) Distance (ii) Displacement. Give two differences between displacement and distance.
Ans. (i) Distance : The length of the path travelled by a body in certain interval of time is called distance.
(ii) Displacement : The shortest distance between the initial position and the final position of a body is called
displacement.
Differences between displacement and distance
Displacement Distance
(i) It is a vector quantity. It is a scalar quantity.
(ii) It can have both positive and negative values. It has only positive values.
6. By giving one example of each, define
(i) Variable velocity (ii) Average velocity (iii) Uniform velocity
Ans. (i) When a body covers unequal distances in equal intervals of time, in a specified direction however small
intervals may be, or changes its direction of motion; the body is said to be moving with variable velocity.
For example, a rotating fan at constant speed has variable velocity because of continuous change in direction.
(ii) The ratio of the total distance travelled in a specified direction to the total time taken by the body to travel
that distance is called average velocity.
Thus, average velocity
Total distance travelled in specified direction .
=
Total timee taken
(iii) When a body covers equal distances in equal intervals of time (however small may be the time interval)
in a specified direction, the body is said to be moving with uniform velocity.
7. What do you understand by the term acceleration? When is the acceleration (i) positive (ii) negative?
Ans. The rate of change of velocity of a body is called acceleration.
Change in velocity

Thus, acceleration =
Time
(i) Positive Acceleration : If the velocity of a body is increasing with respect to time, the acceleration is said
to be positive.
(ii) Negative Acceleration : If the velocity of a body is decreasing with respect to time, the acceleration is
said to be negative. The negative acceleration is sometimes called deceleration or retardation.
8. Define the term acceleration due to gravity. State its value in C.G.S. as well as in S.I. system. When is acceleration
due to gravity (i) positive (ii) negative?
Ans. The acceleration of a freely falling body, under the action of gravity of earth, is called acceleration due to gravity.
The average value of acceleration due to gravity is in C.G.S. system is 980 cm/s2 and in S.I. system 9.8 m/s2.
(i) Acceleration due to gravity is always acting downward. The value of g is positive if a body falls towards
the earth.
(ii) The value of acceleration due to gravity is negative, if the body rises vertically upwards.

23
9. What is the relation between distance and time when (i) body is moving uniform velocity (ii) body is moving
with variable velocity?
Ans.
(i) When a body is moving with uniform velocity it covers equal distances in equal interval of time (however
small may be the time interval) in a specified direction.
(ii) When a body is moving with variable velocity it covers unequal distances in equal intervals of time in a
specified direction or changes its direction of motion.
10. (a) Distinguish between scalar and vector quantities.
(b) State whether following are scalar or vector quantities
(i) speed (ii) force (iii) acceleration (iv) energy.
Ans. (a) Differences between Scalar and Vector Quantities
Scalar Quantities Vector Quantities
1. They are expressed in magnitude only. They are expressed in magnitude as well as
direction.
2. T
 hey can be added by simple arithmetic They cannot be added by simple arithmetic means.
means.
3. They cannot be easily plotted on graph paper. They can be easily plotted on graph paper.

(b) Scalar quantities Vector quantities


(i) Speed (ii) Force
(iv) Energy (iii) Acceleration

11. Copy the following table and fill in the blank spaces.

Quantity S.I. Unit Scalar or Vector

Displacement ............................. ................................

......................... kg m–3 ................................

Ans. Quantity S.I. Unit Scalar or Vector


Displacement metre (m) Vector
Density kg m–3 Scalar

12. Draw a diagram to show the motion of a body whose speed remains constant, but velocity changes continuously.
Ans.
v
B
v

C
A

v
D
v

24
Practice Problems 1.
1
1.
A car covers 90 km in 1 hours towards east. Calculate
2
(i) displacement of car
(ii) its velocity in (a) kmh–1 (b) ms–1
Solution :
(i) Displacement of car = 90 km towards east
Displacement
(ii) (a) Velocity =
Time
90 km 90 ¥ 2
= = kmh -1
3 3
h
2
= 60 kmh–1
5 50
(b) Velocity in ms–1 = 60 × ms–1 = ms-1
18 3
= 16.67 ms –1

2.
A race horse runs straight towards north and covers 540 m in one minute. Calculate
(i) displacement of horse.
(ii) its velocity in (a) ms–1 (b) kmh–1
Solution :
(i) Displacement of horse = 540 m towards north.
Displacement
(ii) (a) Velocity =
Time
540 m
= [ 1 minute = 60 s]
60 s
= 9 ms–1

18
(b) Velocity in kmh–1 = 9 × kmh -1 = 32.4 kmh–1.
5
Practice Problems 2.
1.
The change in velocity of a motor bike is 54 kmh–1 in one minute. Calculate its acceleration in (a) ms–2 (b) kmh–2.
Solution :
(a) Change in velocity of the motor bike = 54 kmh–1 in one minute.
5
Change in velocity in ms–1 = 54 × ms–1 = 15 ms–1 in one minute.
18
Change in velocity
Acceleration =
Time
15 ms -1
=
60 s
= 0.25 ms–2
(b) Change in velocity = 54 kmh–1 in 1 minute
\ Acceleration in kmh–2 = Change in velocity
Time

25
54 kmh -1 Ê 1 ˆ
= ÁË  1 minute = h˜
1 60 ¯
h
60

= 3240 kmh -2
2.
A speeding car changes its velocity from 108 kmh–1 to 36 kmh–1 in 4 s. Calculate its deceleration in (i) ms–2
(ii) kmh–2.
Solution :
(i) Change in velocity = (108 – 36) kmh–1 = 72 kmh–1
5
Change in velocity = 72 kmh–1 = 72 × ms–1 = 20 ms–1
18
Change in velocity (ms -1 )
Deceleration in ms–2 =
Time (s)
20 ms -1
= = 5 ms -2
4s
Change in velocity (kmh -1 )
(ii) Deceleration in kmh–2 =
Time (h)

72 kmh -1
=
4
h
60 ¥ 60
72 ¥ 60 ¥ 60
= kmh -2
4
= 64800 kmh -2

UNIT II

EXERCISE 2
(A) Objective Questions
Multiple Choice Questions.
Select the correct option.
1. A graph is a straight line parallel to the time axis in a distance-time graph. From the graph, it implies :
(a) body is stationary (b) body is moving with a uniform speed
(c) body is moving with a variable speed (d) none of these
2.
The slope of a displacement-time graph represents :
(a) uniform speed (b) non-uniform speed (c) uniform velocity (d) uniform a acceleration
3.
A body dropped from the top of a tower reaches the ground in 4 s. Height of the tower is
(a) 39.2 m (b) 44.1 m (c) 58.8 m (d) 78.4 m
4.
The speed of a car reduces from 15 m/s to 5 m/s over a displacement of 10 m.

The uniform acceleration of the car is :
(a) –10 ms–2 (b) + 10 ms–2 (c) 2 ms–2 (d) 0.5 ms–2

26
5.
A body projected vertically up with a velocity 10 m/s reaches a height of 20 m. If it is projected with a velocity
of 20 m/s, then the maximum height reached by the body is :
(a) 20 m (b) 10 m (c) 80 m (d) 40 m
6.
What does the area of an acceleration-time graph represent?
(a) Uniform velocity (b) Displacement (c) Distance (d) Variable velocity
7.
A driver applies brakes when he sees a child on the railway track, the speed of the train reduces from 54 km/h
to 18 km/h in 5 s. What is the distance travelled by the train during this interval of time?
(a) 52 m (b) 50 m
(c) 25 m (d) 80 m

8.
In velocity time graph, the acceleration is :
(a) – 4 m /s2 (b) 4 m/s2
(c) 10 m/s2 (d) zero

9.
The distance covered in adjoining velocity-time graph is :
(a) 25 m (b) 40 m
(c) 50 m (d) 45 m

10. At the maximum height, a body thrown vertically upwards has :


(a) velocity not zero but acceleration zero.
(b) acceleration not zero but velocity zero.
(c) both acceleration and velocity are zero.
(d) both acceleration and velocity are not zero.
A
ns. 1. (a) 2. (c) 3. (d) 4. (a) 5. (c) 6. (d)
7. (b) 8. (a) 9. (c) 10. (b)

(B) Subjective Questions

1.
Draw displacement-time graphs for the following situations :
(i) When a body is stationary.
(ii) When a body is moving with uniform velocity.
(iii) When a body is moving with variable velocity.
Ans. (i) Displacement-time graph when the body is stationary.
PQ is the required graph.
Displacement in (m)

Q
8 P
6
4
2

0 1 2 3 4 5 6
Time in (seconds)

27
(ii) Displacement-time graph when the body is moving with uniform velocity.

(iii) Displacement-time graph when the body is moving with variable velocity.

2.
Draw velocity – time graphs for the following situations :
(i) When a body is moving with uniform velocity.
(ii) When a body is moving with variable velocity, but uniform acceleration.
(iii) When a body is moving with variable velocity, but uniform retardation.
(iv) When a body is moving with variable velocity and variable acceleration.
Ans. (i) Velocity-time graph parallel to time-axis represents body is moving with a uniform velocity.

(ii) When the velocity-time graph is a straight line but not parallel to time axis represents body moving with
variable velocity, but uniform acceleration.

28
(iii) Velocity-time graph is a straight line not parallel to the time
axis and the slope of the line is negative, then such type of
velocity-time graph represents a body moving with variable
velocity but uniform retardation.

(iv) A curve velocity-time graph represents a body is moving


with variable velocity and variable acceleration.

3.
How can you find the following?
(i) Velocity from a displacement-time graph.
(ii) Acceleration from velocity-time graph.
(iii) Displacement from velocity-time graph.
(iv) Velocity from acceleration-time graph.
Ans. (i) In the displacement-time graph, the displacement is plotted on the Y-axis and time on X-axis. Since, velocity
Displacement
is equal to , therefore, the slope of displacement-time graph gives the velocity.
Time
(ii) In the velocity-time graph, the velocity is plotted on Y-axis and time on X-axis. Since, acceleration is equal
Velocity
to , therefore the slope of velocity-time graph gives the acceleration.
Time
(iii) The area of graph under velocity-time graph gives displacement of the body.

(iv) The area of graph under acceleration-time graph gives velocity of the body.

4.
What do you understand by the term acceleration due to gravity? What is its value in C.G.S. and S.I. systems?
Ans. When a body falls freely, its velocity constantly increases with respect to time, hence, the freely falling body is
acted upon by uniform acceleration. The acceleration of a freely falling body, under the action of gravity of earth,
is known as acceleration due to gravity.

In C.G.S. system the value of acceleration due to gravity is 980 cm/s2.

In S.I. system the value of acceleration due to gravity is 9.8 m/s2.
5.
Can you suggest about the kind of motion of a body from the following distance-time graphs?

29
Ans. In the Fig (a) the distance-time graph is parallel to time axis, it means the body is not changing its position with
respect to time. In other words, the body is stationary.
In the Fig (b) the distance-time graph is a straight line inclined to time axis and starts from origin shows that the
body is moving with uniform speed.
In the Fig (c) the portion OA is a straight line and it is slopping upwards from O to A, therefore the graph of
the line OA represents uniform speed.
AB is a straight line graph between distance-time, which is parallel to the time axis so it represents that the body
is stationary.
Thus, graph (c) shows that body moves with uniform velocity for some time then stops.
6. Can you suggest real life examples about the motion of a body from the following velocity-time graphs?

Ans. (a) A car moving with uniform velocity on a straight road.


(b) A freely falling stone under the action of gravity.
(c) A car starts with uniform acceleration, picks up its uniform velocity, then moves with a uniform velocity
and finally is brought to rest by uniform retarding force.
(d) A ball thrown vertically upward then reaches its highest point.
(e) A ball thrown vertically upward then returns back to the ground level.
7. Diagram shows a velocity-time graph for a car starting from rest. The
graph has three sections AB, BC and CD.

(i) From a study of this graph, state how the distance travelled in any
section is determined.

30
(ii) Compare the distance travelled in section BC with distance travelled in section AB.
(iii) In which section, car has a zero acceleration?
(iv) Is the magnitude of acceleration higher or lower than that of retardation? Give reason.
Ans. (i) Area under the curve ABCD represents the distance covered.
(ii) Distance travelled in the section BC = V0 × t
1 1
Distance travelled in the section AB = × V0 × t = V0t
2 2
Distance travelled in the section BC V0 t
Thus, = = 2
Distance travellled in the section AB 1
V0 t
2
So, required ratio = 2 : 1.
(iii) In section BC the car has zero acceleration.
(iv) The magnitude of acceleration is lower than that of retardation. This is because the magnitude of slope of
AB is less than that of DC.
8.
Write down the type of motion of a body along the A – O – B in each of the following distance-time graphs.

Ans. (a) The body is at rest.


(b) The body is moving with constant speed.
(c) The body is moving on a perfectly smooth floor then hits a obstruction and finally comes back along the
same path.
Practice Problems 1.
1. From the displacement-time graph
shown alongside calculate :
(i) Average velocity in first three seconds.
(ii) Displacement from initial position at the end of 13 s.
(iii) Time after which the body is at the initial position.
(iv) Average velocity after 8 s.
Solution :
8m
(i) Average velocity between 0 – 3 s = = 2.67 ms -1
3s
(ii) Displacement from initial position at the end of 13 s is –8 m.
(iii) After 8 s and 17 s the body is at its initial position.
(iv) Average velocity after 8 s is zero.
2.
From the displacement-time graph shown alongside calculate :
(i) Velocity between 0 – 2 s.
(ii) Velocity between 8 s – 12 s.
(iii) Average velocity between 5 s – 12 s.

31
Solution :
(i) Velocity between 0 – 2 s = Displacement
Time interval
10 - 25 m -15
= = = -7.5 ms -1
2-0 2
25 - 20 m
(ii) Velocity between 8 s – 12 s =
4s
5
= ms -1 = 1.25 ms -1
4
(25 - 10) m =
15 m
= 2.1 ms -1
(iii) Average velocity between 5 s – 12 s =
(12 - 5) s 7s

Practice Problems 2.
1.
A train starting from rest, picks up a speed of 20 ms–1 in 200 s. It continues to move at the same rate for next
500 s, and is then brought to rest in another 100 s.
(i) Plot a speed-time graph.
(ii) From graph calculate (a) uniform rate of acceleration (b) uniform rate of retardation (c) total distance
covered before stopping (d) average speed.
Solution :
(i)

B C
20
Speed (ms )
–1

15

10

5
F E D
O
A
100

300

500

700
200

600

800
400

Time (s)

Change in speed
(ii) (a) Uniform rate of acceleration = (slope of graph line AB)
Time
20
= = 0.1 ms -2
200
Change in speed
(b) Uniform rate of retardation = (slope of graph line CD)
Time

20 m/s
= = 0.2 ms -2
100 s

(c) Total distance covered = Area of triangle ABF + Area of rectangle BCEF + Area of DCDE
1 1
= × AF × BF + BC × BF + × EC × ED
2 2

32
1 1
= × 200 × 20 + 500 × 20 + × 20 × 100
2 2
= 2000 m + 10000 m + 1000 m
= 13000 m = 13 km
Total distance travelled
(d) Average speed =
Total time taken
13000 m 65 -1
= = ms = 16.25 ms-1
800 s 4
2. A ball is thrown up vertically, and returns back to thrower in 6 s. Assuming there is no air friction, plot a graph
between speed and time. From the graph calculate (i) deceleration (ii) acceleration (iii) total distance covered by
ball (iv) average speed.
Solution :

(i) When the ball is thrown up vertically the acceleration due to gravity acting downwards. So, the value of
deceleration is = – 10 ms–2.
(ii) When the ball returns back to the thrower. The acceleration due to gravity acting downwards, and the ball
is moving downward. So the value of acceleration is equal to 10 ms–2.
(iii) Let u = initial velocity of the ball
v = 0 (final velocity of the ball)
From relation, v = u + at
As the total time of flight is 6, so the ball will reach the maximum height in 3 s,
⇒ 0 = u – 10 × 3
\ u = 30 ms–1
Hence, distance travelled by the ball
1 1
Area of DABC = × OA × AC = × OA × OE ( AC = OE)
2 2
1
= × 30 m/s × 6 s
2
= 90 m
Total distance travelled 90 m
(iv) Average speed = = = 15 ms -1
Total time taken 6 s
3. A racing car is moving with a velocity of 50 m/s. On applying brakes, it is uniformly retarded and comes to rest
in 20 seconds. Calculate its acceleration.

33
Solution :
Initial velocity u = 50 m/s
Time taken to come to rest = 20 seconds
From relation v = u + at
Þ 0 = 50 + a × 20 s
-50
Þ a = ms-2 = - 2.5 ms-2
20
4. A body falls freely downward from a certain height. Show graphically the relation between the distance fallen
and square of time. How will you determine ‘g’ from the graph?

Ans. For a freely falling body, the distance is directly proportional to the square of time (s ∝ t2). If a graph is plotted
by taking the distance (s) on Y-axis and the square of time (t2) on X-axis is a straight line as shown below.

(Time)2 t2(s2) 1 4 9 16 25 30
Distance (m) 5 20 45 80 125 150


The slope of the graph gives the value of half of the acceleration due to gravity. Thus the value of the acceleration
due to gravity can be obtained by doubling the slope of s–t2 graph for freely falling body.

5. A body at rest is thrown downward from the top of tower. Draw a distance-time graph of its free fall under gravity
during first 3 seconds. Show your table of values starting t = 0 with an interval of 1 second. (g = 10 ms–2).

34
Solution :

Time in (s) Distance in (m)

0 0

1 5

2 20

3 45

35
Practice Problem 3.
1. From the diagram given alongside, calculate
(i) acceleration (ii) deceleration (iii) distance covered by body.
Solution :
(i) Acceleration = Slope of graph line AB
BD 15 ms -1
= = = 1.5 ms -2
AD 10 s
(ii) Deceleration = Slope of graph line BC
BD 15 ms -1
= = = 2.5 ms -2
CD 6s
(iii) Distance covered by the body
1
= Area of DABC = × AC × BD
2
1
= × 16 s × 15 ms–1
2
= 120 m
2.
From the velocity-time graph given alongside, calculate :
(i) Acceleration in the region AB.
(ii) Deceleration in region BC.
(iii) Distance covered in the region ABCE.
(iv) Average velocity in region CED.
Solution :
(i) Acceleration in the region AB
= Slope of the line AB
BF 14
= = = 1.16 ms -2
AF 12
(ii) Deceleration in region BC
= Slope of graph line BC
BG 8
= = = 2 ms -2
GC 4
(iii) Distance covered in the region ABCE
= Area of DABF + Area of trapezium BCEF
1 1
= × AF × BF + × EF(BF + CE)
2 2
1 1
= × 12 × 14 + × 4 × (14 + 6)
2 2
= 84 + 40 = 124 m

(iv) Average velocity in the region CED


Total distance travelled
=
Time taken

36
Distance travelled in the region CED
1
= × CE × DE
2
1
= × 6 × 12 = 36 m.
2
36 m
\ Average velocity = = 3 ms–1
12 s
Practice Problem 4.
1. Diagram alongside shows velocity - time graphs
of car P and Q, starting from same place and in
same direction.
Calculate :
(i) Acceleration of car P.
(ii) Acceleration of car Q between 2 s – 5 s.
(iii) At what time intervals both cars have same
velocity?
(iv) Which car is ahead after 10 s and by how
much?

Solution :
35 ms -1
(i) Acceleration of car P = = 3.5 ms–2.
10 s
25 ms -1
(ii) Acceleration of car Q between 2 s – 5 s = = 8.33 ms–2.
3s
(iii) From the graph, at 3 s and 7 s both the cars have same velocity.
1
(iv) Distance travelled by the car P = × 35 × 10 m = 175 m
2
1
Distance travelled by the car Q = × 25 × 3 + 25 × 5 = 37.5 + 125 = 162.5 m.
2
\ After 10 s car P ahead by = (175 – 162.5) m = 12.5 m

UNIT III
Practice Problems 1.
1. A motor bike, initially at rest, picks up a velocity of 72 kmh–1 over a distance of 40 m. Calculate (i) acceleration
(ii) time in which it picks up above velocity.
Solution :
Initial velocity of motorbike (u) = 0.
5

Final velocity of the motorbike = 72 kmh–1 = 72 × ms–1 = 20 ms–1.
18

Distance covered by motorbike (S) = 40 m
(i) Acceleration (a) = ?
Applying v2 – u2 = 2aS
Þ (20)2 – (0)2 = 2 × a × 40
Þ 80a = 400
Þ a = 5 ms–2
37
(ii) Time (t) = ?
Applying, v = u + at
Þ 20 = 0 + 5 × t
t = 4 s.

2.
A cyclist driving at 5 ms–1, picks a velocity of 10 ms–1, over a distance of 50 m. Calculate (i) acceleration
(ii) time in which the cyclist picks up above velocity.
Solution :
Initial velocity of cyclist (u) = 5 ms–1.

Final velocity (v) = 10 ms–1

Distance covered (S) = 50 m
(i) Acceleration = ?
Applying v2 – u2 = 2 S
Þ (10)2 – (5)2 = 2a × 50

Þ 100 a = 100 – 25 = 75

\ a = 0.75 ms–2

(ii) Time (t) = ?


Applying, v = u + at
10 = 5 + 0.75 × t
5 20
t = = = 6.67 s.
0.75 3
Practice Problems 2.
1.
An aeroplane lands at 216 kmh–1 and stops after covering a runway of 2 km. Calculate the acceleration and the
time, in which it comes to rest.
Solution :
5

Initial velocity (u) = 216 kmh–1 = 216 × ms–1 = 60 ms–1
18

Final velocity (v) = 0

Distance (S) = 2 km = 2000 m
Acceleration = ?

Applying v2 – u2 = 2 S
Þ (0)2 – (60)2 = 2 × a × 2000

-3600 9
Þ ms–2 = - a =
4000 10
\ = – 0.9 ms –2


Applying v = u + at

0 = 60 – 0.9 × t
60 200
t = = = 66.67 s.
0.9 3
2.
A truck running at 90 kmh–1, is brought to rest over a distance of 25 m. Calculate the retardation and time for
which brakes are applied.

38
Solution :
5

Initial velocity of truck (u) = 90 × ms–1 = 25 ms–1
18

Final velocity of the truck = 0

Distance (S) = 25 m

Applying v2 – u2 = 2aS
02 – (25)2 = 2 × a × 25
-25 ¥ 25
Þ      a = ms -2
2 ¥ 25
\     a = –12.5 ms–2

So, retardation = –a = 12.5 ms–2

Time (t) = ?

Applying v = u + at

Þ 0 = 25 + (–12.5)t
25
Þ
t = s
12.5
\
t = 2 s
Practice Problems 3
1. A racing car, initially at rest, picks up a velocity of 180 kmh–1 in 4.5 s. Calculate (i) acceleration (ii) distance
covered by car.
Solution :

Initial velocity of racing car (u) = 0
5

Final velocity of the racing car (v) = 180 kmh–1 = 180 × ms–1 = 50 ms–1
18

Time (t) = 4.5 s
(i) Acceleration ‘a’ = ?
Applying v = u + at
⇒ 50 = 0 + a × 4.5
50 100 −2
\ a = ms–2 = ms = 11.11 ms–2
4.5 9
(ii) Let distance covered by the car = S
Applying v2 – u2 = 2aS
50

Þ (50)2 – 02 = 2 × ¥S
4.5
50 ¥ 50 ¥ 45 225
Þ
S = = = 112.5 m
500 ¥ 2 2
2.
A motor bike running at 5 ms–1, picks up a velocity of 30 ms–1 in 5 s. Calculate (i) acceleration (ii) distance
covered during acceleration.
Solution :

Initial velocity of the motor bike (u) = 5 ms–1

Final velocity of the motor bike (v) = 30 ms–1

39

Time (t) = 5 s
(i) Acceleration ‘a’ = ?
Applying v = u + at
Þ 30 = 5 + a × 5
Þ 5a = 25
\ a = 5 ms–2
(ii) Distance covered during 5 s = S

Applying v2 – u2 = 2aS

Þ (30)2 – (5)2 = 2 × 5 × S

Þ 10 S = 875

S = 87.5 m.
Practice Problems 4
1.
A motor bike running at 90 kmh–1 is slowed down to 18 kmh–1 in 2.5 s. Calculate (i) acceleration (ii) distance
covered during slow down.
Solution :
5

Initial velocity of (u) = 90 kmh–1 = 90 × ms–1 = 25 ms–1
18

Final velocity (v) = 18 kmh–1 = 5 ms–1

Time (t) = 2.5 s
(i) Acceleration = ?
Applying v = u + at
Þ 5 = 25 + a × 2.5
Þ 2.5a = –20
\ a = –8 ms–2
(ii) Let distance covered during slow down = S
1
Applying, S = ut + at2
2
1

= 25 × 2.5 + × (–8) × (2.5)2
2

= 62.5 – 25
\
S = 37.5 m

2.
A cyclist driving at 36 kmh–1 stops his motion in 2 s, by the application of brakes. Calculate (i) retardation
(ii) distance covered during the application of brakes.
Solution :

Initial velocity (u) = 36 kmh–1 = 10 ms–1

Final velocity (v) = 0

Time (t) = 2 s
(i) Retardation = ?
Applying v = u + at
Þ 0 = 10 + a × 2
\ a = –5 ms–2
Retardation = –(a) = 5 ms–2.

40
(ii) Distance covered S
1 2
Applying, S = ut + at
2
1

= 10 × 2 – × 5 × (2)2
2
\
S = 20 – 10 = 10 m

Practice Problems 5
1.
A motor bike running at 90 kmh–1, is slowed down to 54 kmh–1 by the application of brakes, over a distance of
40 m. If the brakes are applied with the same force, calculate (i) total time in which bike comes to rest (ii) total
distance travelled by bike.
Solution :
5
(i) Initial velocity of motor bike u = 90 kmh–1 = 90 × ms–1 = 25 ms–1
18
5
Velocity after slowdown of the bike, vs = 54 × ms–1 = 15 ms–1
18
Distance covered S = 40 m
Applying the relation, vs2 – u2 = 2aS
Þ (15)2 – (25)2 = 2 × a × 40
Þ 80a = –400
a = –5 ms–2
Final velocity v = 0
Applying relation v = u + at
Þ 0 = 25 – 5 × t
\ t = 5 s
(ii) Total distance travelled by the bike (S) = ?
1
Applying, S = ut + at2
2
1
= 25 × 5 – × 5 × (5)2
2
= 125 – 62.5 = 62.5 m

2.
A motor car slows down from 72 kmh–1 to 36 kmh–1 over at distance of 25 m. If the brakes are applied with the
same force calculate (i) total time in which car comes to rest (ii) distance travelled by it.
Solution :
5
(i) Initial velocity of motor car (u) = 72 km/h = 72 × ms–1 = 20 ms–1
18
5
Velocity after slowdown of the motor car, vs = 36 kmh–1 = 36 × ms–1 = 10 ms–1
18
Distance covered = 25 m
Final velocity of the motor car v = 0;
Applying vs2 – u2 = 2aS
Þ (10)2 – (20)2 = 2a × 25
Þ 50a = –300
\ a = –6 ms–2
Applying v = u + at
Þ 0 = 20 – 6t
20
Þ t = s = 3.33 s
6
41
(ii) Let S = total distance travelled before coming to rest
Applying, v2 = u2 + 2aS
Þ 02 = (20)2 + 2(–6) × S
400 100
Þ S = = = 33.33 m
12 3
Practice Problems 6
1.
A packet is dropped from a stationary helicopter, hovering at a height ‘h’ from ground level, reaches ground in
12 s. Calculate (i) value of h (ii) final velocity of packet on reaching ground.

(Take g = 9.8 ms–2)
Solution :

Initial velocity of the packet, u = 0

Time (t) = 12 s

Acceleration due to gravity (g) = 9.8 ms–2

Final velocity of the packet = v
h = height of the helicopter

1 2
(i) Applying h = ut + gt
2
1
= 0 × 12 + + 9.8 × (12)2
2
Þ h = 705.6 m
(ii) Applying, v = u + gt
Þ v = 0 + 9.8 × 12
\ v = 117.6 ms–1
2.
A boy drops a stone from a cliff, reaches the ground in 8 seconds. Calculate (i) final velocity of stone (ii) height
of cliff. (Take g = 9.8 ms–2)
Solution :
Initial velocity of the stone u = 0;

Time (t) = 8 s

Final velocity of the stone (v) = ?
(i) Applying v = u + gt
= 0 × 9.8 × 8
\ v = 78.4 ms–1
(ii) Let height of cliff = h
1
Applying h = ut + gt2
2
1
=0×t+ × 9.8 × (8)2
2
= 313.6 m
\ Height of cliff = 313.6 m
Practice Problems 7
1.
A stone thrown vertically upwards, takes 3 s to attain maximum height. Calculate (i) initial velocity of the stone
(ii) maximum height attained by the stone. (Take g = 9.8 ms–2)
Solution :
Initial velocity of the stone u = ?

Final velocity of the stone (v) = 0
42

Acceleration due to gravity = – 9.8 ms–2
Time t = 3 s
(i) Applying v = u + gt
0 = u – 9.8 × 3
u = 29.4 ms–1
(ii) Let h = maximum height attained by the stone.
1
Applying h = ut + gt2
2
1
= 29.4 × 3 – × 9.8 × (3)2
2
= 88.2 – 44.1
\ h = 44.1 m
2.
A stone thrown vertically upwards, takes 4 s to return to thrower. Calculate (i) initial velocity of the stone
(ii) maximum height attained by stone. (Take g = 10 ms–2)

[Hint : Time for upward journey = 2 s.]
Solution :

Initial velocity of the stone u = ?

Final velocity of the stone (v) = 0;

Acceleration due to gravity = – 10 ms–2

Time taken to reach maximum height (t) = 2 s
(i) Applying v = u + gt
Þ 0 = u – 10 × 2
\ u = 20 ms–1
(ii) Let h = maximum height attained by the stone.
1
h = ut + gt2
2
1
= 20 × 2 – × 10 × (2)2
2
= 40 – 20 = 20 m
Practice Problems 8
1.
A spaceship is moving in space with a velocity of 50 kms–1. Its engine fires for 10 s, such that its velocity
1
increases to 60 kms–1. Calculate the total distance travelled by spaceship in minute, from the time of firing
its engine. 2

Solution :

Initial velocity of the spaceship (u) = 50 kms–1

Final velocity of the spaceship (v) = 60 kms–1

Time (t) = 10 s

Applying v = u + at

Þ 60 = 50 + a × 10

Þ a = 1 kms–2
1

Applying S = ut +
× 1 × at2
2
1

= 50 × 10 + × 1 × (10)2
2

= 550 km
43

When the spaceship is accelerating the distance travelled by the spaceship = 550 km

Next, the space is moving with uniform velocity for next 20 s

Hence, distance covered = 60 kms–1 × 20 s

= 1200 km

\ Total distance travelled by spaceship in 1/2 minute

= 550 + 1200 = 1750 km
2.
A spaceship is moving in space with a velocity of 60 kms–1. It fires its retro engines for 20 second and velocity
is reduced to 55 kms–1. Calculate the distance travelled by the spaceship in 40 s, from the time of firing of the
retro-rockets.
Solution :

Initial velocity of the spaceship (u) = 60 kms–1

Final velocity of the spaceship (v) = 55 kms–1

Time (t) = 20 s for which retro engines fires.

Applying v = u + at

Þ 55 = 60 + a × 20
-5

\ a == – 0.25 kms–2
20
1

Applying S = ut + at2
2
1

Þ S = 60 × 20 – × 0.25 × (20)2
2

Þ S = 1200 – 50 = 1150 km

When the spaceship is decelerating the distance travelled = 1150 km

When the spaceship is moving with uniform velocity of 55 kms–1 for next 20 s

Distance covered = Uniform velocity × Time

= 55 × 20 = 1100 km

\ Total distance travelled by the spaceship in 40 s

= (1150 + 1100) km

= 2250 km.

EEEEE

44
3 Laws of Motion

UNIT I
Practice Problems 1.
1.
Calculate the velocity of a body of mass 0.5 kg, when it has a linear momentum of 5 Ns.
Solution :
Mass (m) = 0.5 kg
Velocity (v) = ?
Linear momentum (P) = 5 Ns.
By applying p = m × v
p 5
\ v = m = 0.5

Þ v = 10 ms–1

2.
An electron of mass 9 × 10–31 kg is moving with a linear velocity of 6 × 107 ms–1. Calculate the linear momentum
of electron.
Solution :
Mass of electron (m) = 9 × 10–31 kg
Velocity of electron (v) = 6 × 107 ms–1.
Momentum = p
By applying p = mv
Þ p = 9 × 10–31 × 6 × 107 Ns
= 54 × 10–24 Ns.

3.
A body of mass 200 g is moving with velocity of 5 ms–1. If the velocity of the body changes to 17 ms–1, calculate
the change in linear momentum of the body.
Solution :
Mass of the body (m) = 200 g = 0.2 kg
Initial velocity of the body, vi = 5 ms–1.
Final velocity of the body, vf = 17 ms–1
Change in momentum = mvf – mvi = m(vf – vi)
\ = 0.2 (17 – 5) Ns

= 2.4 Ns

4.
A motorcycle of mass 100 kg is running at 10 ms–1. If its engine develops an extra linear momentum of
2000 Ns, calculate the new velocity of motorcycle.
Solution :
Initial velocity of the motorcycle vi = 10 ms–1
Mass of the motorcycle m = 100 kg
Let vf ® New velocity of the motorcycle

45
Extra linear momentum = 2000 Ns
Þ 2000 Ns = mvf – mvi
= 100(vf – 10)
Þ vf – 10 = 20
\ vf = 30 ms–1

Practice Problems 1.
1.
A car initially at rest, picks up a velocity of 72 kmh–1 in 20 seconds. If the mass of the car is 1000 kg, find
(i) Force developed by its engine (ii) Distance covered by the car.
Solution :
(i) Initial velocity of the car, vi = 0
5
Final velocity of the car, vf = 72 kmh–1 = 72 × ms–1 = 20 ms–1
18
Time = 20 s; Mass of the car, m = 1000 kg
(
m v f - vi ) 1000 ¥ (20 - 0)
Force developed by its engine = = N
t 20
= 1000 N
(ii) Distance covered by the car = S
Applying v = u + at
Þ 20 = 0 + a × 20
a = 1 ms–2
Applying v2 = u2 + 2aS
Þ (20)2 = (0)2 + 2 × 1 × S
400
Þ S = = 200 m
2
\ Distance covered by the car = 200 m.

2.
A golfer hits a ball at rest, such that the contact between the ball and golf stick is for 0.1 s. If the golf ball covers
a linear distance of 400 m in 2 s, find the magnitude of force applied. Mass of golf ball is 50 g.
Solution :

Mass of the golf ball = 50 g = 0.050 kg

Distance covered by the ball = 400 m

Time = 2 s
400 m

Uniform velocity of the ball = = 200 ms–1
2s

\ Final velocity of the ball after force acting on it = 200 ms–1

Initial velocity of the ball = 0

Time for which the force acts = 0.1 s.

Applying v = u + at

Þ 200 ms–1 = 0 + a × 0.1 s
\
a = 2000 ms–2


Force acting on the ball = F = ma = 0.050 × 2000 N = 100 N.

46
Practice Problems 2.
1.
A car of mass 800 kg, moving at 54 kmh–1 is brought to rest over a distance of 15 m. Find the retarding force
developed by the brakes of the car.
Solution :

Mass of the car = 800 kg
5

Initial velocity = 54 kmh–1 = 54 × ms–1 = 15 ms–1
18

Final velocity = 0

S = 15 m

Applying, v2 – u2 = 2aS

Þ 02 – 152 = 2 × a × 15
15

a = – ms–2
2
15

Retardation = ms–2
2

Retardation force = Mass × Retardation

15

= 800 × N = 6000 N.
2
2.
A cricket player holds a cricket ball of mass 100 g by moving his hands backward by 0.75 m. If the initial
velocity of the ball is 108 kmh–1, find the retarding force applied by the player.
Solution :
5

Initial velocity of the ball = 108 kmh–1 = 108 × ms–1 = 30 ms–1
18

Final velocity of the ball = 0

Distance, S = 0.75 m

Applying v2 – u2 = 2aS

Þ 02 – (30)2 = 2 × a × 0.75
900
Þ
= - 600 ms -2 a = -
1.5
\
Retardation = 600 ms–2
\
Retarding force = Mass × Retardation
100
= ¥ 600 = 60 N

1000

Practice Problems 3.
1.
A force of 600 dynes acts on a glass ball of mass 200 g for 12 s. If initially the ball is at rest, find (i) Final
velocity (ii) Distance covered.
Solution :
(i) Force (F) = 600 dynes = 600 gcms–2; mass (m) = 200 g
F 600
Now, a = = cms -2 = 3 cms–2
m 200
Applying v = u + at

47
= 0 + 3 × 12 = 36 cms–1
\ Final velocity of the ball = 36 cms–1
(ii) Let S = distance covered
Applying, v2 = u2 + 2aS
⇒ (36)2 = (0)2 + 2 × 3 × S
36 ¥ 36
S = = 216 cm
6
\ Distance covered by the ball = 216 cm.

2.
A bullet of mass 30 g, and moving with a velocity x hits a wooden target with a force of 187.5 N. If the bullet
penetrates 80 cm, find the value of x.
Solution :

Mass of bullet = 30 g = 0.03 kg

Velocity of the bullet = x

Force, F = 187.5 N

Distance travelled by the bullet in wood = S = 80 cm = 0.80 m
187.5
\ Retardation produced =
= ms–2 = 6250 ms–2
0.03
Applying v2 – u2 = 2aS

Þ 02 – x2 = 2(–6250) × 0.8

Þ
x2 = 10,000

x2 = 100 ms–1

Hence, bullet is moving with a velocity = 100 ms–1.
3.
A car of mass 1000 kg develops a force of 500 N over a distance of 49 m. If initially the car is at rest find
(i) Final velocity (ii) Time for which it accelerates.
Solution :
(i) Mass of car, m = 1000 kg
Force, F = 500 N

Distance, S = 49 m

F 500
Applying, a = = = 0.5 ms–2
m 1000
Initial velocity u = 0,
Final velocity = v
Applying v2 – u2 = 2aS
Þ v2 – 02 = 2 × 0.5 × 49
 \ v = 7 ms–1
\ Final velocity of the ball = 7 ms–1
(ii) Applying v = u + at
Þ 7 = 0 + 0.5t
\ t = 14 s

Hence, time for which it accelerates = 14 s.

48
EXERCISE 1
1.
(a) Define force.

(b) State four effects which a force can bring about. Give two examples in each case.
Ans.
(a) A force may be defined as an external cause which changes or tends to change the state of rest or uniform
motion of a body in a straight line.
(b) Four effects which a force can bring about are as follows :
1.
To set a stationary body into motion.
(a) A horse can make a carriage move by applying the force in the forward direction.
(b) A player can set a ball in motion by hitting it with some suitable material.
2.
To stop the moving bodies.

(a) A speeding car is stopped by the force of friction of brakes.

(b) A rolling football stops because of friction from the ground.
To change the speed or direction of a moving body.
3.

(a) A stone projected vertically upward changes its speed as well as direction because of the force of
gravity.

(b) A moving bicycle starts running faster, when more force is applied on its paddles.
4.
To bring about change in dimensions.
(a) Length of a rubber band increases, when a stretching force is applied.
(b) Wet clay can be moulded in any shape by applying a force with hands.
2.
What do you understand by the following terms? (a) Contact forces (b) Non-contact forces.
Ans. (a) Forces which act on a body directly or indirectly through a connector are called contact forces.
The force of pull or push, the force of friction between two surfaces, etc., are the examples of contact
forces.
(b) Force which acts on a body without any direct contact or a connector is called non-contact force.
The force of gravitation, the magnetic force, the electric force are the examples of non-contact forces.
3. (a) What do you understand by the term inertia?
(b) What are its kinds?
(c) Give two examples of each kinds, stated in (b).
Ans. (a) The tendency of a body to continue in its state of rest or uniform motion, in a straight line, even when
some external unbalanced force acts on it is called inertia.
(b) There are three kinds of inertia.
(i) Inertia of rest.
(ii) Inertia of motion and
(iii) Inertia of direction
Inertia of Rest : The tendency of a body to continue in its state of rest, even when some external unbalanced
force is applied on it, is called inertia of rest.
Inertia of Motion : The tendency of a body to continue in its state of motion, in a straight line, even when
some external unbalanced force acts on it, is called inertia of motion.
Inertia of Direction : The tendency of a body by which it is unable to change the direction of motion,
unless some external unbalanced force is applied on it is called inertia of direction.
(c) Inertia of Rest : The passengers sitting in a bus fall backwards, when the bus suddenly starts. The reason
is that when the bus moves, the passengers, due to the property of the inertia of rest, are left behind, and
hence, fall backwards.

49
When the branches of a tree, laden with fruits, are shaken, the fruits fall down. The reason is that when
branches move suddenly, the fruits, on account of the inertia of rest, are left behind and fall.
Inertia of motion : A running boy falls in the forward direction if he is tripped by a stone, etc., because
the stone stops his foot, whereas rest of the body continues moving forwards, and hence, the boy falls in
the forward direction.
Before taking a long jump, a boy runs a certain distance, because in doing so he picks up the inertia of
motion, which helps him in taking a longer leap.
Inertia of Direction : Passengers tend to fall sideways, when a speeding bus takes a sharp turn. It is
because, when the bus is moving along straight line in a particular direction suddenly takes a sharp turn,
the passengers on account of inertia of direction continue along their direction and hence fall sideways.
Standing passengers in the bus hold on to some support to prevent themselves from falling sideways, when
the speeding bus takes sharp turns.
4.
What do you understand by the term momentum?
Ans. The instantaneous force which a body possesses due to combined effect of mass and velocity is called momentum.
It is generally denoted by p. Mathematical expression for momentum is
p = m × v.
m ® mass of the body
v ® velocity of the body
5.
State two factors which determine the momentum of a body.
Ans. Factors which determine the momentum of a body are :
(i) mass ® More the mass of a moving body more is the momentum.
(ii) velocity ® More the velocity of a moving body more is the momentum.
6.
State units of momentum (i) CGS system (ii) SI system.
Ans. (i) In C.G.S system the unit of momentum is dyne second,
(ii) In SI system the unit of momentum is newton second (Ns)
7. Define Newton’s second law of motion.
Ans. According to Newton’s second law of motion “The rate of change of momentum is proportional to the applied
force.”
8.
Prove mathematically F = ma.
Ans. As per Newton’s second law of motion the rate of change of momentum is proportional to the applied force.


Consider a body of mass ‘m’, moving with an initial velocity ‘u’. Let a uniform force ‘F’ acts on the body for
‘t’ seconds, such that ‘v’ is its final velocity. Then :

Initial linear momentum of the body = Mass × Initial velocity

= mu

Final linear momentum of the body = Mass × Final velocity

= mv

\ Change in linear momentum in t seconds = mv – mu
m (v - u )

\ Change in linear momentum in 1 second =
t

50

But, change in linear momentum in one second = Rate of change of momentum
m (v - u )

\ Rate of change of linear momentum = ......... (1)
t

Also, we know, v = u + at
v-u

\ a = ..........(2)
t
v-u

Substituting the value of from (2) in (1), we have
t

Rate of change of linear momentum = ma ..........(3)

Also, according to Newton’s second law of motion :

Rate of change of linear momentum ∝ applied force (F) ...........(4)

\ Combining (3) and (4)

F ∝ ma

F = K ma ..........(5)

[Where K is the constant of proportionality]
Now, if there be a body with unit mass, moving with unit acceleration, such that force possessed by it is also

one unit, then :

1 = K × 1 × 1 or K = 1

Substituting the value of K in (5), we get
F = ma

Note : This is true only if a unit mass moving with unit acceleration produces a unit force.

Mathematically, force is the product of mass and acceleration.
9.
Define absolute unit of force in CGS as well as SI system.
Ans. Absolute Units of Force

A force which produces unit acceleration in a body of unit mass is called absolute unit of force.

In CGS system, absolute unit of force is called dyne.

Definition : When a body of mass 1 g, moves with an acceleration of 1 cms–2, then the force acting on the body
is called one dyne.

Thus, 1 dyne = 1 g × 1 cms–2 = 1 gcms–2

In SI system, the absolute unit of force is called newton (N).

Definition : When a body of mass 1 kg, moves with an acceleration of 1 ms–2, then the force acting on the body
is called one newton.

Thus, 1 newton (N) = 1 kg × 1 ms–2

= 1 kgms–2
10. Derive the relation between newton and dyne.
Ans. Relation between newton and dyne

1 N = 1 kg × 1 ms–2

= 1000 g × 100 cms–2

= 100000 gcms–2

\ 1 N = 105 dynes. [1 dyne = 1 gcms–2]
11. State Newton’s third law of motion and give two examples.
Ans. Newton’s third law of motion states that “to every action there is an equal and opposite reaction.

51
Example 1. While walking, we exert a force by our feet to push the ground backwards, the ground exerts a
force of the same magnitude on our feet forward, which makes us to move forward.
Example 2. When a boatman pushes the bank with a pole, the bank offers an equal and opposite reaction which
makes the boat move.
12. Explain the following :
(i) Why do we jerk wet clothes before spreading them on a line?
(ii) Why does dust fly off, when carpet is hit with a stick?
(iii) Why do fruits fall off the branches in the strong wind?
(iv) Why does a pillion rider fall forward, when the driver of a two-wheeler suddenly applies the brakes?
(v) Why does a boatman push the bank backward with a long bamboo pole, on launching his boat in water?
(vi) Why is it difficult to walk on marshy ground?
(vii) Why is it dangerous to jump out of a moving vehicle? How can the danger be minimised?
(viii) Why does a boat-man push water backward with the oars, while rowing a boat?
Ans. (i) When we jerk the clothes before spreading them on a line, clothes are in motion but water tends to stay at
rest due to inertia. Therefore, water falls out as gravity acts on it. A
s there will be less water in clothes,
the faster the rate of evaporation and hence, get dried off quickly.
(ii) When carpet is hit with a stick, the dust flies off. Because of sudden movement, the carpet moves, but dust
on account of inertia, is left behind.
(iii) When a strong wind blows, the branches of the trees shake due to which the fruits fall down. This is
because, when branches move suddenly, the fruits on account of inertia of rest are left behind and fall.
(iv) During the ride, pillion rider and driver are in a state of motion. But when the driver applies sudden, brakes,
the body of pillion rider continues moving forward due to inertia of motion. Therefore, the pillion rider
falls forward.
(v) When the boatman pushes the riverbank with a long bamboo pole, the bank reacts back according to
Newton’s third law and pushes the pole in the opposite direction. As the pole is in the hands of the boat
man, so the whole system i.e. boat moves in the opposite direction.
(vi) When we walk on the marshy ground, we push ground, but the marshy ground does not react back with
the same force. That is why it is difficult to walk on marshy ground.
(vii) The man jumping out from a moving vehicle possesses the inertia of motion. As the man lands on the
ground, feet come to rest immediately while the upper part of body continue to move due to inertia of
motion and hence, the person may fall forward and get hurt. It can be minimised by running in the same
direction for some time and gradually decreasing your speed.
(viii) During the rowing of a boat, the boat-man pushes the water with the oars backward. According to Newton’s
third law the water exerts an equal and opposite push on boat which makes the boat move forward.

UNIT II

EXERCISE 2
(A) Objective Type Questions
Multiple Choice Questions.
Select the correct option.
1. The mass of earth is 6 × 1024 kg and radius of earth is 6.4 × 106 m. The magnitude of force between the mass
of 1 kg and the earth is : (G = 6.7 × 10–11 Nm2 kg–2)
(a) 9.770 N (b) 9.810 N (c) 9.830 N (d) 9.790 N
2.
A man is walking from east to west on a rough surface. The force on the man is directed :
(a) from west to east (b) from east to west (c) along the north (d) along the west

52
3.
Inertia is the property of a body by virtue of which the body is :
(a) unable to change by itself the state of rest
(b) unable to change by itself the state of uniform motion
(c) unable to change by itself the direction of motion
(d) unable to change by itself the state of rest or uniform motion.
4.
The impulse of a body is equal to :
(a) rate of change of its momentum
(b) change in its momentum
(c) the product of force applied on it and the time of application of force.
(d) both (b) & (c).
5.
A force acts on a body of mass 3 kg such that its velocity changes from 4 ms–1 to 10 ms–1. The change in
momentum of the body is
(a) 42 kgms–1 (b) 2 kgms–1 (c) 18 kgms–1 (d) 14 kgms–1
6.
Action-reaction forces
(a) act on the same body (b) act on different bodies
(c) act along different lines (d) act in same direction
7.
Which of the following are vector quantities?
(a) Momentum (b) Velocity (c) Force (d) All of these
8.
A woman drawing water from a village well, falls backward, when the rope snaps. This is on account of
(a) Newton’s third law of motion (b) Newton’s law of gravitation
(c) Newton’s second law of motion (d) Newton’s first law of motion
9.
When you kick a stone, you get hurt. Due to which property this happens?
(a) Inertia of stone (b) Velocity of the kick
(c) Momentum of the kick (d) Reaction of the stone.
Ans. 1. (b) 2. (b) 3. (d) 4. (d) 5. (c)
6.
(b) 7. (d) 8. (d) 9. (d)

(B) Subjective Questions

1.
State Newton’s law of gravitation.
Ans. Newton’s law of gravitation states that :
‘Force of attraction between any two bodies is directly proportional to the product of their masses and inversely
proportional to the square of the distance between their centres of gravity”.

2.
How is acceleration due to gravity related to (i) mass of a planet (ii) distance of a body from the centre of earth?
Ans.
(i) Consider a body of mass ‘m’ on the surface of any planet such that it falls towards it with an acceleration
‘g’. Let ‘M’ be the mass of planet and ‘d’ the distance between the centre of body and the centre of planet.
If ‘F’ is the force acting on the body, then
F = mg ... (i)
Mm
F = G 2 ... (ii)
d
Comparing (i) and (ii),
Mm
mg = G 2
d
M
or g = G ... (iii)
d2
53
Assuming ‘d’ as a constant and G is already a constant.
g ∝ M
Thus, acceleration due to gravity on any planet is proportional to its mass, i.e., the heavier the planet, the
greater is the acceleration due to gravity, and vice-versa.
(ii) For a given planet, its mass is a constant quantity. Thus in the equation :
M
g = G.
d2
G and M are constants and hence their product is also constant.
constant
\ g =
d2
1
or g ∝
d2

Thus, acceleration due to gravity on a given planet is inversely proportional to the square of distance between
the centre of gravity of planet and centre of body.

3. (a) What do you understand by the term mass?


(b) State two important characteristics of mass.
(c) State units of mass in CGS and SI systems.
(d) Name the device used for measuring mass.
Ans. (a) The amount of matter contained in a body is called its mass.
Or
The measure of the quantity of matter is called its mass.
Mass is an intrinsic i.e. fundamental property of a matter.
(b) Two important characteristics of mass are
1. It is independent of the position and the surroundings of a body.
2. It is a scalar quantity.
(c) Units of Mass
In CGS system, unit of mass is gram (g).
In SI system, unit of mass is kilogram (kg).
(d) Mass is measured by a physical balance.

4. (a) What do you understand by the term weight?


(b) State two important characteristics of weight.
(c) State the units of weight in CGS and SI system.
(d) Name the device used for measuring weight.
Ans. (a) The weight of a body is the force with which it is attracted towards the centre of the earth weight is a
vector quantity.
(b) Two important characteristics of weight are
1. It depends upon the position and the surroundings of the body.
2. It changes from place to place on the surface of Earth on account of the change in the value of ‘g’
acceleration due to gravity.
(c) In CGS system, unit of weight is dyne.
In SI system, unit of weight is newton.
(d) The weight of an object is measured with a spring balance.

54
5.
State four differences between mass and weight :
Ans. Differences between mass and weight :
Mass Weight
1. It is the measure of matter contained in a body. It is the force with which a body is attracted towards
the centre of Earth.
2. It is a constant quantity. Its magnitude It is a variable quantity. It changes with the change
remains unaffected with respect to surroundings in the acceleration due to gravity.
of body.
3. It is found by means of physical balance. It is found by means of spring balance.
4. It is measured in grams or kilograms. It is measured in dynes or newtons.
6. Does a body weight same at all places of the Earth? Give a reason for your answer.
Ans. No, a body does not weigh same at all the places of the Earth.
Expression for weight of a body = mg
Where m mass of the body and g acceleration due to gravity.
Since, earth is not a perfect sphere, so value of acceleration due to gravity changes from place to place so as
weight.
7.
Why is gold is not weighed by a spring balance?
Ans. Weight of a substance is not constant and varies with places. Weight is measured by spring balance. Mass is the
measure of the amount of substance of the body. Since we must know the mass of gold and not its weight so
we do not measure gold using spring balance.
8.
A man sits in a machine which generates acceleration five times more than acceleration due to gravity. If the
mass of man is 80 kg, what is his weight? Take g = 10 ms–2.
Ans. Mass of the man ‘m’ = 80 kg

Acceleration produce by the machine, gm = 5 × g
where g ® Acceleration due to gravity

\ gm = 5 × 10 ms–2 = 50 ms–2

Weight of the man in the machine = mgm

Þ
Weight = 80 × 50 N

= 4000 N
9.
A man weighs 800 N at the equator. How does the weight of man change at the following places?
(a) At poles (b) 100 km up in space (c) 10 km down in a mine
Ans. (a) The earth is not a perfect square. It is somewhat flatterened on the pole and bulging at equator. Thus, polar
radius is somewhat smaller than the equatorial radius. So, the value of acceleration due to gravity is slightly
1
more at poles than at the equator as, g ∝
d2
where d distance of the body from the centre of the earth.
As weight = mg, so a person weighing 800 N; weigh more at poles.
(b) At higher altitude, the distance of the body from the centre of the earth increases hence, acceleration due
to gravity decreases.
Since, weight (W) = mg,
So a person weighing 800 N on the earth, weighs less when moved 100 km up in space.
(c) The acceleration due to gravity decreases as we go deep inside the mine. So, a person weighing 800 N,
weighs less, when he moved 10 km down in a mine.

55
10. How is weight affected in the following cases, when initially the body is weighed in Delhi with a spring balance?
(i) Body is taken to Moscow. (ii) Body is taken to Ceylon
(iii) Body is taken to sea level. (iv) Body is taken to a high mountain.
(v) Body is taken deep inside a mine.
Ans. (i) Weight = mg m ® mass of the body
g ® Acceleration due to gravity
Value of ‘g’ is more at the poles and lesser at the equator. Moscow is more towards the pole, so weight of
a body measured in Moscow will be more than that measured in Delhi.
(ii) The value of ‘g’ is less near the equator Ceylon is located more towards the equator, so the weight of the
body measured in Ceylon will be less than that of measured in Delhi.
(iii) At sea level the body will weight more than that of Delhi.
(iv) At higher altitude (high mountain) the value of ‘g’ decreases. So, weight of the body will less than that in
Delhi.
(v) In deep inside the mine the acceleration due to gravity decreases. So, if the body is taken deep inside a
mine the body will weighted less as compared to Delhi.
11. Describe, briefly, how can you calculate the value of ‘g’ with a simple pendulum.
Ans. A simple pendulum is allowed to oscillate and its time period for one oscillation is noted with the help of stop-
watch by observing large number of oscillations. The length of the simple pendulum is changed several times
and the time period is determined in each case.
l

From the relation T = 2π
g
l

We have, g = 4p2 …(i)
T2
l

By substituting the values of in the above equation the value of ‘g’ can be calculated.
T2

To get a accurate value of ‘g’
l The simple pendulum should have large value of l (1 m or more)
l The angular amplitude should not be more than 5°.

EEEEE

56
4 Pressure in Fluids

UNIT I
Numerical Problems on Pressure in Fluids

Practice Problems 1.
Calculate pressure exerted by 0.8 vertical length of alcohol of density 0.80 g cm–3 in SI units. [Take g = 10 ms–2­]
1.
Solution :
Vertical length of alcohol column (h) = 0.8 m
Density of alcohol (r) = 0.80 gcm–3
= 800 kgm–3
Acceleration due to gravity (g) = 10 ms–2.
\ Pressure (P) = h.r.g. = 0.8 × 800 × 10 Pa


= 6400 Pa

What is the pressure exerted by 75 cm vertical column of mercury of density 13600 kgm–3 in SI units?
2.
[Take g = 9.8 ms–2]
Solution :
Vertical length of mercury column (h) = 75 cm
= 0.75 m
Density of mercury (rHg) = 13600 kgm–3
Acceleration due to gravity (g) = 9.8 ms–2.
\ Pressure (P) = P rHg g


= 0.75 × 13600 × 9.8

= 99960 Pa.

Practice Problems 2.
66640 Pa pressure is exerted by 0.50 m vertical column of a liquid. If g = 9.8 Nkg–1, calculate density of the
1.
liquid.
Solution :
Vertical height of liquid (h) = 0.50 m
Pressure P = 66640 Pa
Acceleration due to gravity (g) = 9.8 Nkg–1
Density of liquid = ?
From the relation
Pressure, P = h.r.g
P
We have, r =
hg

57
66640 66640 ¥ 100

\ r = = = 136 ¥ 100
0.5 ¥ 9.8 5 ¥ 98
= 13600 kgm–3

What vertical height of water will exert pressure of 333200 Pa? Density of water is 1000 kgm–3 and
2.
g = 9.8 ms–2.
Solution :
Pressure (P) = 333200 Pa
Density of water (r) = 1000 kgm–3
Acceleration due to gravity (g) = 9.8 ms–2
Vertical height of water column (h) = ?
From the relation
P = h.r.g
P
We have h =
ρg
333200 33200
\ h = = = 34 m
1000 ¥ 9.8 100 ¥ 98

Pressure at bottom of sea at some particular place is 8968960 Pa. If density of sea water is 1040 kgm–3, calculate
3.
the depth of sea. Take g = 9.8 ms–2. Neglect the pressure of the atmosphere.
Solution :

Pressure at the bottom (P) = 8968960 Pa
Density of sea water (rsea ) = 1040 kgm–3
Depth of the sea (h) = ?
Acceleration due to gravity (g) = 9.8 ms–2
From the relation P = h.r.g
P
We have h =
ρsea ¥ g

8968960
\ h =
1040 ¥ 9.8

= 880 m

Practice Problems 3.
1.
Atmospheric pressure at sea level is 76 cm of mercury. Calculate the vertical height of air column exerting the
above pressure. Assume the density of air 1.29 kgm–3 and that of mercury is 13600 kgm–3. Why the height
calculated by you is far less than actual height of atmosphere?
Solution :
Pressure due to mercury = h Hg ¥ ρHg ¥ g

PHg = 0.76 × 13600 × g …(1)
Let height of air column hair exert some pressure as that of mercury. Density of air (Pair) = 1.29 kgm–3
PAir = hair × rair × g …(2)
Now, equating equation (1) and (2) we have

58
or, hair × rair × g = 0.76 × 13600 × g

0.76 ¥ 13600
or m hair =
1.29
= 8012.4 m

The calculated height is much less than the actual height of atmosphere because the density of atmosphere changes

with height. A fall of 1 cm of mercurry in pressure does not mean that we have covered a vertical height of 105
m. At higher altitudes the vertical height air is far in excess of 105 m because of low density of air.
2.
Calculate the equivalent height of mercury, which will exert as much pressure as 960 m of sea water of density
1040 kgm–3. Density of mercury is 13600 kgm–3.

Solution : ( )
Density of mercury rH g = 13,600 kg m–3
Density of sea water (ρsea water ) = 1040 kg m–3
Height of mercury column (hHg) = ?
Height of sea water column (hsea water) = 960 m
According to question,
Pressure due to mercury column = Pressure due to sea water.
ρHg ¥ h Hg ¥ g = rsea water × hsea water × g
ρ ¥ h sea water

or, hHg = sea water
ρHg
1040 ¥ 960
= = 73.41 m
13600

Practice Problems 4.
1.
The pressure of water on the ground floor, in a water pipe is 150000 Pa, whereas pressure on the fourth floor is
30000 Pa. Calculate height of fourth floor. Take g = 10 ms–2.
Solution :

Pressure of water at ground floor (P1) = 1,50,000 Pa
Pressure of water at fourth floor (P2) = 30,000 Pa
Let h be the height of fourth floor.
Difference in pressure of water at ground floor and fourth floor = P1 – P2
= (150000 – 30000) Pa
= 120000 Pa
Pressure due to height h(Ph) = hrg = hrg = 120000
120000
⇒ h =
ρg
120000

\  h = = 12 m
1000 ¥ 10
2.
The pressure of water on ground floor is 160000 Pa. Calculate the pressure at the fifth floor, at a height of
15 m.

59
Solution :
Height of fifth floor (h) = 15 m
Density of water (r) = 1000 kg m–3
Pressure at ground floor = P1
Pressure at fifth floor = P2
Difference in pressure of water at ground and fifth floor = P1 – P2
Pressure of water due to height h = hrg
P1 – P2 = hrg
Or, 160000 – P2 = 15 × 1000 × 10
Or P2 = 160000 – 150000
\  P2 = 10000 Pa
Practice Problems 5.
1. (a) The area of cross-sections of the pump plunger and press plunger of a hydraulic press are 0.02 m2 and
8 m2 respectively. If the hydraulic press overcomes a load of 800 kgf, calculate the force acting on pump
plunger.
(b) If the mechanical advantage of the handle of pump plunger is 8, calculate the force applied at the end of
the handle of pump plunger.
Solution :
(a) Area of cross-section of pump plunger (A1) = 0.02 m2
Area of cross-section of press plunger (A2) = 8 m2
Load on the press plunger (L) = 800 kgf
Let the effort acting on the pump plunger = E
L A
From the relation, = 2
E A1
800 8
=
E 0.02
800 ¥ 0.02
Or, E = 8
E = 2 kgf
\ Force acting on the pump plunger = 2 kgf

(b) Mechanical advantage of handle of pump plunger = 8


L
Now, M.A = E
handle

Ehandle is the force applied at the handle of the pump plunger.
L 2kgf
Ehandle = =
MA 8
= 0.25 kgf
2.
The radii of the press plunger and pump plunger are in ratio of 50 : 4. If an effort of 20 kgf acts on the pump
plunger, calculate the maximum effort which the press plunger can over come.
Solution :

Let the radius of the press plunger, R = 50x

Radius of the pump plunger, r = 4x

60

Effort acting on the pump plunger = 20 kgf
Maximum effort which press plunger can overcome = L
L πR 2
Now, E = πr 2

L (50 x ) 2


Or, =
20 ( 4 x )2
50 ¥ 50 ¥ 20

Or L =
16
= 3125 kgf

\ Maximum effort which the press plunger can overcome = 3125 kgf

Questions based on ICSE Examination

(A) Objective Questions


I. Multiple Choice Questions.
Select the correct option:
1.
Unit of thrust in SI system is
(a) dynes (b) joule (c) N/m2 (d) newton
The unit Nm–2 is the unit of
2.
(a) force (b) pressure (c) thrust (d) momentum
3.
One Pascal is equal to :
(a) 1 Ncm–2 (b) 1 Nm–2 (c) 1 Nm2 (d) 1 Nm–1
4.
Thrust acting perpendicularly on the unit surface area is called :
(a) pressure (b) moment of force (c) down thrust (d) none of these
5.
Pressure applied in liquids is transmitted with undiminished force :
(a) in downward direction (b) upward direction only
(c) sides of containing vessel (d) in all directions
6.
As we move upwards, the atmospheric pressure:
(a) increases (b) decreases (c) remains same (d) cannot be said
7.
A dam for water reservoir is built thicker at the bottom than at the top because :
(a) pressure of water is very large at the bottom due to its large depth
(b) water is likely to have more density at the bottom due to its large depth
(c) quantity of water at the bottom is large
(c) variation in value of ‘g’.
The pressure exerted by 50 kg (g = 10 m/s2) on an area of cross section of 2 m2 is :
8.
(a) 50 Pa (b) 200 Pa (c) 250 Pa (d) 1000 Pa
9.
Pressure at a point inside a liquid does not depend on :
(a) the depth of the point below the surface of t he liquid
(b) the nature of the liquid
(c) the acceleration due to gravity at that point
(d) the shape of the containing vessel

61
10.
The atmospheric pressure on earth’s surface is approximately
(a) 105 Pa (b) 104 Pa (c) 9.6 × 104 N/m2 (d) 10–4 Pa
Ans. 1. (d) 2.
(b) 3.
(b) 4.
(a) 5.
(d) 6.
(b)
7.
(a) 8.
(c) 9.
(d) 10.
(a)

(B) Subjective Questions


1.
State three factors on which the pressure at a point in a liquid depends.
Ans. Factors on which the pressure at a point in a liquid depends are :
1. Pressure in a fluid is directly proportional to its height (or depth).
2. Pressure in a fluid is directly proportional to its density.
3. Pressure in a fluid is directly proportional to the acceleration due to gravity.
2.
The normal pressure of air is 76 cm of mercury. Calculate the pressure in SI units.
[Density of mercury = 13600 kg/m2 and g = 10 m/s2]

Solution :
Height of mercury column (h) = 76 cm
= 0.76 m
Density of mercury (r) = 13600 kg/m3
Acceleration due to gravity (g) = 10 ms–2
Pressure (P) = hrg­
= 0.76 × 13600 × 10
= 103360 Nm–2
3. At a given place, a barometer records 70 cm of Hg. If the mercury in barometer is replaced by water, what would
be resulting reading?
Solution :
Height of mercury column (h) = 70 cm = 0.70 m
Density of mercury = (r) = 13600 kg/m3
Acceleration due to gravity (g) = 10 ms–2
Pressure (P) = hrg­
Height of water column hw = ?
Pressure due to water column = Pw
Density of water = rw = 1000 kg/m3
Pw = hwrw­ ­g
But, Pw = P
h  0.7 ×13600
or, hwrw g = hrg or hw  = = 9.52 m
w 1000
\ Height of water column = 9.52 m

The base of cylindrical vessel measures 300 cm2. Water is poured into it upto a depth of 6 cm. Calculate the
4.
pressure of water on the base in vessel.
Solution :
Area of base of cylinder (A) = 300 cm2
6
Height (or depth) of water column (h) = 6 cm = = 0.06 m
100

62
Density of water (r) = 1000 kg/m3
Acceleration due to gravity (g) = 10 m/s2
Pressure at the base (P) = hrg
= 0.06 × 1000 × 10
= 600 Pa
Pressure on the base in vessel = 600 Pa.
5.
The pressure in water pipe on the ground floor of a building is 40000 pascals, whereas on the first floor it’s 10000
pascals. Find the height of first floor. (Acceleration due to gravity g = 10 ms–2)
Solution :
Pressure in water pipe on the ground floor of the building, P1 = 40000 Pa
Pressure in water pipe on the first floor of the building, P2 = 10000 Pa
Density of water (r) = 1000 kg m–3
Acceleration due to graity (g) = 10 m s–2
Difference in pressure (P1 – P2) = 40000 – 10000 = 30000 Pa
Let h = height of the first floor
Pressure due to height (h) = hrg or, 30000 = h × 1000 × 10
or h = 3 m
\ Height of first floor = 3 m
6. (a) Define SI unit of pressure.
(b) The atmospheric pressure at a place is 650 mm of Hg. Calculate this pressure in Pascals (Pa).
Ans. (a) The SI unit of pressure is pascal (Pa) or Nm–2.
When a force of one newton acts normally on an area of one square metre (1 m2) then pressure acting on
the surface is called one pascal.
(b) Height of mercury column (h) = 650 mm = 0.65 m
Density of mercury (r) = 13600 kg/m3
Acceleration due to gravity (g) = 10 ms–2
Pressure (P) = hrg = 0.65 × 13600 × 10 = 88400 Pa.
7. Pressure in a water pipe on the ground floor of a building is 100,000 Pa. Calculate the pressure in water pipe on
first floor at a height of 3 m. [Density of water = 1000 kgm–3; g = 10 ms–2]
Ans. Pressure in a water pipes on the ground floor = P1 = 100,000 Pa
Pressure in water pipe on the first floor = P2 = ?
Height of first floor, h = 3 m
Density of water, r = 1000 kg/m3
Acceleration due to gravity, g = 10 m/s2
Now, P1 – P2 = hrg
⇒ 100000 – P2 = 3 × 1000 × 10
P2 = 100000 – 30000 = 70,000 Pa

\ Pressure of water in pipe at first floor of a building is 70000 Pa.

8.
P is the pressure at some point in a liquid. State whether pressure P is a scalar or vector quantity.
Ans. Pressure exerted on an enclosed fluid gets transmitted equally and undiminishingly in all directions. So specific
direction is not associated with pressure.
That is why pressure is a scalar quantity.

63
9.
A beaker contains a liquid of density ‘r’ upto height ‘h’, such that ‘PA’ is atmospheric pressure and ‘g’ is
acceleration due to gravity. Answer the following questions :
(a) What is the pressure on the free surface of liquid?
(b) What is the pressure on the base of beaker?
(c) What is the lateral pressure at the base on the inner walls of beaker?
Ans. (a) Pressure on the free surface of liquid is equal to the atmospheric
pressure (PA).
(b) Consider a liquid contained in a beaker, such that the ‘r’ is the
density of liquid. Consider a point B at the base of liquid and the
liquid column of area of cross-section ‘A’ around it, such that ‘h’
is the height of the liquid column as shown in the figure.
\ Thrust exerted by liquid on the base of the beaker (F) = A.hr.g.
\ Pressure due of liquid column of height h
Force F A.hρg
P= = =
Area A A
P = hρg
So, pressure on the base of beaker = hrg
\ Total pressure at the base of the beaker.
= Atmospheric pressure + rgh
= PA + rgh.
(c) Lateral pressure at the base on the inner walls of beaker = PA + hrg

10. State the law of transmission of pressure in liquids.


Ans. Pascal’s Law :

The pressure applied on the surface of a confined liquid is transmitted equally and undiminished in all directions
throughout the liquid.
11. Calculate the hydrostatic pressure exerted by water at the bottom of a beaker. Take the depth of water as
40 cm, the density of water 1000 kgm–3 and g = 9.8 ms–2.
Ans. Depth of water (h) = 40 cm = 0.4 m
Density of water (r) = 1000 kg/m3
Acceleration due to gravity (g) = 9.8 ms–2
Pressure P = ?
P = hrg
= 0.4 × 1000 × 9.8 Pa = 3920 Pa
12. State Pascal’s law of transmission of pressure in a liquid.
Ans. Pascal’s law states that ‘whenever any pressure is applied to any part of the boundary of a confined fluid, it is
transmitted equally in all directions irrespective of the area on which it acts and always acts at right angles to
the surface of containing vessel.
13. State briefly, how and why the atmospheric pressure of a place varies with the altitude. Draw an approximate
graph to illustrate this variation.
Ans. Atmospheric pressure P = hrg
where h ® Height of air column
r ® density of air
g ® Acceleration due to gravity

64
As we go up, the length of the air column exerting pressure
decreases and hence the atmospheric pressure decreases.
Moreover, while going upward the density of air decreases,
which also accounts for the decrease in the pressure.
If we take the average density of air as 1.29 kgm–3
and the density of mercury as 13600 kg m–3, we can
find the height of air column which will exert as much
pressure as it is exerted by 1 cm column of mercury as
(0.01 m) as below :
hair × rair × g = hHg × rHg × g
0.01 × 13600
or hair = m
1.29
≈ 105 m
Thus, 1 cm of mercury column exerts pressure = 105 m
of air column
 105 × 76 m
76 cm of mercury column exerts pressure =
= 7980 m ≈ 8 km
Thus, 8 km of air column will exert as much pressure as 76 cm of mercury column. However, it does not mean
that atmosphere extends only 8 km. As the density of atmosphere also changes with height. Thus, a fall of one
cm in pressure does not mean that we have covered a vertical height of 105 m. On higher altitude vertical height
of air is far excess of 105 m, because of low density of air. A graph showing fall in pressure is shown in figure.
14. The blood pressure reading of a patient is recorded 160/100. Express the lower pressure in SI units.
[Take density of mercury as 13.6 × 103 kgm–3 and the value of ‘g’ as 10 ms–2]

Ans. Lower pressure of the patient (P) = hrg
h ® h eight of the mercury column = 100 mm = 0.1 m

Density of mercury = 13.6 × 103 kg m–3
 0.1 × 13.6 × 103 × 10 Pa
Pressure =
= 13600 Pa
15. State two advantages of aneroid barometer
Ans. Two advantages of aneroid barometer are as follows :
1.
It does not contain any liquid and there is no chance of spilling over of liquid as in mercury barometer.
2. It is compact and portable and can be carried anywhere.
16. Explain, why a gas bubble released at the bottom of a lake grows in size as it rises to the surface of the lake.
Ans. Bubbles released at the bottom of a lake grows in size as it rises to the surface of the lake because the pressure
exerted on it by the water decreases. Due to decrease in depth, the pressure due to water column decreases.

EEEEE

65
5 Upthrust and Archimedes' Principle

UNIT I
Numerical Problems on Upthrust and Archimedes’ Principle

Practice Problems 1.
A solid of density 2700 kgm–3 and of volume 0.0015 m3 is completely immersed in alcohol of density 800 kgm–3.
1.
Calculate :
(i) Weight of solid in SI system.
(ii) Upthrust on solid in SI system.
(iii) Apparent weight of solid in alcohol.
(iv) Will the apparent weight of solid be less or more, if it is immersed completely in brine solution? Give
a reason. [g = 10 ms–2]
Solution :
Density of solid (rsolid) = 2700 kg/m3
Volume of solid (V) = 0.0015 m3
Density of alcohol (ralcohol) = 800 kg/m3
(i) Mass of the solid (m) = Vr
= 0.0015 × 2700 kg
= 4.05 kg
\ Weight of solid = mg = 4.05 × 10
= 40.5
(ii) Upthrust = Weight of alcohol displaced
= mass of alcohol displaced × Acceleration due to gravity
= 0.0015 × 800 × 10
= 12 N
(iii) Apparent weight of solid in alcohol
= Actual weight of solid – Upthrust
= 40.5 – 12 = 28.5 N
(iv) When a solid is immersed completely in brine solution, then upthrust acts on it in upward direction, as
a result, its apparent weight of solid will be less than the actual weight of solid. Thus, solid will weigh
less.
A stone of density 3000 kgm–3 is lying submerged in water of density 1000 kgm–3. If the mass of stone in air
2.
is 150 kg, calculate the force required to lift the stone. [g = 10 ms–2]
Solution :
Density of stone (rstone) = 3000 kgm–3
Density of water (rwater) = 1000 kgm–3
Mass of stone (m) = 150 kg
Acceleration due to gravity = 10 ms–2
m 150 5
Volume of stone V = = = = 0.05 m3
ρstone 3000 100
Actual weight of the stone = 150 × 10 = 1500 N

66
Volume of water displaced = Volume of stone
= 0.05 m3
Mass of water displaced = m ρ water = V × ρwater
= 0.05 × 1000
= 50 kg
Upthrust = mwater × g
= 50 × 10 = 500 N
Force required to lift the stone = Actual weight of the stone – upthrust
= 1500 – 500 = 1000 N

A solid of area of cross-section 0.004 m2 and length 0.60 m is completely immersed in water of density
3.
1000 kgm–3.

Calculate : (i) Wt. of solid in SI system

(ii) Upthrust acting on the solid in SI system.

(iii) Apparent weight of solid in water.

(iv) Apparent weight of solid in brine solution of density 1050 kgm–3.
[Take g = 10 N/kg; Density of solid = 7200 kgm–3]

Solution :
Area of cross-section of solid, (A) = 0.004 m2
Length of the solid (l) = 0.60 m.
Density of water (rwater) = 1000 kg/m3
Density of solid (rsolid) = 7200 kg/m3
Acceleration due to gravity g = 10 ms–2
(i) Volume of solid V = A × l = 0.004 × 0.60 = 0.0024 m3
Mass of solid = V × rsolid
= 0.0024 × 7200
= 17.28 kg
Weight of the solid = 17.28 kg × 10
= 172.8 N
(ii) Volume of water displaced = Volume of solid
= 0.0024 m3
Mass of water displaced = m1 = V × rwater
= 0.0024 × 1000 = 2.4 kg
Upthrust = Weight of water displaced
= 2.4 × 10 = 24 N
(iii) Apparent weight of solid in water = Actual weight of solid – upthrust
= 172.8 – 24
= 148.8 N
(iv) Density of brine solution (rbrine) = 1050 kgm–3.
Volume of brine solution displaced = Volume of solid = V = 0.0024 m3
Mass of brine solution displaced = 0.0024 × 1050 kg
= 2.52 kg
Upthrust acting on the solid = Weight of brine solution
= 2.52 × 10 = 25.2 N
\ Apparent weight of solid in brine solution = Actual weight – Upthrust
= 172.8 – 25.2 = 147.6 N

67
Practice Problems 2.
4
A solid of density 7600 kgm–3 is found to weigh 0.950 kgf in air. If
1. volume of solid is completely immersed
5
in a solution of density 900 kgm–3, find the apparent weight of solid in liquid.
Density of solid (rsolid) = 7600 kgm–3
Solution :
0.950 95 1.25
Volume = = = = 0.000125 m3
7600 7600 ×100 10000
Density of solution (rsolution) = 900 kgm–3
4
Q volume of the solid is completely immersed in the given solution
5
4
\ Volume of solution displaced V = × Volume of solid
5
4 0.0005
= × 0.000125 =
5 5
= 0.0001 m3
Mass of solution displaced = Volume of solution displaced × density of solution
= 0.0001 × 900 = 0.09 kg

Upthrust =  Weight of solution displaced


= 0.09 × 10 = 0.9 N = 0.09 kgf
Apparent weight of solid in liquid
= Actual weight – Upthrust
= 0.950 – 0.09 = 0.860 kgf
2. A glass cylinder of length 12 × 10–2 m and area of cross-section 5 × 10–4 m2 has a density of 2500 kgm–3. It is
3
immersed in a liquid of density 1500 kgm–3, such that of its length is above liquid. Find the apparent weight
of glass cylinder in newtons. 8

Solution :
Length of glass cylinder (l) = 12 × 10–2 m
Area of cross-section (A) = 5 × 10–4 m2
Volume of the cylinder (V) = A × l
= 5 × 10–4 × 12 × 10–2 m3
= 0.00006 m3
Acceleration due to gravity g = 10 N/kg
3
Since length of the cylinder is above the liquid
8
3 5
Hence, length of the cylinder inside the liquid = 1 − =
8 8
5
\ Volume of the liquid displaced by the glass cylinder = × volume of glass cylinder
8
5 0.0003
= × 0.00006 = = 0.0000375 m3
8 8
Mass of the glass cylinder = V × r
= 0.00006 × 2500
= 0.15 kg
Weight of glass cylinder = mg = 0.15 × 10 = 1.5 N

68
Mass of liquid displaced by glass cylinder = 0.0000375 × 1500 = 0.05625 kg
Upthrust = weight of liquid displaced by the glass cylinder
= 0.05625 × 10 = 0.5625 N
Apparent weight of glass cylinder in liquid
= Actual weight of glass cylinder – Upthrust
= 1.5 – 0.5625 = 0.9375 N
Practice Problems 3.
1. A solid weighs 0.08 kgf in air and 0.065 kgf in water. Find (i) R.D. of solid (ii) Density of solid in SI system.
[Density of water = 1000 kgm–3]
Solution :
Weight of solid in air = 0.08 kgf
Weight of solid in water = 0.065 kgf
Density of water = 1000 kgm–3
Wt. of solid in air
(i) Relative density of solid =
Wt. of solid in air − Wt. of solid in watter
0.08 0.08 80 16
= = = = = 5.3333
0.08 − 0.065 0.015 15 3
Density of solid
(ii) Relative density of solid =
Density of water
\ Density of solid = R.D of solid × Density of water
= 5.3333 × 1000 kgm–3
= 5333.3 kgm–3
2. A solid of R.D. 2.5 is found to weigh 0.120 kgf in water. Find the wt. of solid in air.
Solution :
Relative density of solid = 2.5

Weight of solid in water (Wwater) = 0.120 kgf

Weight of solid in air (Wair) = ?
Weight of solid in air
Relative density =
Wt. of solid in air − Wt. of solid in water
Wair

Or, R.D.
=
Wair − Wwater
Wair
=
⇒ 2.5
Wair − 0.120
Or, 2.5 Wair – 2.5 × 0.120 = Wair

Or, 1.5 Wair = 0.3

0.3 1

⇒ Wair = =
1.5 5

⇒ Wair = 0.20 kgf.

\ Weight of solid in air = 0.20 kgf.

69
3. A solid of R.D. 4.2 is found to weigh 0.200 kgf in air. Find its apparent weight in water.
Solution : Relative density of solid R.D. = 4.2
Wt. of solid in air
⇒ R.D. =
Upthrust
0.200
⇒  4.2 =
Upthrust
0.200 1
Or Upthrust = = = 0.0476 kgf
4.2 21
So, apparent weight of the solid in water
= Wt of solid in air – Upthrust
= 0.200 – 0.0476
= 0.1524 » 0.15 kgf

Practice Problems 4.
1.
A sinker is found to weigh 56.7 gf in water. When the sinker is tied to a cork of weight 6 gf, the combination
is found to weigh 40.5 gf in water. Calculate R.D. of cork.
Solution :

Weight of sinker in water = 56.7 gf
Weight of cork in air = 6 gf
Weight of sinker in water + weight of cork in air = 56.7 + 6 = 62.7 gf …(i)
Weight of sinker in water + weight of cork in water = 40.5 gf …(ii)
Subtracting equation (ii) from equation (i) we have
Weight of cork in air – weight of cork in water = 62.7 – 40.5 = 22.2 gf
Weight of cork in air
R.D. of cork =
Weight of cork air − Weight of cork in water
6
= = 0.27
22.2

Relative density of cork = 0.27


2. A solid lighter than water is found to weigh 7.5 gf in air. When tied to a sinker the combination is found to
weigh 62.5 gf. If the sinker alone weighs 72.5 gf in water, find R.D. of solid.
Solution :

Weight of solid in air = 7.5 gf
Weight of sinker in water = 72.5 gf
Weight of sinker in water + weight of solid in air = 72.5 + 7.5 = 80 gf …(i)
Weight of sinker in water + weight of solid in water = 62.5 gf …(ii)
Subtraction equation (ii) from (i)
Weight of solid in air – weight of solid in water = 80 – 62.5 = 17.5 gf
weight of solid in air
Relative density of solid =
weight of solid in air − weight of solid in water
7.5
= = 0.428

17.5

70
Practice Problems 5.
1.
An aluminium cube of side 5 cm and R.D. 2.7 is suspended by a thread in alcohol of relative density 0.80. Find
the tension in thread.
Solution :
Side of an aluminium cube = l = 5 cm
Volume of aluminium cube (V) = l3 = 125 cm3
Relative density of aluminium (R.D.) = 2.7
Relative density of alcohol (R.D.) = 0.80
\ Density of aluminium = r = 2.7 g cm–3
Mass of aluminium = V × r
= 125 × 2.7 = 337.5 g
Wt. of aluminium cube acting downward = 337.5 gf
Volume of alcohol displaced = Volume of cube = 125 cm3
Upthrust due to alcohol = V × ralcohol × g

Now, relative density of alcohol = ρalcohol = 0.80 gcm −3


\ Upthrust due to alcohol = V × ralcohol × g
= 125 × 0.80 × g = 100 gf
So, tension in thread = Wt of aluminium cube – Upthrust
= 337.5 – 100 = 237.5 gf
2.
A cube of lead of side 8 cm and R.D. 10.6 is suspended from the hook of a spring balance. Find the reading of
spring balance. The cube is now completely immersed in sugar solution of R.D. 1.4. Calculate the new reading
of spring balance.
Solution : Length of side of cube = l = 8 cm
Volume of cube = l3 = (8)3 = 512 cm3
Relative density of lead cube = R.D. = 10.6
Relative density of sugar solution R.D. = 1.4

\ Density of lead = rlead = 10.6 gcm–3
Mass of lead = V × rlead
= 512 × 10.6
= 5427.2 g
Weight of lead cube = mg
= 5427.2 × g
= 5427.2 gf
Volume of sugar solution displaced = Volume of lead cube
= 512 cm3
\ Density of sugar solution = rsugar = 1.4 g cm–3­
Upthrust due to sugar solution = V × rsugar × g
= 512 × 1.4 × g
= 716.8 gf
Now, reading of spring balance = Actual weight – Upthrust
= 5427.2 – 716.8
= 4710.4 gf

71
Numerical Problems on The Law of Floatation
Practice Problems 1.
3
A hollow cylinder of copper of length 25 cm and area of cross-section 15 cm2, floats in water with
1. of its
5
length inside water. Calculate : (i) apparent density of hollow copper cylinder (ii) wt. of cylinder (iii) extra force
required to completely submerge it in water.
Solution :

Length of the cylinder = 25 cm
Area of cross section = 15 cm2
Let hwater = length of cylinder inside the water
3 3
× l = × 25 = 15 cm
hwater =
5 5
Density of water (rwater) = 1 gcm –3

(i) Apparent density of hollow cylinder = rCu = ?


By law of flotation
hCu × rCu = hwater × rwater
or, 25 × rCu = 15 × 1
15
rCu = = 0.6 gcm–3
25
(ii) Volume of cylinder V = A × hCu
Mass of hollow cylinder = Ahr
=15 × 25 × 0.6
= 225 g
Weight of hollow cylinder of copper = mg
= 225 × g = 225 gf
(iii) Total upthrust when hollow copper cylinder is completely immersed in water = Vrwater g
= 15 × 25 × 1 × g = 375 gf
Extra force required to submerge completely the cylinder in water
= Upthrust – down thrust
= 375 – 225 = 150 gf

3
A cork cut in the form of a cylinder floats in alcohol of density 0.8 gcm–3, such that
2. of its length is outside
7
alcohol. If the total length of cylinder is 35 cm and area of cross-section 25 cm2, calculate : (i) density of cork
(ii) wt. of cork (iii) extra force required to submerge it in alcohol.
Solution :
Density of alcohol = 0.8 gcm–3
Let l = length of the cork = 35 cm
 3
(i) Length of the cork inside the alcohol = 1 −  × l
 7

72
 7 − 3
= × 35 = 20 cm
 7 

Density of water = rwater = 1 g cm­–3
By law of floatation, hcork × rcork = halcohol × ralcohol

35 × rcork = 20 × 0.8
20 × 0.8 16
= or, rcork =
35 35
= 0.457 g cm –3

(ii) Weight of cork = ?


Area of cross section of cork = 25 cm2
Volume of Cork = A × hcork.
= 25 × 35
= 875 cm3
Mass of the cork = V × rcork
= 875 × 0.457
= 399.875 » 400 g
Weight of cork = 400 gf
(iii) Total upthrust when cork is completely immersed in water

= V × ralcohol × g

= 875 × 0.8 × g = 700 gf

\ Extra force required to submerged completely the cork in alcohol = Upthrust – downthrust

= 700 – 400

= 300 gf
Practice Problems 2.
A cylinder made of copper and aluminium floats in mercury of density 13.6 gcm–3, such that 0.26th part
1.
of it is below mercury. Find the density of solid.
Solution :

Density of mercury (rHg) = 13.6 gcm3

Density of solid cylinder = rsolid = ?
0.26th part of the cylinder is below mercury


Let Vsolid = Volume of solid cylinder

Volume of the mercury displaced by immersed part of the solid cylinder = 0.26 × Vsolid

By the law of floatation,

Vsolid × rsolid = 0.26 Vsolid × rHg

⇒    rsolid = 0.26 × 13.6
= 3.536 gcm–3
So density of solid = 3.536 g cm–3
2
2. An iceberg floats in sea water of density 1.17 gcm–3, such that of its volume is above sea water. Find
9
the density of iceberg.

73
Solution :

Density of sea water (rsea) = 1.17 g cm–3

Density of iceberg (rice) = ?

Let V = volume of ice
2
th volume of iceberg is above the sea water
9
 2 7
\ Volume of iceberg inside sea water = 1 −  V = V
 9 9
7
Volume of sea water displaced by iceberg (Vwater) = V
9
By law of floatation
Weight of iceberg = Weight of sea water displaced by iceberg
V × rice × g = Vwater × rsea × g

⇒    Vrice = 7 V × 1.17
9
7 × 1.17
rice = = 7 × 0.13 = 0.91 g cm–3
9
Practice Problems 3.
3
1.
A wooden block floats in alcohol with of its length above alcohol. If it is made to float in water, what fraction
8
of its length is above water? Density of alcohol is 0.80 gcm–3.
Solution :

Let l be the length of the wooden block
3

\ Length of the block above alcohol =  l
8
 3 5l
Length of block below alcohol = 1 −  l =
 8 8

Density of water rw = 1gcm–3


Density of alcohol (ralcohol) = 0.80 gcm–3
By the law of floatation,
lblock × rblock = lalcohol × ralcohol
5l
⇒   l × rblock = × 0.80
8
Or,
rblock = 0.5 gcm–3
When block is floating in water
By the law of floatation,
lblock × rblock = lwater × rwater


⇒   l × 0.5 = lwater × 1

1
lwater = l.
2
1
So, length of the block below water =
l
2

74
1 1
Length of the block above water = l – l= .
2 2
l 1 1
\ Fraction of block above water = × = part
2 l 2
1
Hence, th part of wooden block is above the water.
2
A hollow metal cylinder of length 10 cm floats in alcohol of density 0.80 gcm–3, with 1 cm of its length above
2.
it. What length of cylinder will be above copper sulphate solution of density 1.25 gcm–3?
Solution :

Length of hollow metal cylinder = 10 cm
Length of cylinder above the alcohol = 1 cm

\ Length of cylinder below alcohol = 9 cm
By the law of floatation
10 × rcylinder = 9 × ralcohol
9 × 0.80
rcylinder = = 0.72 g cm −3
10
When the block is floats in copper sulphate solution
By the law of floatation
lcylinder × rcylinder = lCuSO × rCuSO
4 4
⇒     10 × 0.72 = lCuSO × 1.25
4
10 × 0.72
Or, lCuSO = = 5.76 cm
4 1.25
\ Length of metal cylinder below copper sulphate solution = 5.76 cm.
So, length of cylinder above copper sulphate solution = 10 – 5.76 = 4.24 cm
Practice Problems 4.
What fraction of an iceberg of density 910 kgm–3 will be above the surface of sea water of density
1.
1170 kgm–3?
Solution :
Let the volume of iceberg = Vice
Volume of iceberg inside water = Vwater
Density of iceberg (rice) = 910 kgm–3
Density of sea water (rwater) = 1170 kg m–3
By law of floatation,
Weight of iceberg = Weight of sea water displaced by the iceberg
Vice × rice × g = Vwater × rwater × g
⇒     Vice × 910 = Vwater × 1170
Vwater 910 7
Or = =
Vice 1170 9
7 2
\ Volume of iceberg above the sea water = Vice − Vice = Vice
9 9
2
Hence, fraction iceberg above sea water = th part
9

75
What fraction of metal of density 3400 kgm–3 will be above the surface of mercury of density 13600 kgm–3,
2.
while floating in mercury?
Solution :
Density of metal (rm) = 3400 kg m–3
Density of mercury (rHg) = 13600 kgm–3
Let the volume of the metal = V1
Volume of the metal inside mercury = V2
By the law of floatation,
Weight of mercury displaced by metal = Weight of metal
Volume of mercury × density of mercury × g = Volume of metal × density of metal × g
Or, V2 × 13600 = V1 × 3400
V2 3400 1
Or, = 13600 = 4
V1
Volume of the metal above the surface of mercury
= V1 – V2
V1
= V1 –
4
3
= V1
4
3 1 3
Fraction of metal above the surface of mercury =
V1 × =
4 V1 4
\ 3 th part of metal lies above the surface of mercury.

4
Practice Problems 5.
A balloon of volume 1000 m3 is filled with a mixture of hydrogen and helium of density 0.32 kgm–3. If the
1.
fabric of balloon weighs 40 kgf and the density of cold air is 1.32 kgm–3, find the tension in the rope, which is
holding the balloon to ground.
Solution :
Volume of balloon (V) = 1000 m3
Density of mixture of hydrogen and helium (r) = 0.32 kgm–3
Weight of empty balloon = 40 kgf
Density of cold air = rcold air = 1.32 kgm–3
Volume of balloon = Volume of mixture of hydrogen and helium
= Volume of cold air displaced
= 1000 m3
Weight of mixture of hydrogen and helium gas in balloon
= Vrg = 1000 × 0.32 × g = 320 kgf
Downthrust = Weight of empty balloon + Weight of mixture of hydrogen and helium gas
= 40 + 320 = 360 kgf
Upthrust = Weight of cold air displaced by balloon = Vrcold air g
= 1000 × 1.32 × g = 1320 kgf
Tension in the rope = Upthrust – down thrust = 1320 – 360 = 960 kgf

76
A balloon of volume 800 cm3 is filled with hydrogen gas of density 9 × 10–5 gcm–3. If the empty balloon weighs
2.
0.3 gf and density of air is 1.3 × 10–3 gcm–3, calculate the lifting power of balloon.
Solution :
Volume of balloon = 800 cm3
Density of hydrogen (rH) = 9 × 10–5 gcm–3
Weight of empty balloon = 0.3 gf
Density of air = rair = 1.3 × 10–3 gcm–3
Volume of hydrogen gas = Volume of balloon = 800 cm3
Weight of hydrogen gas = 800 × 9 × 10–5 × g
= 0.072 gf
Down thrust = Weight of empty balloon + Weight of hydrogen gas in balloon
= 0.3 + 0.072 = 0.372 gf
Volume of air displaced by balloon = Volume of balloon
Upthrust = Wt. of air displaced by balloon
= Vrair g
= 800 × 1.3 × 10–3 × g
= 1.04 gf
Lifting power of balloon = Upthrust – Downthrust
= 1.04 – 0.372
= 0.668 gf.
A balloon of volume 120 m3 is filled with hot air of density 0.38 kgm–3. If the fabric of balloon weighs 12 kg,
3.
such that an additional equipment of wt. x is attached to it, calculate the magnitude of x. Density of cold air is
1.30 kgm–3.
Solution :
Volume of balloon (V) = 120 m3
Density of hot air = 0.38 kgm–3
Mass of the empty balloon = 12 kg
Weight of empty balloon = 12 kgf
Weight of additional equipment attached with the balloon = x kgf.
Density of cold air, rcold air = 1.30 kgm–3
Volume of balloon = Volume of hot air inside the balloon
= Volume of cold air displaced by balloon = 120 m3
Weight of hot air = V rhot air g
= 120 × 0.38 × g
= 45.6 kgf
Downthrust = Weight of empty balloon + Weight of hot air inside
+ Weight of equipment
= 12 + 45.6 + x = 57.6 + x
Upthrust = Weight of cold air displaced by balloon
= V × rcold air × g
= 120 × 1.30 × g
= 156 kgf
By Law of floatation
Down thrust = Upthrust
57.6 + x = 156
x = 156 – 57.6
= 98.4 kgf
77
Practice Problems 6.
1.
A test tube weighing 17 gf, floats in alcohol to the level P. When the test tube is made to float in water to the
level P, 3 gf of the lead shots are added in it. Find the R.D. of alcohol.
Solution :
When the test tube floats in alcohol
Weight of test tube = 17 gf
By law of floatation
Weight of alcohol displaced by test tube =
Weight of test tube = 17 gf
Weight of water displaced =
Weight of test tube + Weight of lead stones
= 17 + 3 = 20 kgf
As, volume of alcohol displaced =
Volume of water displaced
Weight of alcohol displaced
\ R.D of alcohol =
Weight of equal volume of waterr displaced
17
= = 0.85
20
2. A test tube loaded with lead shots, weighs 150 gf and floats upto the mark X in water. The test tube is then made
to float in alcohol. It is found that 27 gf of lead shots have to be removed, so as to float it to level X. Find R.D.
of alcohol.
Solution :
Weight of test tube = 150 gf
Weight of water displaced = Weight of test tube = 150 gf
Weight of alcohol displaced = 150 – 27 = 123 gf
By law of floatation,
Volume of water displaced = Volume of alcohol displaced
Weight of alcohol displaced
Now, R.D. of alcohol =
Weight of equal volume of water displaced

123
= = 0.82
150

\ Relative density of alcohol = 0.82

Questions based on ICSE Examinations

(A) Objective Questions


Multiple Choice Questions.
Select the correct option:
1.
The force experienced by a body when partially or fully immersed in water is called :
(a) apparent weight (b) upthrust (c) down thrust (d) none of these
2.
When a body is floating in a liquid :
(a) the weight of the body is less than the u pthrust due to immersed part of the body
(b) the weight of body is more than the upthrust d ue to the immersed part of the body
(c) the weight of body is equal to the upthrust d ue to the immersed part of the body
(d) none of the above

78
3.
With the increase in the density of the fluid, the upthrust experienced by a body immersed in it :
(a) decreases (b) increases (c) remains same (d) none of these
4.
The apparent weight of a body in a fluid is:
(a) equal to weight of fluid displaced (b) volume of fluid displaced
(c) difference between its weight in air and w
eight of fluid displaced
(d) none of the above
5.
The phenomenon due to which a solid experiences upward force when immersed in water is called :
(a) floatation (b) buoyancy (c) density (d) none of these
6.
When an object sinks in a liquid, its :
(a) buoyant force is more than the weight of object
(b) buoyant force is less than the weight of object
(c) buoyant force is equal to the weight of the object
(d) none of the above
7. The SI unit of density is :
(a) gcm–3 (b) kgcm–3 (c)
kgm–3 (d) gm–3
8. When a body is wholly or partially immersed in a liquid, it experiences a buoyant force which is equal
to:
(a) volume of liquid displaced by it (b) weight of liquid displaced by it
(c) both (a) and (b) (d) none of these
9.
The ratio between the mass of a substance and the mass of an equal volume of water at 4°C is called :
(a) relative density (b) density (c) weight (d) pressure
A body has density 9.6 gcm–3. Its density in SI system is :
10.
(a) 96 kgm–3 (b) 960 kgm–3 (c) 9600 kgm–3 (d) 96,000 kgm–3

Answers
1.
(b) 2. (a) 3. (b) 4. (a) 5. (b) 6. (b) 7. (c)
8. (b) 9. (a) 10. (c)

(B) Subjective Questions


1.
A wooden block floats in water with two third of its volume submerged.
(i) Calculate density of wood.
(ii) When the same block is placed in oil, three quarter of its volume is immersed in oil. Calculate the
density of oil.

Solution : Let the volume of wood = V


2
(i) Volume of wood submerged (V¢) = V
3
Let density of wood = rwood
density of water = rwater
2
V
ρwood V  3 2
  
ρwater V V 3

or, rwood = 2 × rwater


3
79
Q Density of water = 1 gcm–3

\ rwood = 2 g cm–3 = 0.66 g cm–3



3
(ii) Let roil is the density of oil
3
ρblock V
3
Now, = 4 =
ρoil V 4
4 4 2
⇒ ρoil = × ρblock = ×
3 3 3
8
⇒ roil =
= 0.89 g cm–3
9

2.
A metal cube of 5 cm edge and relative density 9 is suspended by a thread so as to be completely immersed
in a liquid of relative density 1.2. Find the tension in the thread.
Solution :

Volume of metal cube = (side)3 = (5)3 = 125 cm3


Density of cube = 9 g cm–3
Weight of the cube acting downward
= mg = 125 × 9 × g = 1125 gf
Density of fluid (rfluid) = 1.2 g cm–3

Upthrust due to liquid = V rfluid g


= 125 × 1.2 g
= 150 gf
Tension in the string = Net downward force
= 1125 – 150 = 975 gf
A weather forecasting plastic balloon of volume 15 m3 contains hydrogen of density 0.09 kgm–3. The
3.
volume of equipment carried by the balloon is negligible compared to its own volume. The mass of the
empty balloon is 7.15 kg. The balloon is floating in air of density 1.3 kgm–3.
(i) Calculate the mass of hydrogen in balloon.
(ii) Calculate the mass of hydrogen and the balloon.
(iii) If the mass of equipment is x kg, write down the total mass of hydrogen, the balloon and the
equipment.
(iv) Calculate the mass of air displaced by balloon.
(v) Using the law of floatation, calculate the mass of equipment.

Solution :
Volume of hydrogen balloon = 15 m3
Density of hydrogen = 0.09 kgm–3
(i) Mass of hydrogen in balloon
= 15 × 0.09 kg
= 1.35 kg
(ii) The mass of empty balloon = 7.15 kg
\ The mass of hydrogen and balloon

80
= 7.15 + 1.35
= 8.5 kg
(iii) Mass of equipment = x kg,
Total mass of hydrogen, the balloon and the equipment = x + 8.5 kg
(iv) Density of air = 1.3 kg m–3
Mass of air displaced by balloon = 15 × 1.3 = 19.5 kg
(v) According to law of floatation,
Total downward weight = upthrust
(8.5 + x)g = 19.5 g
or x = 19.5 – 8.5
= 11 kg
4.
(a) State the principle of floatation.
(b) The mass of a block made of certain material is 1.35 kg and its volume is 1.5 × 10–3 m3.
(i) Find the density of block.
(ii) Will this block float or sink? Give reasons for your answer.

Solution :
Principle of floatation : When a solid is floating in a fluid, then the weight of whole solid acting
(a) 
vertically downward at its centre of gravity, is equal to the weight of fluid displaced by immersed part
of a solid acting vertically upward, at its centre of buoyancy.

(b) (i) Mass of the block = 1.35 kg

Volume of the block = 1.5 × 10–3 m3
Mass

Density of the block =
Volume
1.35
= −3
kgm −3

1 . 5 × 10
= 900 kgm–3

(ii) The density of the block is 900 kgm–3 which is less than the density of water. Hence, the block
will float in water.
5.
(a) State Archimedes’ Principle.
(b) A block of mass 7 kg and volume 0.07 m3 floats in a liquid of density 140 kg/m3. Calculate :
(i) Volume of block above the surface of liquid.
(ii) Density of block.
Solution :
(a) Archimedes’ Principle states that when a body is partially or wholly immersed in a fluid, it experiences an
upthrust or there is apparent loss in its weight, which is equal to the weight of the fluid displaced by the
immersed part of the body.
(b) (i) Mass of block = 7 kg
Volume of block (V) = 0.07 m3 ;


Density of the block = r
Density of liquid (rliquid) = 140 kgm–3


Let V1 = Volume of block immersed in the liquid
By law of floatation,

81
Weight of block = Weight of liquid displaced by the immersed part
Vrg = V1 rliquidg …(i)
7
Density of the block = = 100 kg m–3
0.07
Now from equation (i),
V × ρ 0.07 × 100 7 1
V1 = = = =
ρliquid 140 140 20

V1 = 0.05 m3

\ Volume of the block above the surface
= V – V1
= 0.07 – 0.05 = 0.02 m3
Mass of block
(ii) Density of the block =

Volume of block
7
= = 100 kg m −3

0.07
(a) A body whose volume is 100 cm3 weighs 1 kgf in air. Find its weight in water.
6.
(b) Why is it easier to swim in sea water than in river water?
Solution :
(a) Volume of body = 100 cm3



= 1 × 10–4 m3

Body weighs 1 kgf in air

Upthrust = weight of water displaced by body

= Vrwg

= 10–4 × 1000 × g

= 0.1 kgf

Weight of body in water = Weight of body in air – Upthrust

= 1 kgf – 0.1 kgf
= 0.90 kgf
Answer (b) : Swimming in sea water is easier than river water, because of higher density sea water provides greater
upthrust as compared to river water.
7.
Why does a ship made of iron not sink in water, while an iron nail sinks in it?
Answer : The density of iron is more than density of water, so, it displaces less weight of water than its own weight
hence, it sinks in water. However ship made of iron is hollow from within. Thus it has fairly large volume.
This reduces its apparent density to such an extent, that it becomes less dense than that of water. Thus, it
is able to displace more weight and hence floats on the surface of water.
A solid of density 5000 kgm–3 weighs 0.5 kgf in air. It is completely immersed in a liquid of density
8.
800 kgm–3. Calculate the apparent weight of the solid in liquid.
Solution : Density of solid = 5000 kgm–3

Weight of solid = 0.5 kgf \ Mass of solid = 0.5 kg
0.5

Volume of the solid = = 1 × 10–4 m3
5000
Density of liquid = 800 kgm–3

82
Volume of liquid displaced = 1 × 10–4 m3


Weight of the liquid displaced = Volume of liquid × density of liquid

= 1 × 10–4 × 800 = 0.08 kg

\ Apparent weight of the solid in liquid = 0.5 – 0.08 = 0.42 kgf
9.
(a) A body dipped in a liquid experiences an upthrust. State the factors on which the upthrust depends.

(b) While floating, is the weight of body greater than, equal to or less than upthrust?
Answer :
(a) Factors on which upthrust depends
l The magnitude of upthrust increases with the increase in volume of the body immersed in the

fluid and hence with the increase in the fluid displaced by the body.
l Magnitude of upthrust increases, with the increase in density of the fluid. It means, more the

density of a fluid, more is upthrust and vice-versa.


(b) When the body is floating then weight of the body is equal to the upthrust acting on the body.
10.
A sinker is first weighed alone under water. It is then tied to a cork and again weighed under water. In
which of the two cases weight under water is less and why?
Solution : Weight of sinker when tied to a cork, under water is less than that when it is alone weight under water.
Because cork displaces more water than its own weight i.e., weight of cork and because of which greater
upthrust acts on the sinker.
A solid weighs 105 kgf in air. When completely immersed in water, it displaces 30,000 cm3 of water,
11.
calculate relative density of solid.
Solution : Volume of water displaced (V) = 30,000 cm3 = 0.03 m3

Weight of solid in air = 105 kgf
Weight of water displaced = Vrwg
= 0.03 × 1000 × g = 30 kgf
Weight of solid in air
Relative density of solid =
Weight of water displaced by solid
105
= = 3.5
30

12. A test tube loaded with lead shots weighs 25 gf and floats upto the mark X in water. When the test tube
is made to float in brine solution, it needs 5 gf more of lead shots to float upto level X. Find the relative
density of brine solution.
Solution : Weight of test tube floating in water = 25 gf

By the law of floatation,

Weight of water displaced = Weight of test tube = 25 gf

When test tube is placed in brine solution, it needs 5 gf more of lead shots to float upto same level as in
water.

\ Weight of test tube = 25 + 5 = 30 gf

Since volume of brine solution displaced = volume of water displaced.

So, weight of test tube = 30 gf
Weight of brine solution displaced

R.D. of brine solution =
Weight of water displaceed
30
= = 1.2

25

83
13. A wooden block is weighed with iron, such that combination just floats in water at room temperature. State
your observations when :
(i) water is heated above room temperature
(ii) water is cooled below 4° C. Give reasons to your answers in (i) and (ii).
Solution : (i) We know that the density of water decreases with rise in temperature and hence on heating water
above room temperature upthrust decreases. As a result wooden block weighed with iron, sink more
than earlier.
(ii) Density of water is maximum at 4 °C. When water is cooled below 4°C, then its density decreases
hence upthrust acting on it decreases. So when water is cooled below 4°C wooden block weighed
with iron, sinks more than earlier.
14.
A rubber ball floats in water with 2/7 of its volume above the surface of water. Calculate the average relative
density of rubber ball.
Solution :

Let the volume of rubber ball = V
2
Volume of rubber ball above the water surface =
V
7
2 5V
Volume of the rubber ball below the water surface = V −
V =
7 7

By the law of floatation

Volume of rubber ball × density of rubber ball = Volume of water displaced × density of water
5
⇒ V × ρ= V × ρω
7
where, ρ → density of rubber ball
ρ w → 1 gcm −3
5
or, V×ρ = V ×1
7
5
ρ = = 0.71 gcm −3
7
Density of rubber ball

Average density of rubber ball =
Density of water

ρ 0.71
= = = 0.71
ρω 1


15. A cube of ice whose side is 4.0 cm is allowed to melt. The volume of water formed is found to be
58.24 cm3. Find the density of ice.
Solution :
Side of ice cube = 4.0 cm
\ Volume of ice cube = (4.0 cm)2
= 64 cm3
Volume of water = 58.24 cm3
Density of ice = rice = ?
Density of water = 1 gcm–3
Mass of water having volume 58.24 cm3 = 58.24 × 1 = 58.24 g
Hence, mass of ice cube = 58.24 g

84
Mass of ice 58.24
Density of ice = = = 0.91 gcm −3
Volume of ice 64
16. A jeweller claims to make ornaments of pure gold of relative density 19.3. A customer buys from him a
bangle of weight 25.25 gf. The customer then weighs the bangle under water and finds its weight 23.075
g with the help of suitable calculations explain whether the bangle is of pure gold or not.
Solution :
Relative density of pure gold = 19.3 cm
Weight of bangle in air = 25.25 g
Weight of bangle in water = 23.075 g
Weight of bangle in air
Relative density of bangle =
Weight of bangle in air − Wt. of baangle in water

25.25 25.25
= = = 11.61
25.25 − 23.075 2.175
\ Relative density of gold bangle = 11.61
Which is different from relative density of pure gold 19.3
Hence the bangle is not pure gold.
17. (a) When a piece of ice floating in water melts, the level of water inside the glass remains same. Explain.
(b) An inflated balloon is placed inside a big glass jar which is connected to an evacuating pump. What
will you observe when the evacuating pump starts working? Give a reason for your answer.
Answer : (a) A piece of ice floating in water displaces an amount of water equal to its own weight. Amount of
water displaced is equal to volume of submerged portion of ice piece. When ice piece melts, its vol-
ume decreases and gets occupied in that volume of water which is displaced by it. As a result, level
of water inside the glass remains same when piece of ice melts.
(b) When the evacuating pump starts working pressure inside the glass jar reduces. Pressure inside balloon
is more than the pressure outside the balloon. This imbalance in pressure results in bursting of balloon.
18. (a) A trawler is fully loaded in sea water to maximum capacity. What will happen to this trawler, if moved
to river water? Explain your answer.
(b) A body of mass 50 g is floating in water. What is the apparent weight of body in water? Explain your
answer.
Answer : (a) The density of sea water is higher than the river water. Thus, when a fully loaded trawler in a sea water
of higher density sail into river water of lower density, it is likely to sink. It is because according to
the law of floatation, a greater volume of water is required to be displaced, so that its weight is equal
to the weight of the ship.
(b) Apparent weight of body = Weight of body in air – Weight of liquid displaces by body
Weight of body = 50 gf, when a body floats in water, the weight of body in air is equal to the weight of
water displaced by the body
Thus weight of displaced water = 50 gf
\ Apparent weight = 50 gf – 50 gf = 0
19. A body of mass ‘m’ is floating in a liquid of density ‘r’.
(i) what is the apparent weight of body?
(ii) what is the loss of weight of body?
Answer :
Mass of the body = m
Density of the liquid = r

85
(i) Apparent of weight of a body = Weight of body in air – Weight of liquid displaces by body

When a body floats in the liquid, then weight of the body in a liquid is equal to weight of liquid
displaced by the body.

\ The apparent weight of body = 0
(ii) Loss in weight of body is equal to the weight of liquid displaced by the body or equal to its own weight.
So, it experiences weightlessness.
20. A block of wood of volume 25 cm3 floats in water with 20 cm3 of its volume immersed in water.

Calculate: (i) density of wood (ii) the weight of block of wood.
Solution : Volume of wooden block (V) = 25 cm3
Volume of wooden block immersed in water = 20 cm3
Volume of water displaced by wooden block = Volume of wooden block immersed in water = 20 cm3
Density of water = rwater = 1 g cm–3
Density of wooden block = rwood = ?
(i) By law of floatation,
Volume of wooden block × Density of wood = Volumeof water displaced × Density of water
⇒ 25 × rwood = 20 × 1
20
\   rwood = = 0.8 gcm–3
25
(ii) Weight of wooden block = Vwood × rwood × g
= 25 × 0.8 g = 20 gf
21. A solid body weighs 2.10 N in air. Its relative density is 8.4. How much will the body weigh if placed
(i) in water, (ii) in liquid of relative density 1.2?
Solution :

(i) Weight of solid body = 2.10 N
Relative density of solid = 8.4
Weight of solid in air
Relative density of solid =
Weight of water displaced by body

2.10
⇒ 8.4 =
Weight of water displaced by body

or, Weight of water displaced by body 2.10


= = 0.25 N
8.4
\ Weight of body in water = Weight of body in air – Weight of water displaced by body
= 2.10 – 0.25 = 1.85 N

(ii) Upthrust due to water = Weight of water displaced by the body
= 0.25 N
Upthrust due to liquid = Upthrust due to water × Relative density of liquid
= 0.25 × 1.2 = 0.30 N
\ Weight of body in liquid = Weight of body in air – Upthrust due to liquid
= 2.10 – 0.30 = 1.80 N

EEEEE

86
6 Heat and Energy

EXERCISE
1. Define heat energy (a) on conventional basis (b) on the basis of kinetic theory of matter.
Answer :
(a) Heat is a form of energy which causes in us the sensation of hotness or coldness.
(b) On the basis of kinetic model of matter, we can say that heat energy is the sum total of the kinetic and
potential energies of all the molecules of a given substance.
2. Define temperature (a) on conventional basis (b) on the basis of kinetic theory of matter.
Answer :
(a) The degree of hotness or coldness of a body is called temperature.
(b) On the basis of kinetic theory of matter, temperature is the average kinetic energy of all the molecules of
a substance.
3. State three differences between heat and temperature.
Answer :
Heat Temperature
1.
Heat energy is the sum of the potential and 1. Temperature is the measure which indicates the
kinetic energies of all the molecules of a body. ability of a body to give up heat to another body
or absorb heat from another body.
2.
Its S.I. Unit is joule or J. 2.
Its S.I. Unit is kelvin or K.
3.
It flows from not body to cold body. 3.
It determiners the direction of flow of heat when
two bodies at different temperature are placed in
contact.
4.
Define calorie. How many joules make one calorie?
Answer :

One calorie is the amount of heat energy required to raise the temperature of 1 g of water through 1°C.

Calorie and joule are related as follows :

1 cal = 4.186 J = 4.2 J.
5.
A body absorbs 1680 J of heat energy. How many calories of heat is absorbed by the body?
Solution :

Heat energy absorbed by body = 1680 J

We know 4.2 J = 1 cal
1680

Or cal. 1680 J =
4.2

= 400 cal.
6.
A body radiates out 300 calories of heat energy. How much heat energy is radiated out by the body in joules?
Solution : Heat energy radiated by the body = 300 cal
We know that 1 cal = 4.2 J
\ 300 cal = 300 × 4.2 J
= 1260 J
87
7.
What do you understand by the anomalous expansion of water?
Answer :

Liquids generally expand on heating and contract on cooling. However, water is a liquid which does not behave
like the normal liquids.

Considering a flask containing 1000 g of water at 0°C. The flask is fitted with very fine capilary tube and highly
sensitive thermometer. It is found that volume of water recorded by capillary tube is 1000.14 cm3.

If the flask is allowed to warm up, it is found that volume of water start decreasing up to 4°C and its volume at
4°C is 1000 cm3. On further increase in temperature the volume of water increases.

The expansion of water from 4ºC to 0°C is called anomalous expansion of water.
8.
How do fishes survive in frozen lakes?
Answer :

In cold countries the temperature of air falls below 0 °C, the water bodies like lakes freeze from the top to form
solid ice, whereas the water below stays at 4 °C.

As the temperature of water drops, it contracts, its density increases and hence, it sinks to the bottom of lake. This
continues till 4 °C. However, as temperature drops below 4 °C its density decreases on account of its anomalous
expansion of water. Hence upper layer of water stays up. This continues till temperature drops to 0°C.

Finally, change of state takes place and it changes to form ice, which floats on the surface of water. The water
in the lake does not freeze completely, because ice formed on the top is a bad conductor of heat.

Thus, fishes survive in frozen lakes as water below is at 4 °C.
9.
Explain why soft drink bottles are not completely filled?
Answer :

Soft drink bottles are kept in the freezer of a refrigerator. On lowering temperature below 4 °C, water expands
due to its anamalous behaviour.

So, if soft drink bottles are completely filled, then they may burst and cause accident. So, soft drink bottles are
not completely filled.
10. Explain why do water pipes burst during severe winter?
Answer :

In severe winter, water pipe lines often burst, because water freezes at sub-zero temperature and in doing so
expands. Since there is no space within the pipes for expanding ice, it exerts tremendous pressure and bursts
open the steel pipes.
11. Explain, why does a glass bottle completely filled with water and tightly capped burst when placed in a freezer?
Answer :

A glass bottle completely filled with water and tightly capped and placed in a freezer is burst because water
changes into ice and its volume increases. Thus, it exerts tremendous pressure and glass bottle bursts.
12. Explain why are the taps left dripping in sub-zero temperature during winter?
Answer :

Water freezes at sub-zero temperature and in doing so it expands. So taps are left dripping in sub-zero temperature
during winter. So as to provide sufficient space for expansion for freezing water.
13. Explain, why are the exposed water pipes lagged with straw during severe winter?
Answer :

Water can freeze in pipes during severe winter and can cause bursting of pipes. So it exposed water pipes are
logged with straw, then coating of straw acts as bad conductor of heat and minimise the effect of severe cold on
the water pipes.

88
14. Explain why do vegetables and fruits get damaged during severe winter?
Answer :

Vegetables and fruits get damaged during severe winter, because the water present in the vegetables and fruits on
freezing expands and bursts open the cell walls.
15. At what temperature the pure water has (a) maximum density (b) minimum volume.
Answer :
(a) At 4 °C, pure water has maximum density.
(b) At 4 °C, pure water has minimum volume.
16. A deep pond of water has its top layer frozen what will be the likely temperature of water layer (i) Just in contact
with ice (ii) at the bottom of pond?
Answer :
(i) Temperature of water layer Just in contact with ice is 0°C.
(ii) At the bottom of pond is 4°C.
17. What is greenhouse effect?
Answer :

There are some gases such as carbon dioxide which prevent the escape of heat from the earth. An increase in
the percentage of such gases in the atmosphere would increase the average temperature worldwide, called the
greenhouse effect.
18. Which gas is chiefly responsible for global warning?
Answer :

Carbon dioxide gas is chiefly responsible for global warming.
19. How can global warming can be reduced?
Answer :

To reduce global warming following measures can be taken.
(i) Internal combustion engines in vehicles should be replaced by electric vehicles, so that emission of carbon
dioxide can be reduced.
(ii) Reforestation and sustainable used of land should be encouraged. Forest habitats should be maintained.
(iii) Controlling population, through family planning, welfare reform and empowerment of woman. This will
help in reducing population, which in turn consumption for energy and hence will reduce global warming.
20. State four effects of global warming.
Ans. The effects of global warming on the life on earth is as below :
(i) Dislocation and disappearance of plant and animal species.
(ii) Warming of ocean : The temperature of ocean water will increase due to which many species in the ocean
will either die or they will disappear, while various other species which prefer warm water which increased
tremendously.
(iii) Melting of glacier : Due to global warming, the frozen tundra in siberia has started melting.
(iv) Shift in farming region : In the near future warming of nearly 3°C will result in poor yield in the farm of
low latitude regions. The crop yield is expected to increase in middle and high latitude regions. Therefore,
farmers will have to shift their farming region from low to high altitude.

EEEEE

89
Energy Flow and Practices for Conservation
7
of Resources
EXERCISE
1. Name two renewable and the two non-renewable sources of energy.
Answer :
Two renewable sources : Solar energy and wind energy.
Two non-renewable sources : Coal and petroleum.
2. Why should we conserve the non-renewable sources of energy.
Answer :

The formation of fossil (non-renewable sources) fuels have occurred over millions of years due to certain very
slow changes under special circumstances. If these sources once finished cannot be regenerated over a reasonable
period of time. So, we should conserve the non-renewable sources of energy.
3. State the law of conservation of energy.
Answer :

Law of conservation of energy states that energy is neither created, nor destroyed in a system and sum total
energy remains same.
4.
State the main difference between the renewable and non-renewable sources of energy.
Answer :
Renewable sources Non-renewable sources
(i) These are the sources from which energy can be (i) These are the sources from which energy
obtained continuously over a long period of time. cannot be continuously obtained over a long
(ii) They are non-conventional sources. period of time.
(iii) These are the natural sources which will never get (ii) They are non-conventional sources.
exhausted. (iii) These are natural sources which would soon
(iv) These sources can be regenerated. depletes.
(v) Solar energy, wind energy, hydro-energy and geo- (iv) These sources cannot be regenerated.
thermal energy are the examples of renewable (v) Examples of non-renewable sources are coal,
sources of energy. petroleum and natural gas.

5.
Explain why it is not wise to regard wood as a renewable sources of energy?
Answer :

It is not wise to regard wood as a renewable sources of energy because a tree gets mature in 15-30 years. If these
trees are cut for wood it takes for more time to grow.
6.
Why fossil fuels called non-renewable sources of energy?
Solution :

Fossil fuels like coal, petroleum and natural gas are exhaustible i.e., if these are finished can not be regenerated
over a reasonable period of time. That is why fossil fuels are called non-renewable sources of energy.
7.
State two limitations of solar energy.
Answer :
Limitations of solar energy:
1. The solar energy reaches the surface of the Earth in diffused form.
2. Solar energy is not available uniformly all the time and at all the places of the Earth.
90
8.
State four traditional uses of solar energy.
Answer :
Four traditional uses of solar energy :
(i) Solar energy is used in solar cookers for cooking of food.
(ii) Solar energy is used in drying wet clothes.
(iii) Solar energy is used in obtaining salt from sea water.
(iv) It is also used in preservation of fish, meat and vegetables.
9.
State three ways by which you can enhance the collection of solar energy.
Answer :

Following are the three ways by which we can enhance the collection of solar energy.
(i) Uses of black painted surface : Black surfaces are good absorbers as well as good radiators of radiant
heat. Thus, the surface of the objects which are to collect heat are painted in black.
(ii) Use of insulated box with glass cover : The object painted in black, should be placed in well insulated
box provided with glass cover. The insulated box will prevent heat losses due to conduction, convection
and radiation. The glass cover will allow the solar energy to pass through it as it is transparent to radiant
heat of smaller wavelength.
However, when the object with blackened
surface gets hot within the box and
radiates out radiant heat, the glass will
not allow these radiations to pass through,
as they are of longer wavelength. Thus,
the heat radiations trapped within the
insulated box.
Used reflectors : The amount of solar
(iii)
energy per square metre per second is
too small to do any useful work. Thus, if
solar energy is directly allowed to enter a
solar heating device, it does not raise its
temperature sufficiently. So, solar energy
is collected from a sufficiently large area
and then reflected into a small area of
a solar heating device, its temperature
raises sufficiently to do some useful
work. Thus, solar reflector are useful in
solar heating devices.
10. State two advantages of box type solar cooker.
Answer :
Two advantages of box type solar cooker.
(i) It costs nothing for cooking the food.
(ii) They do not cause any pollution in the environment.
11. What is the range of temperature which can be achieved in it?
Answer :

The temperature within solar cooker can be achieved is about 140°C.
12. (a) What are semiconductors? Name two semiconductors.
(b) What are doped semiconductors? Name two materials used for dopping semiconductors. What is the
advantage of doping the semiconductors?

91
Answer :
(a) Those materials which are neither good conductors nor insulators of electricity are called semiconductors.
Germanium (Ge) and silicon (Si) are two examples of semiconductors.
(b) The process of adding impurities to a semiconductors is called dopping.
Two materials used for dopping are Boron and Phosphorus.
Advantages of dopping : On dopping the conductivity of semiconductors increases.
13. Name a device which directly converts solar energy to electric energy.
Answer :
Solar cells converts directly solar energy to electric energy.
14. What is a solar panel? To what uses are the solar panels put?
Answer :
The group of solar cells connected in specific pattern to produce desired potential difference and magnitude of
current is called solar panel.
The potential difference produced by a single solar cell of 4 cm2 is between 0.4 to 0.5 volts and magnitude of
electric current is 60 milliamperes (0.06Å). However, when a large number of solar cells are connected in a mixed
circuit to form a solar panel, a large potential difference and a large current can be obtained.
15. What is a biogas? Which component of biogas is used as fuel? Describe a fixed dome type biogas plant with the
help of a labelled diagram.
Answer :
Biogas is a mixture of gases formed when the slurry of animal dung and water is allowed to ferment in the
absence of oxygen or air. The biogas is a mixture of methane, carbon dioxide, hydrogen and traces of hydrogen
sulphide along with water vapour. The main component of biogas is methane gas which is about 65% by volume.
Fixed dome type biogas plant : Fixed dome type biogas plant consists of following parts.
(i) Digester tank (ii) Slurry mixing tank and (iii) Spent slurry tank
Slurry of cattle Outlet for
dung and water bio-gas

S
Mixing Slab cover Slab cover
tank V Gas valve
M Dome

D
Ground Overflow
level Bio-gas tank
l F
Inlet
chamber
Spent slurry

T O
Dung and water
mixture Outlet
chamber

Underground
digester tank

Fixed Dome Type Biogas plant tank

92
Digester tank : It is an underground tank. Its base and walls are made of bricks and cement. Its roof is made in
the form of dome either with cement or bricks or with concrete.

Dome collect the biogas. It is provided with an outlet for biogas. The outflow of the biogas is controlled by gas
valve.
Slurry mixing tank : On the left side of the digester (Fig) is constructed a brick lined tank above the ground
level. The bottom of this tank connects the digester through a brick-lined channel near the base.
The fresh slurry of animal dung and water in equal proportions is poured into the slurry mixing tank. This slurry
slowly flows into the digester.
Spent Slurry tank : On the right side of the digester (Fig.) is constructed spent slurry tank, below the ground

level. The base of this tank is in line with the lowest point of the dome of the digester. The base of this tank is
connected to the base of the digester through a brick-lined channel.
Working of biogas plant : The slurry of animal dung and water in equal proportion is poured into the mixing

tank daily. This slurry flows into the digester. In the digester anaerobic fermentation starts with the release of
biogas which is collected in the dome. As more and more slurry flows into the digester the level of slurry rises
up. During this time the biogas collects in the dome under pressure.
When the pressure of the biogas exceeds certain limit, the biogas forces the spent slurry into the overflow tank.
The biogas withdrawn from the dome through outlet pipe. Its flow is regulates by a gas valve.
16. Name an agent which decomposes animal dung into biogas.
Answer : The anaerobic bacteria decomposes animal dung into biogas.
17. Write any two uses of biogas.
Answer : (i) Biogas is an excellent fuel which can be used for domestic cooking.
(ii) Biogas does not produce any smoke and hence, it helps in cutting down air pollution.
18. (a) Name the kind of energy possessed by the wind.
(b) State two advantages and two limitations of wind as source of energy.
Answer : (a) Kinetic energy is possessed by wind.
(b) Two advantages of wind energy :
(i) Wind energy is a natural resource and is available without any cost.
(ii) Wind energy does not cause any pollution.
Two limitations of wind energy.
(i) Fast moving wind is not available all the times, at a given place.
(ii) The wind mills and wind turbines remain unoperational when wind is not blowing. This adds up to operational
costs.
19. With the help of a diagram explain how wind energy is converted into electric energy.
Answer :

A windmill consists of two or three blades like an electric fan
which rotates about an axle mounted on a pole. The shaft of the
windmill is connected to the shaft of armature of a generator.
When the fast moving wind rotates the blades of windmill, its
shaft rotates with armature inside the generator. The rotating
armature produces electric current. Thus, the kinetic energy of
the wind is converted into electric energy.
20. (a) What do you understand by the term hydroelectric power?
(b) Explain the energy changes taking place in a hydroelectric
dam.
Answer : (a) Hydroelectric power is the power derived from energy
of falling water or fast running water. A hydroelectric

93
power plant is an arrangement in which the kinetic energy of flowing water is transformed into electric
energy and the electric energy so generated is called hydroelectric energy.
(b) Energy changes taking place in hydroelectric dam
(i) As the water flows into the reservoir from the catchement area, the kinetic energy of flowing
water is converted to potential energy.
(ii) As the water is released through control valve, the potential energy of water changes into
kinetic energy.
(iii) As the water flowing at a high speed strikes the blades of the turbine its kinetic energy
changes to rotational kinetic energy of the turbine.
(iv) As the turbine is coupled to the shaft of generator, the rotational kinetic energy of turbine
changes to the rotational kinetic energy of armature.
(v) As the coil of armature rotate in magnetic field, the rotational kinetic energy of armature
changes to the electric energy in its coils.
21. State two advantages and two limitations of Hydel Power.
Answer :
Advantages of Hydel Power
(i) The water required for generating hydel power is available free of cost.
(ii) Hydel power stations are pollution free.
Limitations of Hydel Power
(i) The flowing water which is required for hydel power is not available everywhere.
(ii) For construction of dams massive investment is required.
22. Give four reasons, why the use of cowdung in biogas preferred to burning of cowdung cakes.
Answer :
Use of cowdung in biogas is preferred to burning of cowdung cakes because :
(i) Biogas does not produce any smoke and hence cause no pollution.
(ii) The biogas leaves behind no ash on buring.
(iii) Calorific value of biogas is very high as compared to cow-dung cakes.
(iv) Biogas can be used for illumination purpose which is not possible in the case of cowdung cakes.
23. Why is charcoal is considered better fuel than wood?
Answer :
Charcoal is considered a better fuel than wood because of the following reasons :
(i) Calorific value of charcoal is higher than the caloric value of wood.
(ii) Charcoal produces no smoke.
(iii) Charcoal burns easily without any flame.
24. Name two forms of sea energy.
Answer :
Two forms of sea energy are :
(i) Tidal energy
(ii) Wave energy
25. By which name heat energy in the interior of earth is known?
Answer :
The heat energy in the interior of earth is known as geothermal energy.
26. Name the major constituent of natural gas.

94
Answer :
Methane is the major constituent of natural gas.
27. Name the nuclear process that responsible for
(i) explosion of an atom bomb.
(ii) release of energy by sun.
Answer :
(i) Nuclear process that is responsible for explosion of an atom bomb is nuclear fission.
(ii) Nuclear fusion is responsible for release of energy by the sun.
28. Define :
(i) Nuclear fission
(ii) Nuclear fusion
Answer :
Nuclear fission : The process by which a heavy unstable nucleus (such as U-235) is broken down into
(i)
two medium-weight nuclei by the bombardment of a slow neutron, so as to liberate more neutrons and
tremendous amount of energy is called nuclear fission.
(ii) Nuclear fusion : The process in which two or more light nuclei combine together to form a heavy nucleus
along with the release of very large amount of energy is called nuclear fusion.
29. Describe the condition for (i) nuclear fission (ii) nuclear fusion
Answer :
(i) Condition for nuclear fission : Bombardment of slow neutrons on heavy unstable nucleus like U-235.
(ii) Condition for nuclear fusion : A very high temperature of the order of millions of degree Celsius required
for nuclear fusion.
30. Explain the difference between nuclear fission and nuclear fusion.
Answer : Difference between nuclear fission and nuclear fusion are :
Nuclear fission Nuclear fusion
(i) In a nuclear fission, a heavy nucleus splits up into (i) In a nuclear fusion, two or more light nuclei
lighter nuclei. combine to form a heavy nucleus.
(ii) Harmful radiation are emitted in nuclear fission. (ii) No harmful radiations are emitted in nuclear
(iii) It starts when slow neutrons bombards the heavy fusion.
nucleus like U-235. (iii) Nuclear fusion states when light nuclei are
(iv) It causes much pollution. heated at very high temperature.
(v) A large amount of energy is released in nuclear (iv) It causes no pollution.
fission. (v) Energy released in nuclear fusion reaction is
(vi) It can be controlled. much higher than nuclear fission.
(vi) Nuclear fusion reaction cannot be controlled.
31. Explain how geothermal energy is used in generating electricity.
Answer :
The places under the surfaces of Earth, where the hot magma collects fairly less depth, are called hot spots of
the Earth. The hot spots are the source of geothermal energy which is used to generate electricity as follows.
(i) In some geological regions of the earth, the underground water comes in contact with hot spots and changes
into steam. The steam so formed get trapped between the underground rocks and get compressed to very
high pressure. This steam is extracted from the ground by sinking pipes from the surface of the earth.
The steam coming up at high pressure is used in running turbines connected to the generator. This in turn
produces electric energy.
(ii) In some places, the steam formed under the surface of the earth does not get trapped. Instead it forces its

95
way up through the cracks in the rocks along with hot water and gushes out from the surface of the earth
to form natural geysers. The heat energy brought up by the natural geysers is used in running turbines
connected to the generator. This in turn produces electric energy.
(iii) In some geological region underground water does not come in contact with hot spots. In such regions, two
holes are made in the Earth’s crust. Through one hole cold water is pumped. The cold water on coming in
contact with hot spot changes to super heated steam, which emerges out from the outer hole. The steam
then made to run a turbine coupled with generator to produce electricity.
32. What are hot spots? How can you extract energy from hot spot, if it does not come in contact with under ground
water?
Answer :
The places under the surface of Earth, where the hot magma collects at fairly less depths are called hot spots of
the Earth. If under ground water does not come in contact with underground water, then to extract energy from
such region, two holes are made in the Earth’s crust. Through one hole cold water is pumped in. The cold water
on coming in contact with hot spot changes to superheated steam, which emerges out from the other hole. The
steam in then made to run a turbine coupled with generator to produce electricity.
33. Explain why nuclear fusion is not being used to meet day to day energy needs.
Answer :
To initiate nuclear fusion reaction, extremely high temperature and pressure requires. Such an extremely high
temperature and pressure cannot be achieved daily life in a controlled way. So, nuclear fusion is not being used
to meet day to day energy needs.
34. What is nuclear waste? What are hazard of nuclear waste to living beings? How is nuclear waste disposed off?
Answer :
Nuclear waste is the material that nuclear fuel becomes after it is used in a reactor.
Hazards of nuclear waste

There is always a risk of accidental radioactive leak at nuclear power plants.

It is very costly to set up nuclear reactors, hence cost of electricity generated is higher.

Nuclear wast is the source of radiations which can lead to the following disorders
(i) Radiations can cause genetic disorders.
(ii) Nuclear waste can cause environmental contamination which can affect the health of millions of people.
(iii) Long exposure to radiations can cause cancer, blindness etc.
Disposal of nuclear waste : The nuclear waste is packed in thick lead lined, steel containers and kept at safe
place, far away from the human habitation. This is done so as to minimise the environmental pollution as far as
possible. It is a very expensive process, but has to be followed as there is no alternative process available for the
safe disposal of nuclear waste.
35. What are the environmental consequences of using fossile fuels?
Answer :
Environmental consequences of using fossil fuels are :
(i) Fossil fuel burning causes air pollution.
(ii) The burning of fossil fuel can cause acid rain which corrode the buildings, monuments and reduces the
fertility of the soil.
(iii) Smog is caused by burning of fossil fuels which blocks the sun’s radiations and which hamper the process
of photosynthesis.
(iv) Burning of fossil fuels release greenhouse gases, which inturn be the source of global warming.
36. Energy from various sources is considered to have been derived from the sun. Do you agree? Justify your answer
by giving two examples.

96
Answer :
Yes, Sun is the ultimate source of energy directly or indirectly all the forms of energy derived from solar energy.
(i) Fossil fuel like coal, petroleum and natural gas are formed due to burial of large plants and ancient creatures
whose ultimate source of energy is sun.
(ii) Renewable sources of energy : They are indirectly derived from the solar energy such as
(a) Wind energy : Wind energy arises due to uneven heating of earth’s surface by the sunrays at two different
adjoining places. Due to the wind possesses kinetic energy.
(b) Wave energy : The waves are generated by strong wind (due to solar energy) blowing across sea.
(c) Bio energy : Plants in the process of photosynthesis convert the solar energy into food (chemical energy).
This food is consumed by animals.
(d) Solar devices : They derive energy directly from the solar energy and convert it into other usable forms of
energy.
(e) Energy of flowing waters is also due to solar energy.

EEEEE

97
8 Light

UNIT I
Reflection at Plane Surfaces

EXERCISE 1
1. (a) What do you understand by the following terms?
(i) Light (ii) Diffused light
(b) By giving one example and one use explain or define (i) Regular reflection (ii) Irregular reflection
Answer :
(a) (i) Light is a form of energy which causes in us sensation of vision.
(ii) Light obtained after reflection from rough surface is known as diffused light. It is a soft light with
neither have the high intensity nor the glare of direct light.
(b) (i) The phenomenon due to which a parallel beam of light travelling through a certain medium, on striking
some smooth polished surface, bounces off from it, as parallel beam, in some other direction is called
regular reflection. Regular reflection is useful in the formation of images. We can see our faces in a
mirror only due to regular reflection.
(ii) Irregular reflection : The phenomenon due to which a parallel beam of light, travelling through some
direction, get reflected in various possible directions, on striking some rough surface is called irregular
reflection. Reflection taking place from ground, walls, trees, suspended particles in air are irregular
reflection.
Irregular reflection helps in the general illumination of places and helps us to see things around us.
2. By drawing a net diagram define the following :

(i) Mirror (ii) Incident ray (iii) Reflected ray (iv) Angle of incidence  (v) Angle of reflection (vi) Normal
Answer :
Mirror : A smooth polished surface from which regular reflection can take place is called mirror.
(i)

Incident ray : A ray of light striking the reflecting surface is called


(ii)
the incident ray.

Reflected ray : The light ray obtained after reflection from the
(iii)
surface, in the same medium in which the incident ray is travelling
is called the reflected ray.
Angle of incidence : The angle which the incident ray makes with
(iv)
the normal at the point of incidence is called the angle of incidence.
It is generally denoted by letter i.

98
Angle of reflection : The angle of which the reflected ray makes with the normal at point of incidence, is
(v)
called the angle of reflection. It is generally denoted by the letter r.
(vi) Normal : The perpendicular drawn at the point of incidence, to the surface of reflecting surface is called
normal.
3.
State the laws of reflection.
Answer :
Laws of reflection :
(i) The incident ray, the reflected ray and the normal lie in the same plane at the point of incidence.
(ii) The angle of incidence is always equal to the angle of reflection.
4.
A ray of light strike a plane mirror, such that angle with the mirror is
20°. What is value of angle of reflection? What is the angle between
the incident ray and the reflected ray?
Answer :

Since light ray makes an angle of 20° with the mirror

\ Ð AOM = 20°

Angle of incidence = Ði = 90° – 20°

Ði = 70°

 Ði = Ðr \ Ðr = 70°

\ Angle between incident ray and reflected ray = Ði + Ðr = 70° + 70° = 140°
5.
Prove geometrically that when plane mirror turns through a certain angle, the reflected ray turns through twice
the angle.
Answer :

A ray of light AB, incident on plane mirror in position MM.
BN is the normal and BC is the reflected ray.

ÐABN = ÐCBN = Ði

ÐABC = 2Ði …(i)
Now, the mirror MM rotated through an angle q about point
B. New position of mirror is M1M1. BN1 is the new position
of normal. Since the position of the incident ray remains same.
So the new angle of incidence is ÐABN1 = Ð(i + q)
Let BD be the reflected ray, such that ÐDBN1 is the new
angle of reflection.
ÐABD = ÐABN1 + ÐBDN1
= Ð(i + q) + Ð(i + q)
= Ði + q) + Ð(i + q)
\ ÐABD = 2Ði + 2Ðq …(ii)

Subtracting (i) from (ii)
ÐABD – ÐABC = 2Ði + 2Ðq – 2Ði
\ ÐCBD = 2Ðq
Thus, for a given incident ray, if the plane mirror is rotated through a certain angle, then the reflected ray rotates
through twice the angle.
6. (a) What do you understand by the term lateral inversion?

99
(b) A printed card has letters PHYSICS. By drawing the
diagram show the appearance of the letters. (No ray
diagram is required)
Answer :
(a) The phenomenon due to which the image of an object
turns through an angle of 180° through vertical axis
rather than horizontal axis, such that right side of
images appears as left or vice versa is called lateral
inversion.

(b)

7. (a) State the mirror formula for the formation of total number of images formed in two plane mirror, held at
an angle.
(b) Calculate the number of images formed in two plane mirrors, when they are held at the angle of (i) 72°
(ii) 36º
Answer :
(a) If q be the angle of inclination between two mirrors and ‘n’ the number of images formed then
360º
n= −1
θ
(b) (i) When q = 72º
Number of images formed
360º
n= −1
72º
= 5 −1 = 4
(ii) When q = 36°
Number of images formed
360º
n= −1
36º
= 10 − 1 = 9
8. (a) State four characteristics of the image formed in a plane mirror.
(b) State three ways in which the image formed in a plane mirror differs from the image formed in a pin hole
camera.
Answer :
(a) Four characteristics of the image formed in a plane mirror.
(i) Image is virtual and erect.
(ii) Image is of the same size as the object.
(iii) Image is formed as far behind the mirror, as the object is in front of it.
(iv) Image is laterally inverted.
(b)
Characteristics of image formed by a plane mirror Characteristics of image formed by a pin hole camera
(i) Image formed by plane mirror is virtual Image formed by pin hole camera is real
(ii) Image cannot be obtained on the screen. Image can be obtained on the screen.
(iii) Size of the image is same as that of the object. The size of the image is smaller than the object.

100
9. (a) What should be the minimum size of a plane mirror, so that a person 182 cm high can see himself
completely?
(b) A boy stands 4 m away from plane mirror. If the boy moves 1/2 m towards mirror, what is now the distance
between the boy and his image? Give a reason for your answer.
Answer :
(a) Height of the person = 182 cm
We know that in order to sea full length, a person requires a plane mirror which is half of his own height.
So, size of the plane mirror required = Height of person
2
182
= cm
2
= 91 cm

1
(b) Distance of boy from plane mirror = 4 m; When the body moves m towards mirror.
2
1 7
Then, the distance between body and mirror = 4 – = m.
2 2
We know, in a plane mirror, the image formed is as behind mirror as the object is in front of it. So, the
7
distance between the image of the boy and mirror = m.
2
Distance between the boy and his image = Distance of boy from mirror + Distance of image from the mirror

7 7
=  + =7m
2 2

10. State four uses of a plane mirror.


Answer :

Four uses of a plane mirror are :
1. Plane mirrors are used as looking glass.
2. They are used in solar cookers for reflecting the rays of sun into the cooker.
3. Plane mirrors are used by opticians to provide false dimensions.
4. They are used in the construction of reflecting periscope.
11. (a) Draw a neat diagram of reflecting periscope.
(b) State two advantages and two disadvantages of the reflecting periscope.
Answer :
(a)

101
(b) Advantages of reflecting periscope
(i) It is used to see above the head of crowds.
(ii) It is used by soldiers in trench warfare.
D isadvantages of Reflecting Periscope
(i) The final image formed by reflecting periscope is not brightly illuminated as light energy is absorbed
due to two successive reflection.
(ii) Any deposition of moisture or dust on the mirror reduces the reflection almost to nil and hence, the
periscope cannot be used in a places where there is a lot of dust or moisture.
12. What must be the minimum length of a plane mirror in which a person can see himself full length? Draw a
diagram to justify your answer. Does the distance of person from the mirror affect the above answer?
Answer :
Consider a person AB, such that A represents the highest point and B represents the lowest point. E is the fixed
eye level.
A Head C

The person will be able to see every part of his body if he can A1
see points A and B. Let MN be the minimum length of the mirror P i (
(
r M
fixed on the wall, such that the rays AM and BN, after reflection, Eye R

Image
reach eye of person, thereby forming an image A1B1. E

In DAEA1, CM is parallel to AE and C is the mid-point of AA1. Mirror

M is the mid point of A1E. r(
Q N

In the same way in DBEB1, ND is parallel to BE and D is the i (
mid-point of BB1
Wall

\ N is the mid-point of B1E

MN is parallel to A1B1 B
D B1
Feet

But A1B1 = AB
1

So, MN = AB
2

Thus, in order to see full length, a person requires a plane mirror which is half his own height.

This relation is true for any distance of object from plane mirror.
13. An insect is sitting in front of a plane mirror at a distance of one metre from it.
(a) Where is the image of insect formed?
(b) What is the distance between insect and its image?
(c) State any two characteristics of image formed in a plane mirror.
Answer :
(a) Distance of insect from plane mirror = u = 1 m.
We know that in case of plane mirror, distance of object from plane mirror is equal to distance of image
of the object from the plane mirror.
\ Distance of the image of insect from the plane mirror v = u = 1 m.
(b) Distance between image and insect = u + v = 1 + 1 = 2 m.
(c) Two characteristics of the image formed by a plane mirror.
(a) image formed is virtual.
(b) image formed is erect.
14. (a) Draw a diagram to show reflection of a ray of light using plane mirror. In the diagram label the incident
ray, the reflected ray, the normal, the angle of incidence and angle of reflection.
(b) State the laws of reflection.

102
Answer :
(a)
N
A

Normal
B = angle of incidence
y
ra

In
= angle of reflection

cid
ed
ct

en
fle

t
ra
Re

y
Plane mirror
M O M
Reflection at a plane surface
(b) Laws of reflection :
(i) The incident ray, the reflected ray and the normal lie in the same plane at the point of incidence.
(ii) The angle of incidence is always equal to the angle of reflection.
15. How many images will be formed when an object is placed between two parallel plane mirrors with their reflecting
surfaces facing each other? Why do more distant images appear fainter?
Answer :

Angle between two mirrors facing each other = q = 0°.
360°
No. of images formed n =
−1
θ
360°
= − 1 = ∞ (Infinity)



So, infinite number of images are formed when an object is placed between two parallel plane mirrors with their
reflecting surface facing each other.

More distant images appear fainter because after every successive reflection, some amount of light energy is
absorbed. Thus luminousity of images goes on decreasing and hence they appear fainter.
16. (a) Describe the principle of simple periscope through an outline ray diagram. Give one of its uses.
(b) Draw diagrams to show difference between regular and
irregular reflection.
Answer :
(a) A simple periscope consists of a cardboard or wooden tube,
bent twice at right angles and it is provided with two opening
as shown in the figure.
Two plane mirrors are fixed at the bends at 45° to the
framework, such that the mirrors face each other. The tube is
completely blackened from inside to avoid any reflection from
its sides.
The parallel rays coming from an object at higher plane, strike
the plane mirror at an angle 45° and hence reflected through
an angle of 45°.
These reflected rays, strike the second mirror at angle 45° and hence further reflected through an angle of
45°. These reflected rays on reaching the eye form the image on retina.
(b)

103
UNIT II : Light
Reflection at Curved Surfaces
Numerical Problems on Reflection by Spherical Mirrors
Practice Problem 1.
1. An object 3 cm high produces a real image 4·5 cm high, when placed at a distance of 20 cm from a concave
mirror. Calculate : (i) the position of image (ii) focal length of the concave mirror.
Solution :
(i) hi = – 4.5 cm, because image is real
ho = 3 cm
Distance of image = v

Object distance (u) = –20 cm
Focal length of concave mirror ( f ) = ?
hi v

Linear magnification = m = =−
ho u
−4.5 v
or, −
3 −20
                 or, v = − 30 cm
1 1 1
(ii) Now, using the formula, + =
u v f
1 1 1
We have, + =
−20 −30 f
1 −3 − 2 −5
or, = =
f 60 60
−60
f = = −12 cm
5

Practice Problems 2.
1.
An object 1·5 cm high when placed in front of a concave mirror, produces a virtual image 3 cm high. If the
object is placed at a distance of 6 cm from the pole of the mirror, calculate : (i) the position of the image;
(ii) the focal length of the mirror.
Solution :
(i) Distance of image from the mirror = v = ?
As the image is virtual, hi = 3 cm is positive
Object distance u is always negative
u = –6 cm
ho = 1.5 cm
hi v
Linear magnification, m = h = − u
o

+3 v
=− ⇒
1.5 −6
v = 12 cm
So, image is formed at a distance of 12 cm behind the concave mirror.

104
(ii) Focal length of the concave mirror = f = ?
Using mirror formula
1 1 1
+ =
u v f
1 1 1
⇒ + =
−6 12 f
1 −2 + 1 1
or, = =−
f 12 12
or, f = −12 cm

2.
A converging mirror, forms a three times magnified virtual image when an object is placed at a distance of 8 cm
from it. Calculate : (i) the position of the image (ii) the focal length of the mirror.
Solution :
(i) Let the size of the object = ho = x.
Size of the image = hi = 3x (it is positive because the image is virtual)
Distance of the object from the mirror = u = –8 cm
Distance of the image from the mirror = v = ?
Linear magnification, hi v
=−
h0 u
3x v
or, =−
x −8
or, v = 24 cm
Thus, image is formed 24 cm behind the mirror.

(ii) Using the formula :


1 1 1
+ =
u v f
1 1 1
or, + =
−8 24 f
1 1 1 −3 + 1 −2
or, =− + = =
f 8 24 24 24
f = −12 m
Thus, the focal length of the converging mirror = –12 cm

Practice Problems 3.
1.
An object 5 cm high forms a virtual image of 1.25 cm high, when placed in front of a convex mirror at a distance
of 24 cm. Calculate : (i) the position of the image, (ii) the focal length of the convex mirror.
Solution :
(i) Size of the object = ho = 5 cm
Size of the image = hi = +1.25 cm
Distance of the object from the mirror = u = –24 cm
Distance of the image from the mirror = v = ?

105
hi v
Linear magnification, =−
h0 u
1.25 v
Or, =−
5 −24
1.25 × 24
Or, v=
5
∴ v = 6 cm
Thus, image is formed at a distance of 6 cm behind the mirror.
(ii) Focal length of the convex mirror = f
1 1 1
Using the formula + =
u v f
1 1 1
We have, + =
−24 6 f
1 −1 + 4 3
Or, = =
f 24 24
∴ f = 8 cm

2.
An object forms a virtual image which is 1/8th of the size of the object. If the object is placed at a distance of
40 cm from the convex mirror, calculate : (i) the position of the image, (ii) the focal length of the convex
mirror.
Solution :
(i) Size of the object = h0 = x
x
Size of the image = hi = (Positive as the image is virtual)
8
Distance of object (u) = –40 cm
Distance of the image from the convex mirror = v = ?
h v
Magnification = m = i = −
h0 u
x
v
or, 8 = −
x −40
or, v = 8 = 5 cm
40

Thus, the image is formed at a distance of 5 cm behind the mirror.
(ii) Using the formula,
1 1 1
= +
f u v
1 1 1
or, = +
f −40 5
1 −1 + 8 7
or, = =
f 40 40
40
f = cm
7
=5.71 cm m
Thus, the focal length of the convex mirror = 5.71 cm
106
EXERCISE 2
(A) Objective Questions
Multiple Choice Questions.
Select the correct option:
1. A concave mirror is made by cutting a portion of a hollow glass sphere of radius 30 cm. The focal length of the
concave mirror is :
(a) 24 cm (b) 12 cm (c) 15 cm (d) 60 cm
2. A mirror forms a virtual image (diminished) of an object, whatever be the position of object :
(a) it must be a concave mirror (b) it must be a convex mirror
(c) it must be a plane mirror (d) it may be (b) or (c) or both
3. A ray of light is incident on a concave mirror. If it is parallel to principal axis, the reflected ray will :
(a) pass through its principal focus (b) pass through its centre of curvature
(c) pass through its pole (d) retraces its path
4. If an incident ray passes through the centre of curvature of a spherical mirror, the reflected ray will :
(a) pass through its pole (b) retraces its path
(c) pass through its focus (d) be parallel to principal axis
5. In case of concave mirror, the minimum distance between an object and its real image is :
(a) f (b) 2 f (c) 4 f (d) zero
6. Looking into a mirror one finds her image diminished, the mirror is :
(a) concave (b) convex (c) cylindrical (d) parabolic
7. Which mirror is used in periscope?
(a) Convex mirror (b) Concave mirror (c) Plane mirror (d) Parabolic mirror
Ans. 1. (c) 15 cm 2. (b) it must be a convex mirror 3. (a) pass through its principal focus
4. (b) retraces its path 5. (d) zero 6. (b) convex 7. (c) Plane mirror

(B) Subjective Questions


1.
Define the following terms :
(i) spherical mirror (ii) convex mirror (iii) concave mirror
Answer :
(i) Spherical mirror : A mirror which is made from a part of a hollow sphere is called spherical mirror.
(ii) Convex mirror : A mirror which is polished from the hollow side of a sphere, such that reflecting surface
is towards its bulging side, is called convex mirror.
(iii) Concave mirror : A mirror which is polished from the bulging side of a hollow sphere, such that the
reflecting side is towards its hollow side, is called concave mirror.
2.
Define the following terms in relation to concave mirror.
(i) Pole (ii) Centre of curvature (iii) Principal axis (iv) Principal focus
(v) Focal length (vi) Radius of curvature (vii) Aperture
Answer :
(i) Pole : The mid-point of a concave mirror is called pole. It is denoted by the letter P.
(ii) Centre of curvature : The centre of the sphere, of which the concave mirror is a part is called centre of
curvature. It is denoted by the letter C.
(iii) Principal axis : An imaginary line passing through the pole and the centre of curvature of a concave mirror
is called principle axis of that concave mirror.
(iv) Principal focus : It is a point on the principal axis, where a beam of light parallel to the principal axis,
after reflection from a concave mirror actually meets. It is denoted by letter F.
107
(v) Focal length : The linear distance between the pole and the principal focus of a concave mirror is called
its focal length. It is denoted by f.
(vi) Radius of curvature : The linear distance between the pole and the centre of curvature of a concave mirror
is called its radius of curvature. It is denoted by the letter R.
(vii) Aperture : The portion of a mirror from which the reflection of light actually takes place is called the
aperture of the mirror. It is also called linear aperture of the mirror. The aperture of a spherical mirror is
denoted by the diameter of its reflecting surface.
3. (a) What do you understand by the term real image?
(b) What type of mirror is used to obtain a real image?
(c) Does the mirror named by you form real image for all locations? Give reason for your answer.
(d) Is real image always inverted?
Answer :
(a) The image which can be obtained on a screen, is called a real image. It is formed when light rays after
reflection or refraction actually meet.
(b) Concave mirror is used to obtain a real image.
(c) No, concave mirror does not form real images for all locations.
The nature and size of the image depends on the position of object. For the object beyond focus, the image
is always real and inverted, whereas for the object situated between the focus and pole, the image is upright
and virtual.
(d) Yes, real image is always inverted.
4.
Copy the Fig. below. By taking two rays from point A, show the formation of image. State four characteristics
of the image.

M

P
C B

Answer :
A
B1 C P
F
B

A1


Characteristics of the image
(i) The image is real.
(ii) The image is inverted.
(iii) The image is enlarged.
(iv) The image is formed beyond the centre of curvature in front of the concave mirror.

108
5.
Copy the Fig. By taking two rays from point A, show the formation of image. State four characteristics of the
image.

Answer :
Four characteristics of the image
(i) The image is virtual
(ii) Image is erect
(iii) Image is diminished
(iv) Image is formed between the pole
and the principal focus, behind the
convex mirror.
6. Compare the characteristics of an image formed by a convex mirror and a concave mirror, when object is beyond
centre of curvature, but not at infinity.
Answer :
Characteristics of an image, when object is beyond centre of curvature but not at infinity.
Convex mirror Concave mirror
(i) Image formed is virtual. Image formed is real.
(ii) Image formed is erect. Image formed is inverted.
(iii) Image formed is diminished. Image formed is diminished.
(iv) Image formed between the pole and the Image is formed between C and F, in front
principle focus, behind the convex mirror. of the concave mirror.

7. (a) Why does a driver use a convex mirror as a rear view mirror?
(b) Illustrate your answer with the help of ray diagram.
Answer :
(a) Convex mirror is used as a rear view mirror in automobiles, because it can cover a very wide field behind
the driver and hence, enables to see him the traffic behind, without turning his head.
(b)

8. (a) What is a real image?


(b) What type of mirror is used to obtain a real image of an object?
(c) Does the mirror named by you above give real images for all locations of object?

109
Answer :
(a) The image which can be obtained on screen, is called a real image. It is formed when light rays after
reflection or refraction actually meet.
(b) Concave mirror is used to obtain the real image of an object.
(c) Concave mirror cannot give real image for all the locations of the object.
9.
In the Fig. below is shown a concave mirror. A is a point on the principal axis. If an object O is kept at A, image
is formed on A itself. Copy the diagram. Draw the image in the diagram. Is the image real or virtual?


Measure the distance PA and write it in the diagram. What is the distance PA called?


Mark a point B on the principal axis, at which, if a point source of light is kept, the rays travel parallel to principal
axis after reflection from M. What is point B called?
Answer :


As the object is placed at A and its image is also formed at A, so object must be at centre of curvature (C).

PA is called radius of curvature and on measuring PA = 4.8 cm. Point B marked on principal axis is called
principal focus.

EEEEE

110
9 Sound

Numerical Problems

Practice Problems 1.
1.
200 waves pass through a point in one second. Calculate the time period of wave.
Solution :

No. of waves passing through a point in 1 s is 200.

\ Frequency of wave = 200 Hz
1 1
T == = 0.005 s

f 200
2.
A bat emits an ultrasonic sound of frequency 0.25 MHz. Calculate the time in which one vibration is completed.
Solution :
Frequency (f ) = 0.25 MHz = 0.25 × 106 Hz

1 1

Time period (T) = = s = 4 × 10−6 s
f 0.25 × 106

Practice Problems 2.
1.
The sonic boom of an aircraft has a time period of 0.00005 s. Calculate the frequency of sound produced.
Solution :
Time period (T) = 0.00005 s
1

Frequency of sound (f) =
T
1
= Hz
0.00005
= 20000 Hz
An electromagnetic wave has a time period of 4 × 10–8 s. Calculate its frequency in MHz.
2.
Solution :
Time period electromagnetic wave = 4 × 10–8 s

1

Frequency (f) =
T
1
= Hz
4 × 10−8
= 25 × 106 Hz
= 25 MHz
Practice Problems 3.
An ultraviolet radiation has a wavelength of 300Å. If the velocity of electromagnetic wave is 3 × 108 ms–1.
1.
Calculate (i) frequency (ii) time period.

111
Solution :
Velocity of electromagnetic wave (v) = 3 × 108 m/s
Wavelength of ultraviolet radiation = 300 Å = 300 × 10–10 m
v 3 × 108 m/s
(i) Frequency (f) = = −
= 1016 Hz
λ 300 × 10 10
m
1 1
(ii) Time period ( T ) = = 16 = 10−16 s
f 10 Hz
The wavelength of the vibrations produced on the surface of water is 2 cm. If the wave velocity is 16 ms–1,
2.
calculate (i) no. of waves produced in one second (ii) time required to produce one wave.
Solution :

Wavelength of vibrations (l) = 2 cm
(l) = 2 × 10–2 m

Velocity of the wave (v) = 16 ms­–1


Let f number of wave produced in one second.
(i) From the relation (v) = fl
⇒ 16 ms–1 = f × 2 × 10–2 m
16
\ f = −2
s −1
2 × 10
= 800 s–1 = 800 Hz
1
(ii) Time to produce one wave =
f
1
= = 0.00125 s
800 Hz

Practice Problems 4.
1.
A continuous progressive transverse wave of frequency 8 Hz moves across the surface of a ripple tank. (a) With
reference to the frequency, describe the movement of water on the surface. (b) If the wavelength of transverse
with wave is 32 mm, calculate the speed with which wave travels across the surface of water.
Solution :
(a) Frequency of wave (f) = 8 Hz
Þ 8 continuous progressive waves move over the surface of water in one second.
(b) Wavelength of transverse wave = l = 32 mm = 32 × 10–3 m
Speed = v = ?
From relation, (v) = fl
= 8 × 32 × 10–3 ms–1
= 256 × 10–3 ms–1
= 256 × 10–3 × 102 cms–1
= 25.6 cms–1
2.
A thin metal plate is placed against the teeth of cog wheel. Cog wheel is rotated at a speed of 120 rotations per
minute and has 160 teeth. Calculate.
(i) frequency of node produced.
(ii) speed of sound, if wavelength is 1.05 m.
(iii) what will be the effect when speed of cog wheel is doubled?

112
Solution :

Frequency of rotation of cog wheel = 120 rotations per minute
120
f = = 2 rotations per second.
60

No. of teeth on the wheel = n = 160
(i) Frequency of nod = 160 × 2 = 320 Hz
(ii) Wavelength = l = 1.05 m
Speed of sound = v = ?
v = lf
= 1.05 × 320 ms–1 = 336 ms–1
(iii) When the speed of cog wheel is doubled, then frequency of nod increases and hence sound becomes shrill.
Practice Problems 5.
1.
A sound wave of wavelength 1/3 m has a frequency 996 Hz. Keeping the medium same, if frequency changes
to 1328 Hz. Calculate (i) velocity of sound (ii) new wavelength.
Solution :
1

Wavelength of sound l = m
3

Frequency f = 996 Hz
(i) Velocity of sound (v) = ?
From relation, v = fl
1 –1
ms  = 996 ×
3
= 332 ms–1

(ii) New frequency (f ¢ ) = 1328 Hz


Let l¢ is the new wavelength from relation, v = f ¢l ¢
Þ 332 = 1328 × l¢
332
Þ l¢ = = 0.25 m
1328
2.
Two tuning forks A and B of frequencies 256 Hz and 192 Hz respectively are vibrated in air. If the wavelength
of A is 1.25 m, calculate the wavelength produced by B.
Solution :
Frequency of tuning form (A) = fA = 256 Hz

Frequency of tuning forks (B) = fB = 192 Hz


Wavelength produced by A = lA = 1.25 m

Wavelength produced by B = lB = ?

Let v = Velocity Ua of the wave

Now, v = fAlA = fBlB

f A  A 256  1.25

Þ  lB =
fB 192
5
= = m 1.67 m

3

113
Practice Problems 6.
1.
The distance between one crest and one trough of a sea wave is 4.5 m. If the waves are produced at the rate of
240/min, calculate (i) time period, (ii) wave velocity.
Solution :

The distance between one crest and one trough of sea wave is 4.5 m.

\ Wavelength (l) = 9 m

Frequency of wave = 240/min
240 −1
= s

60

\ f = 4 s–1
1 1
(i) Time period = = = 0.25 s
f 4
(ii) Wave velocity (v) = fl
= 4 × 9 ms–1
= 36 ms–1
2.
The distance between three consecutive crests of wave is 60 cm. If the waves are produced at the rate of
180/min, calculate (i) wavelength (ii) time period (iii) wave velocity.

(Hint : 3 consecutive crests = 2 waves)
Solution :
(i) Distance between two consecutive crests = l
Þ Distance between 3 consecutive crests = 2 wavelength = 2 l = 60 cm
60
⇒λ= = 30 cm
2
(ii) Time period T = ?
Frequency = 180 min–1
\ f = 180 = 3 s −1
60 1 1
\ Time period= = = 0.33 s
f 3

(iii) Wave velocity v = ?


v = fl
= 3 × 30 cms–1 = 90 cms–1
Practice Problems 7.
1.
The diagram alongside shows a displacement distance graph of a
wave. If the velocity of wave is 160 ms–1, calculate (i) wavelength
(ii) frequency (iii) amplitude.
Solution :
(i) Wavelength (l) = Distance between two consecutive crests
= 1 – 0.2 m
= 0.8 m
(ii) Frequency f = ?
Velocity of wave (v) = 160 ms–1

114
v = fl
160
= f = 200 Hz
0.8
(iii) Amplitude = Maximum displacement from mean position = 10 cm.

2.
From diagram alongside calculate (i) velocity of P and
Q (ii) frequency of P, when frequency of Q is 512 Hz.
Assume that both wave are travelling in same medium.
Solution :
(i) For wave Q


The wavelength (l) = Distance between two consecutive crests.

= 0.5 – 0.1 = 0.4 m

Frequency (f) = 512 Hz

Velocity (v) = fl
= 512 × 0.4 ms–1
(ii) For wave P = 204.8 ms–1
Wavelength = l¢ = 1.0 – 0.2 = 0.8 m
Since both the wave P and Q are moving in the same medium with the same velocity 204.8 ms–1
For wave P.
Velocity (v) = 204.8 ms–1
Frequency of wave P = f ¢ = ?
v = f ¢l¢
v 204.8
f ¢ = =
λ′ 0.8
= 256 Hz

EXERCISE 1
(A) Objective Questions
Multiple Choice Questions.
Select the correct option:
1. In case of longitudinal waves, the particles of the medium vibrate :
(a) in the direction of wave propagation
(b) opposite to the direction of wave propagation
(c) at right angles to the direction of wave propagation
(d) none of the above
2. A longitudinal wave consists of :
(a) crest and trough in the medium (b) compression and rarefaction in the medium
(c) both (a) and (b) (d) neither (a) nor (b)
3.
The longitudinal waves can propagate only in
(a) solids (b) liquids (c) gases (d) all of these
4.
A part of the longitudinal wave in which particles of the medium are closer than the normal particles is called:
(a) rarefaction (b) crest (c) trough (d) compression

115
5.
A part of longitudinal wave in which particles of the medium are farther away than the normal particles is called:
(a) rarefaction (b) trough (c) compression (d) crest
6.
In the region of compression or rarefaction in a longitudinal wave, the physical quantity which does not change
is :
(a) pressure (b) mass (c) density (d) volume
7.
The wavelength is the linear distance between the :
(a) two consecutive compressions (b) two consecutive rarefactions
(c) one compression and one rarefaction (d) both (a) and (b)
8.
The number of oscillations passing through a point in unit time is called :
(a) vibration (b) frequency (c) wavelength (d) amplitude
9.
The SI unit of frequency is
(a) hertz (b) gauss (c) decibel (d) none of these
10. If the frequency of a wave is 25 Hz, the total number of compressions and rarefactions passing through a point
in 1 second is :
(a) 25 (b) 50 (c) 100 (d) none of these
11. Which of the following is an elastic wave?
(a) light wave (b) radio wave (c) sound wave (d) microwave
Ans.
1. (a) in the direction of wave propagation 2. (b) compression and rarefaction in the medium
3. (d) all of these 4. (d) compression
5. (a) rarefaction 6. (b) mass
7. (d) both (a) and (b) 8. (b) frequency
9. (a) hertz 10. (b) 50
11. (c) sound wave

(B) Subjective Questions


1. (a) What do you understand by the term sound energy?
(b) State three conditions necessary for hearing sound.
Answer :
(a) Sound is a form of energy that produces the sensation of hearing. Sound is produced when a body vibrates.
(b) Necessary conditions for hearing sound :
(i) There must be a vibrating body, capable of transferring its energy to its surroundings.
(ii) There must be a material medium to pick up the energy and then propagate it in the forward
direction.
(iii) There must be receiver, so as to receive the sound vibrations and then transmit them to the brain for
final interpretation, such as human ear.
2.
Describe briefly an experiment to prove that vibrating bodies produce
sound.
Answer :

On striking the tuning fork with a rubber hammer and hold it close to ear.
A sound is heard. Now take a freely suspended pith ball and touch one
end of the tuning fork (which is already hit with a rubber hammer) to it.
It is observed that the pith ball repeatedly flies outward. Thus we can say
that sound is produced by a vibrating body.

116
3. (a) What do you understand by the term infrasonic vibrations?
(b) What do you understand by the term sonic vibrations? State the range of sonic vibrations for the human ear.
Answer :
(a) Those vibrations whose frequency is less than 20 Hz are known as infrasonic vibrations.
(b) Those vibrations whose frequency is from 20 Hz to 20,000 Hz are known as sonic vibrations. Sonic vibrations
are also known as audio vibrations.
4. (a) What do you understand by the term ultrasonic vibrations?
(b) Name three animals which can hear ultrasonic vibrations.
Answer :
(a) Those vibrations whose frequency is more than 20,000 Hz and are not perceived by human ear, are known
as ultrasonic vibrations.
(b) Dogs, bats and dolphins can hear ultrasonic vibrations.
5.
How do bats locate their prey during flight?
Answer :
Bats emit ultrasonic vibration during flight. These vibrations on striking the prey are echoed back to the bats. On
receiving the echo bats hit on the target. In this way bats locate their prey during flight.
6.
What is Galton’s whistle? To what use is it put?
Answer :
Galton’s whistle is a special type of whistle which can emits ultrasonic vibrations between 20,000 Hz to 40,000
Hz. Vibrations of this range cannot be perceived by human ears, but can easily perceived by dogs. Thus, the dogs
can be trained by hearing the sound from Galton’s whistle.

It is very useful, if any intruders enter in someone’s house, on blowing the whistle the dog will attract intruder
as intruder not able to hear the sound.
7.
State four practical uses of ultrasonic vibrations.
Answer :
Four practical uses of ultrasonic vibrations :
(i) They are used in the ultrasound scanning of internal organs of human body.
(ii) They are used for dissipating fogs on the runways at the airports.
(iii) They are used in SONAR (sound navigation and ranging) to detect and find the distance of objects under
water.
(iv) They are used in dish washing machines. In these machines water and
detergent are vibrated with ultrasonic vibrators. The vibrating particles
of the dissolved detergent rub against the plates and clean them.
8. Describe an experiment to prove that material medium is necessary for the
propagation of sound.
Answer :

A material medium is necessary for the propagation of sound can be proved
with the help of following experiment.
Things required : An electric circuit consisting of a cell, a switch and an
electric bell is arranged inside a bell-jar, which stands on the platform of
an evacuating pump.
Procedure : The switch of the bell is pressed to close the electric circuit

sound is heard when there is air within the bell-jar. The intensity of the

117
sound goes on decreasing, and no sound is heard when the air is completely removed from bell-jar. It is because,
the air which acts as a medium for the propagation of sound energy is removed. This experiment clearly proves
that material medium (in the present case air) is necessary for the propagation of sound energy.
9.
Why do astronauts talk to each other through radio telephone in space?
Answer :
Sound cannot travel through vacuum. There is no material medium in the space for propagation of sound wave.
Hence astronauts talk to each other through radio telephone in space. When they talk through radio telephone,
the electromagnetic waves are produced. These waves do not required any material medium for the propagation
and hence can be picked up by the radio receiver.
10. Define the terms : (i) wavelength (ii) amplitude (iii) frequency.
Answer :
(i) Wavelength : The linear distance between two consecutive particles of a vibrating medium in the same
phase is called its wavelength. It is denoted by the Greek letter l (lambda).
(ii) Amplitude : The maximum displacement of a vibrating particle about its mean position is called amplitude.
(iii) Frequency : The number of complete vibrations executed by a vibrating particle of a medium about its
mean position in one second is called its frequency.
11. State four differences between the sound wave and the light wave.
Ans. Difference between the sound wave and the light wave.
Sound waves Light waves
(i) They are produced due to vibrations of various They are produced from the electrons in an excited
objects in a material medium. state.
(ii) They travel in air at a speed of 332 ms–1 at 20°C. They travel at a speed of 3 × 108 ms–1 in air.
(iii) They cannot travel through vacuum and always They can travel through vacuum.
require some material medium for propagation.
(iv) Their velocity changes with the change in Their velocity does not change with the change in
temperature, humidity etc. temperature, humidity etc.
12. What is meant by the term wave motion?
Answer :
The transference of energy when the particles of a medium, move about their mean position is called wave motion.
13. State the relation between the wavelength and the frequency?
Answer :
Considering a wave propagating through a medium,
Wave velocity = v; Frequency = f; Wavelength = l; Time period = T
In the time T distance covered by the wave = l
λ
\ In 1 second distance covered by the wave =
T
But distance covered by the wave in one second = velocity of wave
λ

\ Velocity of wave (v) =
T
1

We know that, = f
T
1

\  v = × l = f l
T
Thus,  v = f × l
14. What kinds of the waves are produced in solids, liquids and gases?

118
Answer :
Elastic waves or material waves are produced in solids, liquids and gases.
15. The sound of an explosion on the surface of lake is heard by a boatman 100 m away and a diver 100 m below
the point of explosion.
(a) Of the two persons mentioned (boatman and diver), who would hear the sound first?
(b) Give reason for your answer in (a)
(c) If the sound takes ‘t’ seconds to reach the boatman, approximately how much time it will take to reach
the diver?
Answer :
(a) Diver would hear the sound first.
(b) The diver hear the sound first because velocity of sound in water (1450 ms–1) is more than the velocity of
sound in air (332 ms–1)
(c) If t is the time taken by sound to reach the boatman than time taken to reach the sound would be around
4.36 times less than the time taken by sound to reach the boatman.
16. What is the approximate value of speed of sound in iron as compared to that in air? Illustrate your answer with
a simple experiment.
Answer :
The speed of sound in iron is around fifteen times the speed of sound in air.
The speed of sound in iron = 5100 ms–1
Speed of sound in air = 332 ms–1
If we put our ear to, rail track, we can hear the sound of the train through metal. But at the same time, when
we stand near by the railway track we are not able to hear the sound.
This is because when we put our ears to rail track, the sound travels through iron. But on standing position, sound
travels through air and due to smaller speed of sound in air as compared to iron, we are not able to hear to the
sound.
17. How does a bat avoid obstacles in its way when in flight?
Answer :
Bat makes a series of twittering sound, so high pitched that human ear cannot hear. These sound waves strike
against the object in their path of flight and send back echoes to the bat’s ear. The echoes tell the bat how they
must turn in the air to avoid colliding with the object or with one another. In this way bat avoid obstacles in its
way.
18. A continuous disturbance is created on the surface of water in a ripple tank with a small piece of cork floating
on it. Describe the motion of the cork. What does the motion of the cork tell about the disturbance?
Answer :
Cork will move up and down about the mean position. It tells us that only the disturbance/energy is transferred
from one particle to the other but particle of the medium do not move from one position to the other.
19. Draw a displacement-time graph for water wave.
Answer :



Displacement time graph for water wave.
EEEEE

119
10 Electricity and Magnetism–1

UNIT I
Electric Potential and Electric Circuits

EXERCISE 1
1.
There is a positively charged sphere A and negatively charged sphere B, such that they are brought in electrical
contact by a copper wire. Answer the following questions :
(a) Which sphere is at higher potential before electrical contact on the basis of convention?
(b) Which sphere is at lower potential before electrical contact on the basis of convention?
(c) In which direction conventional current flows?
(d) In which direction electronic current flows?
(e) What is potential of the spheres after electrical contact?
Answer :
(a) On the basis of convention, positively charged sphere A is at higher potential before electrical contact.
(b) Negatively charged sphere B is at lower potential before electrical contact.
(c) Conventional current flows from ‘A’ higher potential to lower potential B.
(d) Electronic current flows from B to A.
(e) After electrical contact both the sphere will be at the same potential.
2. (a) What do you understand by the term electric potential?
(b) Define electric potential in terms of energy spent.
(c) State the unit of electric potential and define it.
Answer :
(a) The amount of work done in moving a unit positive charge from infinity to a given point in an electric
field is called electric potential.
(b) Amount of energy spent in moving a unit positive charge from infinity to a given point in an electric field
is called electric potential.
(c) SI unit of electric potential is volt.
One volt : When one joule of work is done in moving 1 coulomb charge from one point to another point
in an electric field, then the potential difference between these points is one volt.
3. (a) What do you understand by the term quantity of electric charge?
(b) State SI unit of electric charge and define it.
(c) How many electrons constitute one unit electric charge in SI system?
Answer :
(a) The number of charges (electrons) which drift from higher to lower potential is called quantity of charge.
(b) SI unit of electric charge is coulomb.
One coulomb : The quantity of electric charge which will deposite 0.00118 g of silver on the cathode,
when passed through silver nitrate solution is called one coulomb.
(c) 6.25 × 1018 electrons constitute one unit (1C) electric charge in SI system.

120
4. (a) What do you understand by the term electric current?
(b) State and define the SI unit of electric current.
(c) State the relation between electric current, number of electrons moving in a circuit and time in seconds.
Answer
(a) The rate of flow of charge in an electric circuit is called electric current. It is denoted by the letter (I).
(b) SI unit of electric current is ampere (A) when one coloumb charge flows through an electric circuit in one
second, then the current flowing through the circuit is said to be one ampere.
1 coulomb
1 ampere =
1 second
(c) If Q is the charge (in coulombs) flowing through conductor in time t (in seconds) such that the current I
Q
flows through the conductor then rate of flow of charge =
t
We know that the rate of flow of charge = I = Electric current
Q
⇒I=
t
5.
How electric current flows in (i) solids (ii) liquids?
Answer
(i) In solids the positive charges are associated with atomic nuclei. As the nuclei are firmly packed and closely
held by the interatomic forces, therefore positive charges cannot drift. But negative charge electrons are
not held firmly. Thus when a potential difference, however small, is applied they start drifting from higher
potential to lower potential.
The continuous drift of electrons, through the body of a solid conductor constitutes the current.
(ii) Within a liquid no electrons move. However, when a negatively charged and positively charged electrodes
are placed in a liquid, it sets up an electric field. Under the influence of electric field, the positively charged
ions migrate towards the negatively charge electrodes and vice versa.
Thus, simultaneous movement and discharge of positive and negative ions in the opposite directions
constitutes the current in the liquids.
6. (a) Define the term potential difference.
(b) How is potential difference related to work done and quantity of charge?
Answer
(a) The amount of work done in moving a unit positive charge from one point to another point in an electric
field is called potential difference.
(b) If Q units of charge are moved from one point to another point, such that W is the work done and V is
the potential difference then
Work done in moving Q units of charge = W
W
Work done in moving one unit of charge = .
Q
But work done in moving one unit of charge = Potential difference = V
W
\ V =
Q
Numerical Problems

Practice Problems 1.
1.
A charge of 5000 C flows through an electric circuit in 2 hours and 30 minutes. Calculate the magnitude of
current in circuit.
121
Solution :

Charge Q = 5000 C

Time, t = 2 hours 30 minutes

= 2 × 60 + 30 = 150 minutes

= 150 × 60 seconds

= 9000 seconds

Electric current (I) = ?
Q 5000 C

=
Electric current (I) = = 0.555 A
t 9000 s
2.
A charge of 8860 C flows through an electric circuit in 2 min and 40 s. Calculate the magnitude of current in
circuit.
Solution :

Charge Q = 8860 C

Time t = 2 min 40 sec

= (2 × 60 + 40) sec

t = 160 s

Electric current (I) = ?
Q 8860 C 443

I = = = A
t 160 s 8

I = 55.375 A
Practice Problems 2.
A battery can supply a charge of 25 × 104 C. If the current is drawn from battery at the rate of 2.5 A, calculate
1.
the time in which battery will discharge completely.
Solution :

Charge (Q) = 25 × 104 C; Current (I) = 2.5 A; time (t) = ?
Q

From the relation (I) =
t
Q 25 × 104 C

Or, t = = = 100000 s
I 2.5 A

2.
A dry cell can supply a charge of 800 C. If continuous current of 8.0 mA is drawn, calculate the time in which
cell will discharge completely.
Solution :
Charge Q = 800 C; Electric current (I) = 8.0 mA = 8 × 10–3 A

Time t = ?
Q

From the relation (I) =
t
Q 800 C

⇒ t = = = 100000 s
I 8 × 10−3 A

122
Practice Problems 3.
1.
Calculate the total number of electrons flowing through a circuit in 20 mins and 40 s, if a current of 40 µA flows
through the circuit. [1 e– = 1.6 × 10–19 C]
Solution :

Electric current (I) = 40 mA = 40 × 10–6 A

Charge on one electron = 1.6 × 10–19 C

Let number of electrons = n

Time (t) = 20 min 40 sec = 20 × 60 + 40 s = 1240 s

Q

From the relation (I) =
t

Q = I t
= 40 × 10–6 × 1240 C

= 49600 × 10–6 C

= 4.96 × 10–2 C


Now, Q = ne; where e = charge on 1 electron

4.96 × 10–2 = n × 1.6 × 10–19
4.96 × 10−2
⇒ n =
1.6 × 10−19

⇒ n = 3.1 × 1017


\ No. of electrons = 3.1 × 1017
4 × 1020 electrons flow through a circuit in 10 hours. Calculate magnitude of current. [1 e– = 1.6 × 10–19 C]
2.
Solution :

Let number of electrons = n = 4 × 1020 electrons

Charge on one electron = 1.6 × 10–19 C

Time t = 10 hours

= 10 × 60 × 60 s = 36000 s

Charge = C = ne = 4 × 1020 ×1.6 × 10–19 C
= 64 C
Q 64 C

= Electric current (I) =
t 36000 s
64
= × 10−3 A

36
= 1.77 × 10–3 A


= 1.77 mA
Practice Problems 4.
1.
What is the electrical potential at a point in an electric field when 24 J of work is done in moving a charge of
96 C from infinity?
Solution :

Work done = W = 24 J; Charge (Q) = 96 C; Electric potential = ?

123
W 24 1
V = = = V = 0.25 V
Q 96 4
2.
A charge of 75 C is brought from infinity to a given point in an electric field, when amount of work done is
3.75 J. Calculate the electrical potential at that point.
Solution :
Work done (W) = 3.75 J; Charge (Q) = 75 C; E
lectric potential = ?
W 3.75

= V From the relation, V =
Q 75

= 0.05 V
Practice Problems 5.
1.
A work of 25 J and 30 J is done when 5 C charge is moved first to point A and then to point B from infinity.
Calculate the potential difference between points A and B.
Solution :
W 25 J

Potential at point A (VA) = = = 5V
Q 5C

30 J

Potential at point B (VB) = =6V
5C


\ Potential difference between A and B = VB – VA


= 6 V – 5 V


= 1 V

2.
A charge of 25 C is moved from infinity to points A and B in an electric field when the work done to do so is
10 J and 10.5 J respectively. Calculate the potential difference between the points A and B.
Solution :
10 J

Potential at point A ==
= 0.4 V
VA
25 C
10.5 J

Potential at point B = VB = = 0.42 V
25 C

Potential difference between point A and B = VB – VA

= 0.42 – 0.40

= 0.02 V

UNIT II
EXERCISE 2
(A) Objective Questions
Multiple Choice Questions.
Select the correct option
1. SI unit potential difference is :
(a) coulomb (b) kelvin (c) volt (d) ampere

124
2.
Current in a circuit flows :
(a) in a direction from high potential to low potential
(b) in a direction from low potential to high potential
(c) in a direction of flow of electron
(d) in any direction
3.
In a metallic conductor, electric current is thought to be due to movement of :
(a) ions (b) amperes (c) electrons (d) protons
4. –19
Assuming that the charges of an electron is 1.6 × 10 coulombs, the number of electrons passing through a
section of wire per sec, when the wire carries a current of one ampere is :
(a) 0.625 × 1019 (b) 1.6 × 10–19 (c) 1.6 × 1019 (d) 0.627 × 10–17
5.
Which of the following is best conductor of electricity?
(a) copper (b) gold (c) platinum (d) silver
Ans.
1. (c) volt 2. (a) in a direction from high potential to low potential
3. (c) electrons 4. (a) 0.625 × 1019 5. (d) silver

(B) Subjective Questions


1.
What do you understand by the term electric cell?
Ans. An electric cell is the simplest form of arrangement to maintain a constant potential difference between conductors.
In the cell chemical energy maintains one end at higher potential and other at lower potential, thereby making
the electrons to drift from higher potential to lower potential.
2.
Draw a neat and labelled diagram of simple voltaic cell showing clearly the direction of flow of conventional
current and direction of flow of electrons.
Ans. Electric charge flows from copper to zinc and this
current is called conventional current, whereas the
actual flow of electrons is from zinc to copper
which constitutes electronic current.

3.
Briefly describe the theory of simple voltaic cell.
Ans. Theory of simple voltaic cell.

Amongest the zinc and copper plates zinc is more electropositive
as compared to copper in electro chemical series. The dilute
sulphuric acid is used as an electrolyte in the ionised state.

125

 + 2−
H 2 SO 4  2H + SO 4

When zinc plate is in contact with (H+) hydrogen ions, it



being more electropositive, ionises to form zinc ions and free
electrons.
Zn → Zn 2 + + 2e−
The free electrons so formed, take the passage of least
resistance and hence, move out in the external circuit through
the copper wires. The zinc ions, however enter in the dilute
sulphuric acid.
Since Zn2+ ions are positively charged, they repel H+ ions in
the acid solution, with the result that, H+ ions, which form
nascent hydrogen.
2H+ + 2e– ® 2H
The nascent hydrogen atoms so formed unit to form molecular
hydrogen.
2H ® H2
Thus, it is clear that free electrons actually drift from zinc to copper in external circuit, and hence, current should
flow from zinc to copper.
4.
What do you understand by the following terms?
(i) electric circuit (ii) closed electric circuit (iii) open electric circuit.
Answer :
(i) Electric circuit : The path along which electric current flows is known as electric circuit.
(ii) Closed electric circuit : When the path of an electric circuit starting from one terminal of the cell, ends
at the other terminal of cell without any break, then such a circuit is called closed circuit.
(iii) Open electric circuit : When the path of electric current, starting from one terminal of cell is broken at
some points, then such a circuit is called open electric circuit.
5.
State two conditions necessary for a circuit, such that electric current flows through it.
Answer :
For the flow of electric current through a circuit necessary two conditions are as follows :
(i) Electric circuit must be closed or complete.
(ii) Every part of the circuit is a conductor.
6. Draw a neat diagram showing (i) closed electric circuit (ii) open electric circuit.
Answer :
(i)

Closed electric circuit

126
(ii)

Open electric circuit

7.
Name four electric conductors and four electric insulators.
Answer :
Examples of electric conductors : Silver, copper, aluminium and iron.
Examples of electric insulators : Dry wood, plastic, rubber and nylon
8. (a) What do you understand by the term electric resistance?
(b) Why does the filament of an electric bulb in an electric circuit get white hot, but not the connecting wires?
Answer :
(a) The obstruction offered to the passage of electric current by a material is called resistance of material.
(b) Filament of an electric bulb is made up of tungsten having high resistance. On passing electric current
through it, electric energy changes into heat energy. So much heat is produced in the filament that bulb
becomes white hot and gives off light. The resistance of connecting wires have very low resistance so they
are not get heated.
9.
Is it correct to say that a resistance wire is an insulator or a bad conductor? Explain your answer.
Answer :

It is correct to say that a resistance wire is an insulator or bad conductor. From resistance it is implied that a
given material will conduct electric current, but will also offer obstruction to the passage of electric current.
10. (a) What do you understand by the term series circuit?
(b) State two characteristics of resistances in the series circuit.
(c) Draw a diagram showing two bulbs connected in series to a dry cell.
Answer :
(a) When a number of resistances are connected in an electric circuit in such a way that the positive of one
resistor acts as negative of the other resistor, then the resistances are said to be connected in series.
(b) Two characteristics of resistances in series circuit :
(i) The sum total of resistances in series increases with the increase in number of resistors.
(ii) The potential difference remaining constant, the current in series circuit decreases, with the increase
in number of resistors in series.
(c)

+ –

Two bulbs connected in series to a dry cell.


11. (a) What do you understand by the term parallel circuit?
(b) State two characteristics of resistance in the parallel circuit.
(c) Draw a diagram showing two bulbs connected in parallel to a dry cell.

127
Answer :
(a) When a number of resistances are connected in an electric circuit in such a way that all of them are connected
to common positive and common negative terminal of a cell, then the resistance are said to be connected
in parallel.
(b) Two characteristics of resistances in parallel :
(i) The sum total of resistances in parallel decreases with the increase in number of resistors.
(ii) Each resistor in parallel functions independently with respect to the other resistor in parallel.
(c)

Two bulbs connected in parallel to a dry cell.

EEEEE

128
11 Electricity and Magnetism–2

EXERCISE

(A) Objective Questions


Multiple Choice Questions.
Select the correct option.
1. A bar magnet is rubbed on a bar of steel along its length 20 times. The bar of steel gets magnetised due to the
process of :
(a) induction (b) conduction (c) friction (d) none of these
2.
The magnetic strength of a bar magnet is :
(a) maximum at its centre (b) same along the magnet
(c) maximum near its ends (d) none of these
3.
The surest test of magnet is :
(a) repulsion (b) attraction (c) induction (d) none of these
4.
Nickel is a :
(a) ferromagnetic substance (b) paramagnetic substance
(c) diamagnetic substance (d) none of these
5.
The substance which form a strong temporary magnet is :
(a) steel (b) platinum (c) soft iron (d) manganese
6. The place around a magnet where its influence can be detected is called :
(a) magnetic lines of force (b) magnetic pole
(c) magnetic field (d) magnetic space
Ans.
1. (c) friction 2. (c) maximum near its ends
3. (a) repulsion 4. (a) ferromagnetic substance
5. (c) soft iron 6. (c) magnetic field

(B) Subjective Questions


1.
What do you meant by the term pole of a magnet? Magnetically speaking, what is the difference between a piece
of brass, a piece of soft iron and a piece of lode-stone?
Answer :

The ends of a magnet where the magnetic force is maximum, are called poles of the magnet or magnetic poles.
Each magnet has two magnetic poles. Poles of magnets occur in pairs and can not be separated.

Brass is not a magnetic substance and it is not affected by magnetic field. It does not stick to a magnet.

Soft iron is a ferromagnetic substance and gets strongly attracted towards a magnet. Soft iron can not attract other
magnetic substances unless gets magnetised. Lodestone is a naturally occurring magnet made of iron oxide. It can
attract other magnetic substance.
2. (a) What are magnetic and non-magnetic substances? Give at least two examples of each.

129
(b) Fill the blank spaces in the table given below :
Nature of bar Action on compass needle
North Pole South Pole
Non-magnetic No action ______________
_____________ attracted attracted

North pole of a bar magnet _____________ _____________


_____________ attracted repelled

Answer :
(a) The substances which are get attracted to the magnet or affected by the magnetic field are known as magnetic
substances.
Example of magnetic substances are iron, cobalt and nickel.
Those substances that are not affected in any way by a magnet are called non-magnetic substances. Rubber,
plastic, wood, copper are examples of non-magnetic substances.
(b)
Nature of bar Action on compass needle
North Pole South Pole
Non-magnetic No action No action
Soft iron bar attracted attracted
North pole of a bar magnet repelled attracted
South pole of a bar attracted repelled

3.
Define : magnetic field, magnetic meridian, geographical meridian, declination and magnetic equator.
Answer :
Magnetic field : The space around a magnet within which the magnet has its influence is called magnetic field.
Magnetic meridian : The vertical plane containing the magnetic axis of a freely suspended magnet at rest, under
the action of magnetic intensity of earth is called magnetic meridian.
Geographic meridian : The vertical plane which contains geographic north and south poles of earth at a given

place is called geographic meridian. It is different at different places on the surface of the earth.
Declination : The phenomenon due to which the earth’s geographic meridian is inclined to earth’s magnetic
meridian is called declination.
Magnetic equator : An imaginary line bisecting the effective length of a bar magnet at a right angle is called

its magnetic equator.
4.
Define : Isogonic line, agonic line isoclinic line.
Answer :
Isogonic line : A line which join all the places on earth, having same angle of declination is called isogonic line.
Agonic line : A line which joins all the places on earth, having zero angle of declination is called isogonic line.

Isoclinic line : A line joining all the places on the globe, having same angle of dip or inclination is called isoclinic

line.

130
5.
State briefly (a) the molecular theory of magnetism, (b) the modern views on magnetism.
Answer :
(a) Molecular theory of magnetism

Each molecule of a magnetic substance, whether it is magnetised or unmagnetised is an independent magnet.


In a magnetic substance the molecules are arranged in an order so as to produce an external effect. In this
order, all the north poles of the molecules of the magnetised substances point to one direction and all the
south poles point to a direction opposite to that to which their north poles points.
In an unmagnetised substance, the molecules are not arranged in any order, so they neutralise the magnetic
forces of each other.
Modern view on magnetism
Atoms consists of negatively charged particles (electrons) which revolve around the positively charged
nucleus. Electrical current loops are formed in an atom due to the circulation of these electrons. Each current
loop behaves as a magnetic dipole and hence produce magnetic
field. Electrons are also spinning like tops and this adds further
magnetism to the atom.
6.
Describe various methods of magnetising a piece of iron.
Answer :
Methods of magnetisation
1. Single Touch Method : Iron bar to be magnetised is
placed flat on the table. A permanent bar magnet is taken
and its one pole is placed on one end of the iron piece.
The bar magnet is then drawn to the other end, keeping
it in the inclined plane position as shown in the figure.
The permanent magnet is then lifted and the process is
repeated for 20-30 times. The iron piece is then turned
over and other side is also magnetised in the same way.
Its starting end gets the same polarity as the polarity of
the magnet touching it. The polarity at the other end of
the iron bar is opposite to that of magnetising pole.

2. Divided touch method : The iron piece to be magnetised


is placed flat on a table. The piece of iron is supported
by two other magnets. Opposite poles of two strong bar
magnets are placed inclined at the centre of the strip. The
magnets are moved apart as shown. After 20-30 strokes
the strip is turned upside down and is also magnetised
in the same way.

3. Electrical method of magnetisation : The iron piece to


be magnetised is placed inside a long coil of an insulated
copper wire and a strong direct current is passed through
the coil for some time. The iron piece is magnetised.

131
7. What is magnetic induction? Explain it giving a suitable experiment.
Answer :

The phenomenon due to which a piece of steel or iron behaves like a magnet when placed near a strong magnet
is called magnetic induction.


Take a freely suspended magnetic needle and bring near its south pole, the south end of a bar magnet. The needle
gets repelled. Move the south end of the bar magnet backward till it no longer affects the south end of magnetic
needle. Now place a flat piece of iron in between the bar magnet and magnetic needle. It is observed that south
end of needle is repelled. Remove the iron piece, it is observed that needle comes back to its original position.
Now repeat the experiment, but remove bar magnet instead of iron piece. We will observe that needle does not
get repelled and remains continuously in its original position.

From this experiment, it is clear that soft iron piece behaves as magnet only when a bar magnet is placed near
it, when a bar magnet is removed soft iron piece loses its magnetism.
8.
Repulsion is the surer test of magnetic condition of a body than attraction. Explain.
Answer :

The magnetic attraction can be caused between two unlike poles of the two magnets or between the magnet and
the magnetic substances such as iron, cobalt and nickel. However, repulsion is caused only when two similar
poles approach each other. Thus repulsion is the surest test
of magnetism.
9.
Give short account of the earth’s magnetic field.
Answer :

When a bar magnet is suspended freely, then it aligns itself
along geographical north-south direction i.e., north pole of
the magnet points towards the geographical north and south
pole of the magnets points towards the geographical south.
This experiment suggest that the earth behaves like a bar
magnet. The cause of earth’s magnetism is yet not very clear.
However, for simplicity we can have following assumptions.
1. A hugue bar magnet is buried at the centre of the earth.
2. The south end of the earth’s magnet is towards the
earth’s geographic north and vice-versa.
3. The axis of earth’s magnet is not in line with the geographic axis, but
makes a small angle with it.
10. Give the various methods for demagnetising a magnet.
Answer :

The various methods for demagnetising a magnet are as follows :
(i) Electrical Method : An insulated copper coil is wound around a card
board tube and inside it a permanent bar magnet is placed. The coil is

132
placed in East-West direction and its ends are connected to a step down transformer. The alternating current
is switched on for one minute, the bar magnet gets demagnetised.
(ii) By Rough Handling : When a magnet is rough-handled or hammered, it gets completely demagnetised.
(iii) By heating : When a bar magnet is heated to red hot temperature and then allowed to cool, it loses its
magnetism.
(iv) By induction : When a given magnet is placed in contact with another similar magnet, such that their
similar poles are facing each other both the magnets get demagnetised in a couple of days.
(v) By self-Induction : A single bar magnet has a tendency to lose its magnetism.
11. The Fig. shows a freely suspended magnet in rest position. Copy the diagram and on it show :
(a) Angle of declination
(b) Angle of dip.

Answer :
(a) and (b)

12. (a) Since every iron atom is a tiny magnet, why are not all iron bar magnets?
(b) If a magnet is carefully broken into two pieces as shown in Fig. (i), how does the magnetic strength of
each piece compare with that of original magnet? If another magnet is carefully broken in half along its
long axis shown in Fig. (ii), how would the strength of each piece compare with that of original magnet?

(i) (ii)

Answer :
(a) Every atom of iron bar is an independent tiny magnet. In an unmagnetised
iron bar each atoms (tiny magnets) are not arranged in any order.
So they neutralise the magnetic effect of each other. As a result, any
unmagnetised iron piece does not behave as a magnet.

133
(b) (i) When a magnet is cut into two equal parts as shown in figure (i) then pole strength of each piece
remains same as that of original magnet.
(ii) When a magnet is cut into two equal parts as shown in figure (ii) then pole strength of each pole
is half as that of the pole strength of original magnet.
13. Draw the magnetic flux pattern near a bar magnet placed with its axis in the magnetic meridian and the south
pole pointing towards geographic north.

Answer :

Magnetic field lines when the south pole of a bar magnet points
towards geographic north.

14. Draw a clearly labelled diagram, to show how a steel bar is


magnetised by a divided touch method. A written description is
not required.
Answer :

15. Define the terms magnetic declination and dip with reference to freely suspended magnet.
(a) What do you understand by the terms magnetic meridian and geographic meridian?
(b) At what places on the earth will the angle of dip be (i) maximum and (ii) minimum?
Answer :

The angle through which freely suspended magnetic needle is inclined to the geographic axis is known as magnetic
declination.
Magnetic dip : The angle between the horizontal axis passing through freely suspended magnet and the direction
of earth’s magnetic field is called magnetic dip.
(a) Magnetic meridian : The vertical plane containing the magnetic axis of a freely suspended magnet at rest,
under the action of magnetic intensity of earth is called magnetic meridian.
Geographic meridian : The vertical plane which contains geographic north and south poles of earth at a
given place is called geographic meridian.
It is different at different places on the surface of earth.
(b) The angle of dip is maximum i.e., 90° at the magnetic poles.
The angle of dip is zero is i.e. minimum at magnetic equator.
134
16. (a) What are magnetic keepers? What are they used for?
(b) Explain the ‘molecular theory’ of magnetism with the help of a diagram.
Answer :
(a) A magnet has a tendency to lose its magnetism of its own if they are not properly taken care of. To keep
them safe, they are kept in magnetic keepers.
Magnetic keepers are piece of soft iron that are placed across both the poles of a bar magnet or horseshoe
magnet. The bar magnets are kept in pairs with their unlike poles along one side and separated by a piece
of wood.

Bar magnets Horse-shoe


Soft iron magnet
N S
keeper
Soft iron
A wooden keeper N S
piece S N Soft iron keeper
Use of soft iron keepers to prevent soft-demagnetisation of magnets

(b) According to Ewing molecular theory of magnetism


(i) Each molecule of a magnetic substance, whether it is magnetised or unmagnetised, is an independent
magnet.
(ii) In a magnetised substance, the molecules are arranged in an
order so as to produce an external effect. In this order, all the
north pole of the molecules of the magnetise substances point
to one direction and all their south poles point to a direction
opposite to which their north pole points.
(iii) In an unmagnetised substance, the molecules are not arranged in
any order, so they neutralised the magnetic forces of each other.

17. What do you understand by the term magnetic declination?


Answer :

The angle between the geographic meridian and magnetic meridian at a
given place is called declination.

EEEEE

135

You might also like